Elsiver NCLEX Critical Care

Ace your homework & exams now with Quizwiz!

The nurse has a prescription to administer whole blood to a client who does not currently have an intravenous (IV) line inserted. When obtaining supplies to start the blood infusion, the nurse should select an angiocatheter of at least which size? 1. 19 gauge 2. 21 gauge 3. 24 gauge 4. 26 gauge

19 gauge Blood components are usually administered with at least a 19-gauge needle, cannula, or catheter. Larger sizes (e.g., 18- or 16-gauge) may be preferred if rapid transfusions are given. Smaller needles can be used for platelets, albumin, and clotting factor replacement.

The nurse is performing an assessment on a client who sustained circumferential burns of both legs. Which assessment would be the initial priority in caring for this client? 1. Assessing heart rate 2. Assessing respiratory rate 3. Assessing peripheral pulses 4. Assessing blood pressure (BP)

Assessing peripheral pulses The client who receives circumferential burns to the extremities is at risk for altered peripheral circulation. The priority assessment would be to assess for peripheral pulses to ensure that adequate circulation is present. Although the respiratory rate and BP also would be assessed, the priority with a circumferential burn is assessment for the presence of peripheral pulses because the airway is not affected in this case.

The nurse should report which assessment finding to the health care provider (HCP) before initiating thrombolytic therapy in a client with pulmonary embolism? 1. Adventitious breath sounds 2. Temperature of 99.4°F (37.4°C) orally 3. Blood pressure of 198/110 mm Hg 4. Respiratory rate of 28 breaths/minute

Blood pressure of 198/110 mm Hg Thrombolytic therapy is contraindicated in a number of preexisting conditions in which there is a risk of uncontrolled bleeding, similar to the case in anticoagulant therapy. Thrombolytic therapy also is contraindicated in severe uncontrolled hypertension because of the risk of cerebral hemorrhage. Therefore, the nurse would report the results of the blood pressure to the HCP before initiating therapy.

A client sustained a burn injury at 7:00 a.m. The client's spouse states that before the burn, the client's body weight was 198 lbs. The health care provider has estimated that the total body surface area (BSA) burned is 83%. Using the Parkland (Baxter) formula (4 mL × kilograms of body mass × percent total BSA), the nurse determines that the total amount of intravenous lactated Ringer's solution that the client will receive by 3 p.m. of the same day on which the burn occurred is which value? Fill in the blank.

14940 The Parkland (Baxter) formula for estimating fluid requirements is 4 mL × kilograms of body mass × percent total BSA. Half of this total is administered in the first 8 hours after the burn. First, convert pounds to kilograms by dividing 198 lbs by 2.2, which equals 90. Therefore, 4 × 90 × 83 = 29,880 mL, divided by 2 = 14,940 mL.

A client who sustained an inhalation injury arrives in the emergency department. On initial assessment, the nurse notes that the client is very confused and combative. The nurse determines that the client is most likely experiencing which condition? 1. Pain 2. Fear 3. Anxiety 4. Hypoxia

Hypoxia After a burn injury, clients normally are alert. If a client becomes confused or combative, hypoxia may be the cause. Hypoxia occurs after inhalation injury and also may occur after an electrical injury. Although anxiety, fear, and pain may occur, confusion and combativeness are most likely associated with hypoxia.

Cardiopulmonary resuscitation (CPR) is immediately initiated on a client who is unconscious and has no pulse. A monitor is attached and it is determined that the rhythm is shockable, and defibrillation with 1 shock is delivered. Which action should the nurse plan to take next? 1. Defibrillate 1 more time, and then terminate the resuscitation effort. 2. Administer a bolus of fluid intravenously, and resume defibrillation attempts. 3. Perform CPR for 5 cycles, and then defibrillate again if the rhythm is shockable. 4. Perform CPR for 1 minute, assess, and then defibrillate up to 3 more times.

Perform CPR for 5 cycles, and then defibrillate again if the rhythm is shockable. If a client is unconscious and has no pulse, the nurse would shout for help (activate emergency response) and immediately initiate CPR. If the rhythm is shockable, a shock is delivered and then CPR is delivered for 5 cycles. This pattern is repeated 2 more times if the rhythm remains shockable. Treatment with medications is also done during this time to reverse the cause of the ventricular fibrillation. Each of the other options is incorrect.

The emergency department nurse is preparing to administer fomepizole to a client suspected of having ethylene glycol (antifreeze) intoxication. The nurse obtains the vial of medication and notes that the medication has solidified. Which action should the nurse take? 1. Discard the vial. 2. Run the vial under warm water. 3. Contact the health care provider. 4. Call the pharmacy and request another vial of medication.

Run the vial under warm water. Fomepizole is used in the treatment of known or suspected ethylene glycol (antifreeze) intoxication. It is administered via the intravenous (IV) route. It is diluted in at least 100 mL of 0.9% normal saline or 5% dextrose in water and administered over a 30-minute period. If the medication solidifies in the vial, the nurse should run the vial under warm water. The remaining options are inappropriate or unnecessary actions

A client who has been receiving total parenteral nutrition (TPN) by way of a central venous access device complains of shakiness and is diaphoretic. Based on these findings, the nurse should perform which assessment next? 1. Lung sounds 2. Mental status 3. Blood pressure 4. Blood glucose level

Blood glucose level Hypoglycemia is one of the potential complications associated with TPN. Shakiness and diaphoresis are signs of hypoglycemia; therefore, based on these findings, the nurse should first check the blood glucose level. Lung sounds may provide information about refeeding syndrome, which is a complication of TPN causing fluid overload. However, the assessment findings do not indicate that this is occurring. Mental status could be affected by hypoglycemia; however, the nurse has enough information to suspect this complication already and therefore should assess the blood glucose before assessing mental status. Blood pressure is not specifically related to the information in the question and the associated complication of TPN

The nurse is monitoring a child who is receiving ethylenediaminetetraacetic acid (EDTA) with BAL (British anti-Lewisite) for the treatment of lead poisoning. The nurse reviews the laboratory results for the child during treatment with this medication and is particularly concerned with monitoring which laboratory test result? 1. Cholesterol level 2. Blood urea nitrogen (BUN) level 3. Complete blood cell (CBC) count 4. Hemoglobin and hematocrit (H&H) levels

Blood urea nitrogen (BUN) level Kidney function tests should be monitored because EDTA is nephrotoxic. The calcium level should also be monitored because EDTA enhances the excretion of calcium. The remaining laboratory results are not the primary concern during treatment with EDTA.

The nurse is undergoing annual recertification in basic life support (BLS). The BLS instructor asks the nurse to identify the pulse point to use when determining pulselessness on an infant. Which response by the nurse identifies the most appropriate pulse point? 1. Radial 2. Carotid 3. Brachial 4. Popliteal

Brachial When assessing a pulse in an infant (younger than 1 year), the pulse should be checked at the brachial artery. This is because the relatively short, fat neck of an infant makes palpation of the carotid artery difficult. The pulses in the remaining options are also difficult to locate and palpate in an infant

The home health nurse is visiting an older client whose family has gone out for the day. During the visit, the client experiences chest pain that is unrelieved by sublingual nitroglycerin tablets given by the nurse. Which action by the nurse would be appropriate at this time? 1. Notify a family member who is the next of kin. 2. Drive the client to the health care provider's (HCP's) office. 3. Inform the home care agency supervisor that the visit may be prolonged. 4. Call for an ambulance to transport the client to the hospital emergency department.

Call for an ambulance to transport the client to the hospital emergency department. Chest pain that is unrelieved by rest and nitroglycerin might not be typical anginal pain but may signal myocardial infarction (MI). Because the risk of sudden cardiac death is greatest in the first 24 hours after MI, it is imperative that the client receive emergency cardiac care. Communication with the family or home care agency delays client treatment, which is needed immediately. The HCP's office is not equipped to treat MI.

A client receiving total parenteral nutrition (TPN) through a single-lumen central intravenous (IV) line is scheduled to receive an antibiotic by the IV route. Which action by the nurse is appropriate before hanging the antibiotic solution? 1. Ensure a separate IV access for the antibiotic. 2. Turn off the solution for 30 minutes before administering the antibiotic. 3. Flush the central IV line with 60 mL of normal saline before giving the antibiotic. 4. Check with the pharmacy to be sure the antibiotic can be given through the TPN solution line.

Ensure a separate IV access for the antibiotic. The TPN solution line is used only for the administration of the solution. Any other IV medication must be run though a separate IV access site; therefore, the remaining options are incorrect.

A client is brought into the emergency department in ventricular fibrillation (VF). The nurse prepares to defibrillate by placing defibrillation pads on which part of the chest? 1. The upper and lower halves of the sternum 2. Parallel between the umbilicus and the right nipple 3. The right shoulder and the back of the left shoulder 4. To the right of the sternum and to the left of the precordium

To the right of the sternum and to the left of the precordium The nurse would place 1 gel pad to the right of the sternum just below the clavicle and the other gel pad to the left of the precordium. The nurse would then place the electrode paddles over the pads. The remaining options identify incorrect positions.

The nurse is caring for a client receiving total parenteral nutrition (TPN) via a central line. What assessment should the nurse perform to detect the most common complication of TPN? 1. Vital signs 2. Auscultate lungs 3. Kidney function tests 4. Listen for bowel sounds

Vital signs The most common complication associated with TPN is infection. Monitoring the temperature would provide data that would indicate infection in the client. Monitoring kidney function tests would not provide information related to infection. Auscultating lungs assists in identifying if fluid overload is present, not infection. Listening for bowel sounds does not assist in assessing for infection in this case.

The nurse has just received a unit of packed red blood cells from the blood bank for transfusion to an assigned client. The nurse is careful to select tubing especially made for blood products, knowing that this tubing is manufactured with which item? Click on the image to indicate your answer.

3

The nurse is inserting an intravenous (IV) line into a client's vein. After the initial stick, the nurse would continue to advance the catheter in which situation? 1. The catheter advances easily. 2. The vein is distended under the needle. 3. The client does not complain of discomfort. 4. A backflash of blood is noted in the catheter.

A backflash of blood is noted in the catheter. The IV catheter has entered the lumen of the vein successfully when blood backflash shows in the IV catheter. The vein should have been distended by the tourniquet before it was cannulated. Client discomfort varies with the client, the site, and the nurse's insertion technique and is not a reliable measure of catheter placement. The nurse should not advance the catheter until placement in the vein is verified by blood return.

A client with a probable minor head injury resulting from a motor vehicle crash is admitted to the hospital for observation. The nurse leaves the cervical collar applied to the client in place until when? 1. The family comes to visit 2. The nurse needs to do physical care 3. The health care provider makes rounds 4. The results of spinal radiography are known

The results of spinal radiography are known There is a significant association between cervical spine injury and head injury. For this reason, the nurse leaves any form of spinal immobilization in place until spinal radiographs rule out fracture or other damage. The remaining options are incorrect.

The nurse is making initial rounds at the beginning of the shift and notes that the parenteral nutrition (PN) bag of an assigned client is empty. Which solution should the nurse hang until another PN solution is mixed and delivered to the nursing unit? 1. 5% dextrose in water 2. 10% dextrose in water 3. 5% dextrose in Ringer's lactate 4. 5% dextrose in 0.9% sodium chloride

10% dextrose in water The client is at risk for hypoglycemia; therefore, the solution containing the highest amount of glucose should be hung until the new PN solution becomes available. Because PN solutions contain high glucose concentrations, the 10% dextrose in water solution is the best of the choices presented. The solution selected should be one that minimizes the risk of hypoglycemia. The remaining options will not be as effective in minimizing the risk of hypoglycemia.

What electrolytes and amounts are usually contained in total parenteral nutrition (TPN) for an adult client without renal or hepatic impairment? Select all that apply. 1. Calcium 2 to 5 mEq 2. Sodium 1 to 2 mEq/kg 3. Magnesium 8 to 20 mEq 4. Potassium 1 to 2 mEq/kg 5. Phosphate 20 to 40 mmol

2. Sodium 1 to 2 mEq/kg 3. Magnesium 8 to 20 mEq 4. Potassium 1 to 2 mEq/kg 5. Phosphate 20 to 40 mmol Individual requirements for TPN should be assessed daily at the beginning of therapy and then several times a week as the treatment progresses. The following are ranges for average daily electrolyte requirements for adult clients without renal or hepatic impairment: sodium 1 to 2 mEq/kg; potassium 1 to 2 mEq/kg; chloride as needed to maintain acid-base balance; magnesium 8 to 20 mEq; calcium 10 to 15 mEq; and phosphate 20 to 40 mmol.

The nurse is teaching adult cardiopulmonary resuscitation (CPR) guidelines to a group of laypeople. The nurse observes the group correctly demonstrate 2-rescuer CPR when which ratio of compressions to ventilations is performed on the mannequin? 1. 10:1 2. 15:2 3. 20:1 4. 30:2

30:2 When performing CPR on adults, the ratio of chest compressions to breaths should be 30:2 for both 1-rescuer and 2-rescuer CPR. The ratio of 15:2 is used for children and infants during 2-rescuer CPR.

The police arrive at the emergency department with a client who has lacerated both wrists. Which is the initial nursing action 1. Administer an antianxiety agent. 2. Assess and treat the wound sites. 3. Secure and record a detailed history. 4. Encourage and assist the client to ventilate feelings.

Assess and treat the wound sites. The initial nursing action is to assess and treat the self-inflicted injuries. Injuries from lacerated wrists can lead to a life-threatening situation. Other interventions, such as options 1, 3, and 4, may follow after the client has been treated medically.

During the early postoperative period, a client who has undergone a cataract extraction complains of nausea and severe eye pain over the operative site. What should be the initial nursing action? 1. Call the health care provider (HCP). 2. Reassure the client that this is normal. 3. Turn the client onto his or her operative side. 4. Administer the prescribed pain medication and antiemetic.

Call the health care provider (HCP). Severe pain or pain accompanied by nausea following a cataract extraction is an indicator of increased intraocular pressure and should be reported to the HCP immediately. Options 2, 3, and 4 are inappropriate actions.

A client diagnosed with thrombophlebitis 1 day ago suddenly complains of chest pain and shortness of breath and is visibly anxious. Which complication should the nurse immediately assess the client for? 1. Pneumonia 2. Pulmonary edema 3. Pulmonary embolism 4. Myocardial infarction

Pulmonary embolism Pulmonary embolism is a life-threatening complication of deep vein thrombosis and thrombophlebitis. Chest pain is the most common symptom; it is sudden in onset and may be aggravated by breathing. Other signs and symptoms include dyspnea, cough, diaphoresis, and apprehension. The conditions in the remaining options are not associated with thrombophlebitis.

A client arrives in the emergency department with a penetrating eye injury from wood chips that occurred while cutting wood. The nurse assesses the eye and notes a piece of wood protruding from the eye. What is the initial nursing action? 1. Apply an eye patch. 2. Perform visual acuity tests. 3. Irrigate the eye with sterile saline. 4. Remove the piece of wood using a sterile eye clamp.

Perform visual acuity tests. If the eye injury is the result of a penetrating object, the object may be noted protruding from the eye. This object must never be removed except by the ophthalmologist because it may be holding ocular structures in place. Application of an eye patch or irrigation of the eye may disrupt the foreign body and cause further tearing of the cornea.

A client in shock is receiving dopamine hydrochloride by intravenous (IV) infusion. The nurse should have which medication available for local injection if IV infiltration and medication extravasation occur? 1. Vitamin K 2. Phentolamine 3. Atropine sulfate 4. Protamine sulfate

Phentolamine Phentolamine is an alpha-adrenergic blocking agent that prevents dermal necrosis and sloughing after infiltration of norepinephrine or dopamine. Vitamin K is the antidote for warfarin. Atropine sulfate is the antidote for cholinergic crisis. Protamine sulfate is the antidote for heparin

The nurse has received a prescription to transfuse a client with a unit of packed red blood cells. Before explaining the procedure to the client, the nurse should ask which initial question? 1. "Have you ever had a transfusion before?" 2. "Why do you think that you need the transfusion?" 3. "Have you ever gone into shock for any reason in the past?" 4. "Do you know the complications and risks of a transfusion?"

"Have you ever had a transfusion before?" Asking the client about personal experience with transfusion therapy provides a good starting point for client teaching about this procedure. Questioning about previous history of shock and knowledge of complications and risks of transfusion is not helpful because it may elicit a fearful response from the client. Although determining whether the client knows the reason for the transfusion is important, it is not an appropriate statement in terms of eliciting information from the client regarding an understanding of the need for the transfusion.

The nurse provides a list of instructions to a client being discharged to home with a peripherally inserted central catheter (PICC). The nurse determines that the client needs further instructions if the client made which statement? 1. "I need to wear a MedicAlert tag or bracelet." 2. "I need to restrict my activity while this catheter is in place." 3. "I need to keep the insertion site protected when in the shower or bath." 4. "I need to check the markings on the catheter each time the dressing is changed."

"I need to restrict my activity while this catheter is in place." The client should be taught that only minor activity restrictions apply with this type of catheter. The client should carry or wear a MedicAlert identification and should protect the site during bathing to prevent infection. The client should check the markings on the catheter during each dressing change to assess for catheter migration or dislodgement

The nurse provides a list of instructions to a client being discharged to home with a peripherally inserted central catheter (PICC). The nurse determines that the client needs further instruction if the client makes which statement? 1. "I need to wear a MedicAlert tag or bracelet." 2. "I need to restrict my activity while this catheter is in place." 3. "I need to have a repair kit available in the home for use if needed." 4. "I need to keep the insertion site protected when in the shower or bath

"I need to restrict my activity while this catheter is in place." The client should be taught that only minor activity restrictions apply with this type of catheter. The client should protect the site during bathing and should carry or wear a MedicAlert identification. The client should have a repair kit in the home for use as needed because the catheter is for long-term use.

The nursing educator has just completed a lecture to a group of nurses regarding care of the client with a burn injury. A major aspect of the lecture was care of the client at the scene of a fire. Which statement, if made by a nurse, indicates a need for further instruction? 1. "Flames should be doused with water." 2. "The client should be maintained in a standing position." 3. "Flames may be extinguished by rolling the client on the ground." 4. "Flames may be smothered by the use of a blanket or another cover."

"The client should be maintained in a standing position." The client should be placed or maintained in a supine position; otherwise, flames may spread to other parts of the body, causing more extensive injury. Flame burns may be extinguished by rolling the client on the ground, smothering the flames with a blanket or other cover, or dousing the flames with water.

The nurse is picking up a unit of packed red blood cells at the hospital blood bank. After putting the pen down, the nurse glances at the clock, which reads 1300. The nurse calculates that the transfusion must be started by which time? 1. 1330 2. 1400 3. 1430 4. 1500

1330 Blood must be hung as soon as possible (within 30 minutes) after it is obtained from the blood bank. After that time, the blood temperature will be higher than 50°F (10°C), and the blood could be unsafe for use. For this reason, the remaining options are incorrect.

The nurse, caring for a group of adult clients on an acute care medical-surgical nursing unit, determines that which clients would be the most likely candidates for parenteral nutrition (PN)? Select all that apply. 1. A client with extensive burns 2. A client with cancer who is septic 3. A client who has had an open cholecystectomy 4. A client with severe exacerbation of Crohn's disease 5. A client with persistent nausea and vomiting from chemotherapy

1. A client with extensive burns 2. A client with cancer who is septic 4. A client with severe exacerbation of Crohn's disease 5. A client with persistent nausea and vomiting from chemotherapy PN is indicated in clients whose gastrointestinal tracts are not functional or must be rested, cannot take in a diet enterally for extended periods, or have increased metabolic need. Examples of these conditions include those clients with burns, exacerbation of Crohn's disease, and persistent nausea and vomiting due to chemotherapy. Other clients would be those who have had extensive surgery, have multiple fractures, are septic, or have advanced cancer or acquired immunodeficiency syndrome. The client with the open cholecystectomy is not a candidate because this client would resume a regular diet within a few days following surgery.

The nurse in a medical unit is caring for a client with heart failure. The client suddenly develops extreme dyspnea, tachycardia, and lung crackles and the nurse suspects pulmonary edema. The nurse immediately asks another nurse to contact the health care provider and prepares to implement which priority interventions? Select all that apply. 1. Administering oxygen 2. Inserting a Foley catheter 3. Administering furosemide 4. Administering morphine sulfate intravenously 5. Transporting the client to the coronary care unit 6. Placing the client in a low Fowler's side-lying position

1. Administering oxygen 2. Inserting a Foley catheter 3. Administering furosemide 4. Administering morphine sulfate intravenously Pulmonary edema is a life-threatening event that can result from severe heart failure. In pulmonary edema, the left ventricle fails to eject sufficient blood, and pressure increases in the lungs because of the accumulated blood. Oxygen is always prescribed, and the client is placed in a high Fowler's position to ease the work of breathing. Furosemide, a rapid-acting diuretic, will eliminate accumulated fluid. A Foley catheter is inserted to measure output accurately. Intravenously administered morphine sulfate reduces venous return (preload), decreases anxiety, and also reduces the work of breathing. Transporting the client to the coronary care unit is not a priority intervention. In fact, this may not be necessary at all if the client's response to treatment is successful.

The client who has had radical neck dissection begins to hemorrhage at the incision site. Which immediate actions should the nurse take? Select all that apply. 1. Monitor the client's airway. 2. Call the Rapid Response Team. 3. Call the health care provider (HCP). 4. Apply manual pressure over the site. 5. Lower the head of the bed to a flat position.

1. Monitor the client's airway. 2. Call the Rapid Response Team. 3. Call the health care provider (HCP). 4. Apply manual pressure over the site If the client begins to hemorrhage from the surgical site following radical neck dissection, the nurse elevates the head of the bed to maintain airway patency and prevent aspiration. The nurse applies pressure over the bleeding site and stays with the client and ask someone to call the Rapid Response Team and the HCP immediately.

Which interventions would be included in the care of a client with a head injury and a subarachnoid bolt? Select all that apply. 1. Monitor vital signs. 2. Monitor neurological status. 3. Monitor the dressing for signs of infection. 4. Monitor for signs of increased intracranial pressure. 5. Drain cerebrospinal fluid when the intracranial pressure is elevated.

1. Monitor vital signs. 2. Monitor neurological status. 3. Monitor the dressing for signs of infection. 4. Monitor for signs of increased intracranial pressure. A subarachnoid bolt is inserted into the subarachnoid space and is used to measure intracranial pressure. Because a subarachnoid bolt is placed in the subarachnoid space, it is not capable of draining cerebrospinal fluid, which is produced in the ventricles. Therefore, the option to drain cerebrospinal fluid is not an intervention. The remaining options are appropriate interventions.

A client has had radical neck dissection and begins to hemorrhage at the incision site. The nurse should take which actions in this situation? Select all that apply. 1. Monitor vital signs. 2. Monitor the client's airway. 3. Apply manual pressure over the site. 4. Lower the head of the bed to a flat position. 5. Call the health care provider (HCP) immediately.

1. Monitor vital signs. 2. Monitor the client's airway. 3. Apply manual pressure over the site. 5. Call the health care provider (HCP) immediately. If the client begins to hemorrhage from the surgical site after radical neck dissection, the nurse elevates the head of the bed to maintain airway patency and prevent aspiration. The nurse applies pressure over the bleeding site and calls the HCP immediately. The nurse also monitors the client's airway and vital signs.

A client rings the call light and complains of pain at the site of an intravenous (IV) infusion. The nurse assesses the site and determines that phlebitis has developed. The nurse should take which actions in the care of this client? Select all that apply. 1. Remove the IV catheter at that site. 2. Apply warm moist packs to the site. 3. Notify the health care provider (HCP). 4. Start a new IV line in a proximal portion of the same vein. 5. Document the occurrence, actions taken, and the client's response.

1. Remove the IV catheter at that site. 2. Apply warm moist packs to the site. 3. Notify the health care provider (HCP). 5. Document the occurrence, actions taken, and the client's response. Phlebitis is an inflammation of the vein that can occur from mechanical or chemical (medication) trauma or from a local infection and can cause the development of a clot (thrombophlebitis). The nurse should remove the IV at the phlebitic site and apply warm moist compresses to the area to speed resolution of the inflammation. Because phlebitis has occurred, the nurse also notifies the HCP about the IV complication. The nurse should restart the IV in a vein other than the one that has developed phlebitis. Finally, the nurse documents the occurrence, actions taken, and the client's response.

When creating a mechanically ventilated client's plan of care for prevention of ventilator-associated pneumonia (VAP), the nurse should include which measures in the plan? Select all that apply. 1. Suction the oral cavity whenever needed. 2. Apply topical antibiotics to the oral cavity. 3. Change the ventilator circuit tubing every 2 hours. 4. Maintain the client in a supine position at all times. 5. Practice frequent oral hygiene, including teeth brushing. 6. Wear gloves when suctioning or handling the endotracheal tube.

1. Suction the oral cavity whenever needed. 5. Practice frequent oral hygiene, including teeth brushing. 6. Wear gloves when suctioning or handling the endotracheal tube. Measures to prevent VAP include removing subglottic secretions every 2 hours and whenever needed; frequent oral hygiene, which includes teeth brushing; hand hygiene before and after each client contact; and application of gloves after hand washing and before suctioning. Topical antibiotics have no effect. Ventilator circuit tubing does not need to be changed every 2 hours; depending on agency policy, tubing is changed every 24 or 48 hours. Head of the bed elevation is maintained at a minimum of 30 degrees; a supine position can lead to aspiration.

The home care nurse is monitoring a client's response to total parenteral nutrition (TPN). The client's weight 1 week earlier was 114 lbs (52 kg). The nurse determines that the client is gaining weight as expected if which morning weight is noted? 1. 116 lbs (52.6 kg) 2. 119 lbs (53.9 kg) 3. 120 lbs (54.4 kg) 4. 122 lbs (55.3 kg)

116 lbs (52.6 kg) The client receiving TPN should not gain more than 3 lbs (1.4 kg) per week, with optimal weight gain being 1 to 2 lbs (0.5 to 1 kg) per week. The weight goal for the client on TPN is individual and depends on the client's metabolic needs and baseline weight (whether underweight, overweight, or at optimal weight). The correct option identifies a reasonable weight gain of 2 lbs (1 kg) per week. The remaining options indicate weekly weight gains that are greater than expected.

A pulmonary artery catheter is inserted into a client during cardiac surgery. The nurse is monitoring the right atrial pressure (RAP). Which finding requires immediate nursing intervention? 1. 4 mm Hg 2. 6 mm Hg 3. 8 mm Hg 4. 12 mm Hg

12 mm Hg The normal RAP is 1 to 8 mm Hg; therefore, all options within this range (1, 2, and 3) are normal findings. An elevated RAP can be indicative of right ventricular failure.

A client is brought to the emergency department immediately after a smoke inhalation injury. The nurse initially prepares the client for which treatment? 1. Pain medication 2. Endotracheal intubation 3. Oxygen via nasal cannula 4. 100% humidified oxygen by face mask

100% humidified oxygen by face mask With a smoke inhalation injury, the client is immediately treated with 100% humidified oxygen delivered by face mask. This method provides a greater concentration of oxygen than oxygen delivered via nasal cannula. Endotracheal intubation is needed if the client exhibits respiratory stridor, which indicates airway obstruction. Pain medication may be needed but would not be the initial intervention.

A client is brought to the emergency department with partial-thickness burns to his face, neck, arms, and chest after trying to put out a car fire. The nurse should implement which nursing actions for this client? Select all that apply. 1. Restrict fluids. 2. Assess for airway patency. 3. Administer oxygen as prescribed. 4. Place a cooling blanket on the client. 5. Elevate extremities if no fractures are present. 6. Prepare to give oral pain medication as prescribed.

2. Assess for airway patency. 3. Administer oxygen as prescribed. 5. Elevate extremities if no fractures are present. The primary goal for a burn injury is to maintain a patent airway, administer intravenous (IV) fluids to prevent hypovolemic shock, and preserve vital organ functioning. Therefore, the priority actions are to assess for airway patency and maintain a patent airway. The nurse then prepares to administer oxygen. Oxygen is necessary to perfuse vital tissues and organs. An IV line should be obtained and fluid resuscitation started. The extremities are elevated to assist in preventing shock and decrease fluid moving to the extremities, especially in the burn-injured upper extremities. The client is kept warm since the loss of skin integrity causes heat loss. The client is placed on NPO (nothing by mouth) status because of the altered gastrointestinal function that occurs as a result of a burn injury.

The nurse is assessing a client hospitalized with acute pericarditis. The nurse monitors the client for cardiac tamponade, knowing that which signs are associated with this complication of pericarditis? Select all that apply. 1. Bradycardia 2. Pulsus paradoxus 3. Distant heart sounds 4. Falling blood pressure (BP) 5. Distended jugular veins

2. Pulsus paradoxus 3. Distant heart sounds 4. Falling blood pressure (BP) 5. Distended jugular veins Assessment findings with cardiac tamponade include tachycardia, distant or muffled heart sounds, jugular vein distention, and a falling BP, accompanied by pulsus paradoxus (a drop in inspiratory BP by more than 10 mm Hg).

Which clinical manifestations of a tension pneumothorax should be of immediate concern to the nurse? Select all that apply 1. Bradypnea 2. Flattened neck veins 3. Decreased cardiac output 4. Hyperresonance to percussion 5. Tracheal deviation to the opposite side

3. Decreased cardiac output 4. Hyperresonance to percussion 5. Tracheal deviation to the opposite side Tension pneumothorax is the rapid accumulation of air in the pleural space. This causes extremely high intrapleural pressures, resulting in tension on the heart and great vessels. This can cause decreased cardiac output (tachycardia, hypotension), hyperresonance on percussion, and a tracheal shift away from the affected side. Bradypnea and flattened neck veins are incorrect because the client would have tachypnea and distended neck veins.

The nurse is caring for a client who is pulseless and experiencing this dysrhythmia. Which interventions should the nurse anticipate implementing in collaboration with the health care provider (HCP)? Select all that apply. Refer to Figure. 1. Prepare for cardioversion. 2. Prepare to administer digoxin 3. Prepare to administer amiodarone. 4. Prepare to administer epinephrine. 5. Provide cardiopulmonary resuscitation (CPR).

3. Prepare to administer amiodarone. 4. Prepare to administer epinephrine. 5. Provide cardiopulmonary resuscitation (CPR). Pulseless ventricular tachycardia is treated the same way as ventricular fibrillation with measures that include defibrillation, CPR and medication therapy, with agents such as epinephrine and amiodarone and others.

The nurse is obtaining blood from a client's double-lumen central venous catheter for blood cultures. Which actions are correct for performing this procedure? Select all that apply. 1. Use the distal port of the catheter for obtaining the blood specimen. 2. Flush with 5 to 10 mL of normal saline before obtaining the specimen. 3. Turn the infusion off for at least 1 minute before obtaining the specimen. 4. Use the initial specimen of blood obtained from the catheter for the blood cultures. 5. Discard the first syringe of blood and use the second syringe for the blood cultures.

3. Turn the infusion off for at least 1 minute before obtaining the specimen. 4. Use the initial specimen of blood obtained from the catheter for the blood cultures When drawing blood from a double-lumen central venous catheter, the proximal port is used because it is usually the port with the largest lumen. For blood cultures, the initial specimen is used for the sample, and the line is not flushed beforehand. Turning off the infusion for 1 minute prevents contaminating the sample with intravenous solution.

A client involved in a motor vehicle crash presents to the emergency department with severe internal bleeding. The client is severely hypotensive and unresponsive. The nurse anticipates that which intravenous (IV) solution will most likely be prescribed? 1. 5% dextrose in lactated Ringer's 2. 0.33% sodium chloride (⅓ normal saline) 3. 0.225% sodium chloride (¼ normal saline) 4. 0.45% sodium chloride (½ normal saline)

5% dextrose in lactated Ringer's The goal of therapy with this client is to expand intravascular volume as quickly as possible. The 5% dextrose in lactated Ringer's solution (hypertonic solution) would increase intravascular volume and immediately replace lost fluid volume until a transfusion could be administered, resulting in an increase in the client's blood pressure. The solutions in the remaining options would not be given to this client because they are hypotonic solutions and, instead of increasing intravascular space, would move into the cells via osmosis.

The nurse is preparing to assist in administering neonatal resuscitation with a ventilation bag and mask because the newborn is apneic, gasping, and has a heart rate below 100 beats/min. The nurse should perform how many ventilations per minute at which pressure? 1. 20 to 40 breaths/min, 15 to 20 cm H2O pressure 2. 20 to 40 breaths/min, 30 to 40 cm H2O pressure 3. 40 to 60 breaths/min, 15 to 20 cm H2O pressure 4. 40 to 60 breaths/min, 30 to 40 cm H2O pressure

40 to 60 breaths/min, 15 to 20 cm H2O pressure If the newborn is apneic or has gasping respirations after stimulation or if the heart rate is below 100 beats/min, positive pressure ventilation by bag and mask can be given. The anesthesia bag used for neonatal resuscitation should have a pressure gauge. Ventilations should be given at a rate of 40 to 60 breaths/min at pressures of 15 to 20 cm H2O.

The nurse has a new prescription to administer verapamil by the intravenous (IV) route. In administering this medication, the most important nursing action should be to use what item to monitor the client's response to the medication? 1. A pulse oximeter 2. A cardiac monitor 3. Supplemental oxygen 4. A noninvasive blood pressure monitor

A cardiac monitor Verapamil is a calcium channel blocker that may be used to treat rapid-rate supraventricular tachydysrhythmias such as atrial flutter or atrial fibrillation. A cardiac monitor is used to determine the client's response to the medication. A pulse oximeter and oxygen are related to respiratory care and may be other useful adjuncts to care, but they are not directly related to the use of this medication. A noninvasive blood pressure monitor also is helpful but is not as essential or critical as the cardiac monitor.

A client arrives in the emergency department following an automobile crash. The client's forehead hit the steering wheel and a hyphema is diagnosed. The nurse should place the client in which position? 1. Flat in bed 2. A semi Fowler's position 3. Lateral on the affected side 4. Lateral on the unaffected side

A semi Fowler's position A hyphema is the presence of blood in the anterior chamber. Hyphema is produced when a force is sufficient to break the integrity of the blood vessels in the eye and can be caused by direct injury, such as a penetrating injury from a BB or pellet, or indirectly, such as from striking the forehead on a steering wheel during an accident. The client is treated by bed rest in a semi Fowler's position to assist gravity in keeping the hyphema away from the optical center of the cornea.

The nurse is teaching cardiopulmonary resuscitation (CPR) to a group of community members. The nurse tells the group that when chest compressions are performed on infants, the sternum should be depressed how far? 1. At least 2 inches (5 cm) 2. About 1½ inches (4 cm) 3. At least one half the depth of the chest 4. Deep enough to make a finger impression

About 1½ inches (4 cm) According to the American Heart Association's 2015 guidelines, when CPR is performed on infants, the sternum should be depressed at least one third the depth of chest, which is about 1½ inches or 4 cm. The remaining options are incorrect.

The nurse is assisting in the care of a client who is being seen in the clinic with a suspected acetaminophen overdose. What is the nurse's priority of care? 1. Administer acetylcysteine. 2. Obtain a 12-lead electrocardiogram. 3. Ask the client about other medication use. 4. Ask the client why so many acetaminophen were taken.

Administer acetylcysteine. The antidote for acetaminophen is acetylcysteine, which works by preventing the hepatotoxic metabolites of acetaminophen from forming, so early administration is essential. Although the other options may be part of the client's assessment, they do not need to be carried out immediately.

A postpartum client with femoral thrombophlebitis has developed sudden shortness of breath and appears very anxious. What is the nurse's priority action for this client? 1. Check the client's blood pressure immediately. 2. Elevate the head of the bed to 30 to 45 degrees. 3. Initiate an intravenous line if one is not already in place. 4. Administer oxygen by face mask as per protocol at 8 to 10 L/min.

Administer oxygen by face mask as per protocol at 8 to 10 L/min. This client is at increased risk for pulmonary embolus and is exhibiting symptoms. Because pulmonary circulation is compromised in the presence of an embolus, cardiorespiratory support is initiated by oxygen administration. Although the remaining options may be implemented, none of these is the priority nursing action.

The nurse in the hospital emergency department is caring for a client with suspected opioid overdose and is preparing to administer the reversal agent via the intravenous route. Which statement is correct about the administration of this medication? 1. Prepare only 1 dose of the reversal agent. 2. Administer the entire dose by slow intravenous push. 3. Administer the medication rapidly by the intravenous route. 4. After the initial dose, prepare to administer additional intravenous doses if needed.

After the initial dose, prepare to administer additional intravenous doses if needed. The reversal agent for opioids is naloxone hydrochloride, which should be titrated every 2 to 5 minutes according to client response. Therefore, the remaining options are incorrect.

A client receiving total parenteral nutrition (TPN) experiences sudden development of chest pain, dyspnea, tachycardia, cyanosis, and a decreased level of consciousness. What should the nurse suspect as a complication of the TPN? 1. Air embolism 2. Hyperglycemia 3. Catheter-related sepsis 4. Allergic reaction to the catheter

Air embolism Signs and symptoms of air embolism include decreased level of consciousness, tachycardia, dyspnea, anxiety, feelings of impending doom, chest pain, cyanosis, and hypotension. The signs and symptoms in the question do not indicate hyperglycemia, an infection (catheter-related sepsis), or an allergic reaction.

A unit of platelets was just received from the blood bank for transfusion to an assigned client. The nurse should select tubing with which feature for the transfusion? 1. An in-line filter 2. At least 3 Y-ports 3. Self-sealing valves 4. Tinted to protect the blood from light

An in-line filter The tubing used for platelet administration has an in-line filter. This helps ensure that any particles larger than the size of the filter are caught in the filter and are not infused into the client. Self-sealing valves and Y-ports are unnecessary. These features may be used to administer medication. No medication is infused through the intravenous (IV) line that the blood is infusing through. If the client needed medications as a result of a complication while receiving blood or for another reason, it would need to be administered via a different IV site and line. Platelets do not need to be protected from light.

When performing a surgical dressing change of a client's abdominal dressing, the nurse notes an increase in the amount of drainage and separation of the incision line. The underlying tissue is visible to the nurse. What should the nurse do next? 1. Apply a sterile dressing soaked with povidone-iodine. 2. Apply a sterile dressing soaked with normal saline. 3. Irrigate the wound, and apply a dry sterile dressing. 4. Leave the incision exposed to the air to dry the area.

Apply a sterile dressing soaked with normal saline. Wound dehiscence is the separation of wound edges at the suture line. Signs and symptoms include increased drainage and the exposure of underlying tissues. These usually occur 6 to 8 days after surgery. The client should be instructed to remain quiet and avoid coughing or straining. The client should be positioned to prevent further stress on the wound. Sterile dressings soaked with sterile normal saline should be used to cover the wound. The health care provider must be notified after applying this initial dressing to the wound.

The nurse is caring for a client with a chest tube drainage system. During repositioning of the client, the chest tube accidentally pulls out of the pleural cavity. Which is the initial nursing action? 1. Apply an occlusive dressing. 2. Reinsert the chest tube quickly. 3. Contact the respiratory therapist. 4. Contact the health care provider (HCP).

Apply an occlusive dressing. If a chest tube is accidentally pulled out, the nurse would immediately apply an occlusive dressing and then contact the HCP. It is not appropriate and not a nursing role to reinsert a chest tube. It is not necessary to contact the respiratory therapist. The HCP needs to be notified, but this is not the initial nursing action.

The nurse should evaluate that defibrillation of a client was most successful if which observation was made? 1. Arousable, sinus rhythm, blood pressure (BP) 116/72 mm Hg 2. Nonarousable, sinus rhythm, BP 88/60 mm Hg 3. Arousable, marked bradycardia, BP 86/54 mm Hg 4. Nonarousable, supraventricular tachycardia, BP 122/60 mm Hg

Arousable, sinus rhythm, blood pressure (BP) 116/72 mm Hg After defibrillation, the client requires continuous monitoring of electrocardiographic rhythm, hemodynamic status, and neurological status. Respiratory and metabolic acidosis develop during ventricular fibrillation because of lack of respiration and cardiac output. These can cause cerebral and cardiopulmonary complications. Arousable status, adequate BP, and a sinus rhythm indicate successful response to defibrillation.

The nurse notes that a client's total parenteral nutrition (TPN) solution is 4 hours behind. Which action should the nurse take? 1. Assess the infusion pump to be sure it is functioning properly and is set at the correct rate. 2. Increase the infusion rate to a rate that allows the infusion volume to correct itself within a 2-hour period. 3. Replace the TPN solution with 10% dextrose, and restart the solution the following day. 4. Administer the TPN solution using gravity flow because the infusion pump is malfunctioning.

Assess the infusion pump to be sure it is functioning properly and is set at the correct rate. If an infusion falls behind schedule, the nurse should not increase the rate in an attempt to catch up, because a hyperosmotic reaction, among other reactions, could result. The solution should not be replaced by another or restarted the next day. An infusion pump should always be used to administer TPN solution.

n order of priority, how should the nurse perform abdominal thrusts on an unconscious adult? Arrange the actions in the order that they should be performed. All options must be used.

Assess unconsciousness. Open the airway. Look in the mouth and remove the object blocking the airway, if seen. Attempt ventilation. Perform abdominal thrusts. For health care providers (HCPs) such as the nurse, the sequence for removing a foreign body airway obstruction in an adult is as follows. After determining unconsciousness, the airway is opened and the rescuer looks into the mouth of the victim and removes the object blocking the airway, if it is seen. Next, the HCP attempts to ventilate the victim. If unsuccessful, the victim's head is repositioned and ventilation is reattempted. Five abdominal thrusts are then delivered. The sequence is repeated until successful.

The nurse is watching the cardiac monitor, and a client's rhythm suddenly changes. There are no P waves; instead, there are fibrillatory waves before each QRS complex. How should the nurse correctly interpret the client's heart rhythm? 1. Atrial fibrillation 2. Sinus tachycardia 3. Ventricular fibrillation 4. Ventricular tachycardia

Atrial fibrillation Atrial fibrillation is characterized by a loss of P waves and fibrillatory waves before each QRS complex. The atria quiver, which can lead to thrombus formation.

The family of a client with a spinal cord injury rushes to the nursing station, saying that the client needs immediate help. On entering the room, the nurse notes that the client is diaphoretic with a flushed face and neck and is complaining of a severe headache. The pulse rate is 40 beats/minute, and the blood pressure is 230/100 mm Hg. The nurse acts quickly, suspecting that the client is experiencing which condition? 1. Spinal shock 2. Pulmonary embolism 3. Autonomic dysreflexia 4. Malignant hyperthermia

Autonomic dysreflexia The client with a spinal cord injury is at risk for autonomic dysreflexia with an injury above the level of the seventh thoracic vertebra (T7). Autonomic dysreflexia is characterized by severe, throbbing headache; flushing of the face and neck; bradycardia; and sudden severe hypertension. Other signs include nasal stuffiness, blurred vision, nausea, and sweating. Autonomic dysreflexia is a life-threatening syndrome triggered by a noxious stimulus below the level of the injury. The data in the question are not associated with the conditions noted in the remaining options.

The nurse is performing an assessment on a client admitted to the nursing unit who has sustained an extensive burn injury involving 45% of total body surface area. When planning for fluid resuscitation, the nurse should consider that fluid shifting to the interstitial spaces is greatest during which time period? 1. Immediately after the injury 2. Within 12 hours after the injury 3. Between 18 and 24 hours after the injury 4. Between 42 and 72 hours after the injury

Between 18 and 24 hours after the injury The maximum amount of edema in a client with a burn injury is seen between 18 and 24 hours after the injury. With adequate fluid resuscitation the transmembrane potential is restored to normal within 24 to 36 hours after the burn. The remaining options are incorrect

The nurse has discontinued a unit of blood that was infusing into a client because the client experienced a transfusion reaction. After documenting the incident appropriately, the nurse sends the blood bag and tubing to which department? 1. Blood bank 2. Infection control 3. Risk management 4. Environmental services

Blood bank The nurse returns the blood transfusion bag containing any remaining blood to the blood bank. This allows the blood bank to complete any follow-up testing procedures needed once a transfusion reaction has been documented. The other options identify incorrect departments.

The nurse is creating a plan of care for a client who is receiving total parenteral nutrition (TPN). Which assessment should be included in the plan of care? 1. Apical rate every hour 2. Continuous pulse oximetry 3. Blood glucose levels every 6 hours 4. Hemoglobin and hematocrit every 8 hour

Blood glucose levels every 6 hours Complications associated with TPN therapy include hypoglycemia or hyperglycemia, infection, fluid overload, air embolism, and electrolyte imbalance. It is standard care to monitor blood glucose levels at 6-hour intervals to assess for hyperglycemia. The remaining options contain assessments that are not routine for a client receiving TPN.

The nurse is inserting an intravenous (IV) line into a client's vein. After the initial stick, the nurse would continue to advance the catheter in which situation? 1. The catheter advances easily. 2. The vein is distended under the needle. 3. The client does not complain of discomfort. 4. Blood return shows in the backflash chamber of the catheter.

Blood return shows in the backflash chamber of the catheter. The IV catheter has entered the lumen of the vein successfully when blood backflash shows in the IV catheter. The vein should have been distended by the tourniquet before the vein was cannulated, and if further distention occurs after venipuncture, this could mean the needle went through the vein and into the tissue; therefore, the catheter should not be advanced. Client discomfort varies with the client, the site, and the nurse's insertion technique and is not a reliable measure of catheter placement. The nurse should not advance the catheter until placement in the vein is verified by blood return.

The nursing instructor teaches a group of students about cardiopulmonary resuscitation. The instructor asks a student to identify the most appropriate location at which to assess the pulse of an infant younger than 1 year of age. Which response would indicate that the student understands the appropriate assessment procedure? 1. Radial artery 2. Carotid artery 3. Brachial artery 4. Popliteal artery

Brachial artery To assess a pulse in an infant (younger than 1 year), the pulse is checked at the brachial or femoral artery. The infant's relatively short, fat neck makes palpation of the carotid artery difficult. The popliteal and radial pulses are also difficult to palpate in an infant.

A client in cardiogenic shock has a pulmonary artery catheter (Swan-Ganz type) placed. The nurse would interpret which cardiac output (CO) and pulmonary capillary wedge pressure (PCWP) readings as indicating that the client is most unstable? 1. CO 5 L/min, PCWP low 2. CO 3 L/min, PCWP low 3. CO 4 L/min, PCWP high 4. CO 3 L/min, PCWP high

CO 3 L/min, PCWP high The normal cardiac output is 4 to 7 L/min. With cardiogenic shock, the CO falls below normal because of failure of the heart as a pump. The PCWP, however, rises because it is a reflection of the left ventricular end-diastolic pressure, which rises with pump failure.

The nurse has administered a dose of salmeterol to a client. The client develops a generalized rash and urticaria, and the eyelids begin to swell. Which action should the nurse take? 1. Apply a lanolin-based cream to the rash. 2. Encourage the client to drink fluids quickly. 3. Assess the client's vision with a Snellen chart. 4. Call the health care provider (HCP) immediately.

Call the health care provider (HCP) immediately. Hypersensitivity reaction can occur in clients taking salmeterol. Signs include rash; urticaria; and swelling of the face, lips, or eyelids. The nurse should call the HCP immediately if any of these occur. The other options are incorrect.

A client in cardiogenic shock had an intra-aortic balloon pump inserted 24 hours earlier via the left femoral approach. The nurse notes that the client's left foot is cool and mottled and the left pedal pulse is weak. Which action should the nurse take? 1. Call the health care provider immediately. 2. Document these findings, which are expected. 3. Re-evaluate the neurovascular status in 1 hour. 4. Increase the rate of the intravenous nitroglycerin infusion.

Call the health care provider immediately. The nursing interventions for the client with an intra-aortic balloon pump are the same as for any client who has had cardiovascular surgery. The peripheral circulation to the affected limb is monitored for signs of occlusion, such as coolness, mottling, pain, tingling, and decreased or absent distal pulse. Adverse changes are reported immediately. The remaining options are incorrect

The nurse is providing morning care to a client who has a closed chest tube drainage system to treat a pneumothorax. When the nurse turns the client to the side, the chest tube is accidentally dislodged from the chest. The nurse immediately applies sterile gauze over the chest tube insertion site. Which is the nurse's next action? 1. Call the health care provider. 2. Replace the chest tube system. 3. Obtain a pulse oximetry reading. 4. Place the client in a Trendelenburg's position.

Call the health care provider. If the chest drainage system is dislodged from the insertion site, the nurse immediately applies sterile gauze over the site and calls the health care provider. The nurse would maintain the client in an upright position. A new chest tube system may be attached if the tube requires insertion, but this would not be the next action. Pulse oximetry readings would assist in determining the client's respiratory status, but the priority action would be to call the health care provider in this emergency situation.

The nurse is reviewing the medical record of a client transferred to the medical unit from the critical care unit. The nurse notes that the client received intra-aortic balloon pump (IABP) therapy while in the critical care unit. The nurse suspects that the client received this therapy for which condition? 1. Heart failure 2. Pulmonary edema 3. Cardiogenic shock 4. Aortic insufficiency

Cardiogenic shock IABP therapy most often is used in the treatment of cardiogenic shock and is most effective if instituted early in the course of treatment. Use of IABP therapy is contraindicated in clients with aortic insufficiency and thoracic and abdominal aneurysms. This therapy is not used in the treatment of congestive heart failure or pulmonary edema.

The nurse has completed 5 cycles of compressions after beginning cardiopulmonary resuscitation (CPR) on a hospitalized adult client who experienced unmonitored cardiac arrest. What should the nurse plan to do next? 1. Prepare epinephrine. 2. Charge the defibrillator. 3. Check the client's heart rhythm. 4. Pause CPR for 20 seconds and reassess.

Charge the defibrillator. For witnessed adult cardiac arrest when a defibrillator is immediately available, it is reasonable that the defibrillator be used as soon as possible. For adults with unmonitored cardiac arrest or for whom a defibrillator is not immediately available, it is reasonable that CPR be initiated while the defibrillator equipment is being retrieved and applied and that defibrillation, if indicated, be attempted as soon as the device is ready for use. After completing 5 cycles of compressions and ventilations, the nurse should reassess the client by checking the heart rhythm. Defibrillation may be warranted depending on the assessed rhythm. Epinephrine may be prepared depending on the rhythm, but this would be prescribed by a health care provider (HCP). Chest compressions should not be interrupted for more than 10 seconds.

A client has just undergone insertion of a central venous catheter at the bedside under ultrasound. The nurse would be sure to check which results before initiating the flow rate of the client's intravenous (IV) solution at 100 mL/hour? 1. Serum osmolality 2. Serum electrolyte levels 3. Intake and output record 4. Chest radiology results

Chest radiology results Before beginning administration of IV solution, the nurse should assess whether the chest radiology results reveal that the central catheter is in the proper place. This is necessary to prevent infusion of IV fluid into pulmonary or subcutaneous tissues. The other options represent items that are useful for the nurse to be aware of in the general care of this client, but they do not relate to this procedure.

The nurse is caring for a client who is receiving a blood transfusion and is complaining of a cough. The nurse checks the client's vital signs, which include temperature of 97.2°F (36.2°C), pulse of 108 beats per minute, blood pressure of 152/76 mm Hg, respiratory rate of 24 breaths per minute, and an oxygen saturation level of 95% on room air. The client denies pain at this time. Based on this information, what initial action should the nurse take? 1. Collect a urine sample for analysis. 2. Place the client in an upright position. 3. Compare current data to baseline data. 4. Slow the rate of the blood transfusion

Compare current data to baseline data. For the client receiving a blood transfusion, the nurse should monitor for potential complications of a transfusion. One of the complications is circulatory overload. Signs and symptoms of circulatory overload include cough, dyspnea, chest pain, wheezing on auscultation of the lungs, headache, hypertension, tachycardia and a bounding pulse, and distended neck veins. Based on the data in the question, the nurse should compare current data to baseline data. The nurse should also further assess the client for other signs and symptoms of circulatory overload. If the nurse still suspects this complication after comparing to baseline data, the nurse should then place the client in an upright position with the feet in a dependent position and slow the rate of the infusion. Collection of a urine sample should occur if the nurse suspects a transfusion reaction, such as a hemolytic reaction.

The nurse in a neonatal intensive care unit (NICU) receives a telephone call to prepare for the admission of a 43-week gestation newborn with Apgar scores of 1 and 4. In planning for admission of this newborn, what is the nurse's highest priority? 1. Turn on the apnea and cardiorespiratory monitors. 2. Connect the resuscitation bag to the oxygen outlet. 3. Set up the intravenous line with 5% dextrose in water. 4. Set the radiant warmer control temperature at 36.5°C (97.6°F).

Connect the resuscitation bag to the oxygen outlet. The highest priority on admission to the nursery for a newborn with a low Apgar score is the airway, which would involve preparing respiratory resuscitation equipment and oxygen. The remaining options are also important, although they are of lower priority. The newborn would be placed on an apnea and cardiorespiratory monitor. Setting up an intravenous line with 5% dextrose in water would provide circulatory support. The radiant warmer would provide an external heat source, which is necessary to prevent further respiratory distress.

A client whose cardiac rhythm was normal sinus rhythm suddenly exhibits a different rhythm on the monitor. The nurse should take which action? Refer to Figure. 1. Continue to watch the monitor. 2. Contact the health care provider (HCP). 3. Check to see if cardiac medications are due. 4. Call respiratory therapy for a respiratory treatment.

Contact the health care provider (HCP). Atrial fibrillation is characterized by multiple rapid impulses from many atrial foci in a totally disorganized manner at a rate of 350 to 600 times per minute. The atria quiver in fibrillation. As a result, thrombi can form within the right atrium and move through the right ventricle to the lungs. This can be a life-threatening situation requiring pharmacological therapy. Therefore, the HCP needs to be contacted. Continuing to monitor and checking to see if cardiac medications are due delay necessary and required interventions. A respiratory treatment is not useful for this situation.

A client with myocardial infarction suddenly becomes tachycardic, shows signs of air hunger, and begins coughing frothy, pink-tinged sputum. Which finding would the nurse anticipate when auscultating the client's breath sounds? 1. Stridor 2. Crackles 3. Scattered rhonchi 4. Diminished breath sounds

Crackles Pulmonary edema is characterized by extreme breathlessness, dyspnea, air hunger, and the production of frothy, pink-tinged sputum. Auscultation of the lungs reveals crackles. Rhonchi and diminished breath sounds are not associated with pulmonary edema. Stridor is a crowing sound associated with laryngospasm or edema of the upper airway.

A client receiving parenteral nutrition (PN) in the home setting has a weight gain of 5 lb in 1 week. The nurse should next assess the client for the presence of which condition? 1. Thirst 2. Polyuria 3. Decreased blood pressure 4. Crackles on auscultation of the lungs

Crackles on auscultation of the lungs Optimal weight gain when the client is receiving PN is 1 to 2 lb/week. The client who has a weight gain of 5 lb/week while receiving PN is likely to have fluid retention. This can result in hypervolemia. Signs of hypervolemia include increased blood pressure, crackles on lung auscultation, a bounding pulse, jugular vein distention, headache, peripheral edema, and weight gain more than desired. Thirst and polyuria are associated with hyperglycemia. A decreased blood pressure is likely to be noted in deficient fluid volume.

Which nursing action is essential prior to initiating a new prescription for 500 mL of fat emulsion (lipids) to infuse at 50 mL/hour? 1. Ensure that the client does not have diabetes. 2. Determine whether the client has an allergy to eggs. 3. Add regular insulin to the fat emulsion, using aseptic technique. 4. Contact the health care provider (HCP) to have a central line inserted for fat emulsion infusion.

Determine whether the client has an allergy to eggs. The client beginning infusions of fat emulsions must be first assessed for known allergies to eggs to prevent anaphylaxis. Egg yolk is a component of the solution and provides emulsification. The remaining options are unnecessary and are not related specifically to the administration of fat emulsion.

The emergency department nurse is preparing to administer fomepizole to a client suspected of ingesting antifreeze solution during a suicidal attempt. The nurse should prepare to administer this medication by which method? 1. Direct intravenous (IV) bolus 2. Diluting the medication and administering it rapidly by the IV route 3. Administering the medication through a nasogastric tube, followed by activated charcoal 4. Diluting the medication in 100 mL of 0.9% normal saline and administering it over 30 minutes

Diluting the medication in 100 mL of 0.9% normal saline and administering it over 30 minutes Fomepizole is used for the treatment of known or suspected ethylene glycol (antifreeze) intoxication. It is administered via the IV route, is not administered undiluted, and is not administered by rapid IV infusion. It is diluted in at least 100 mL of 0.9% normal saline or 5% dextrose in water and administered over a 30-minute period

Acetylcysteine is prescribed for a client in the hospital emergency department after diagnosis of acetaminophen overdose. The nurse prepares to administer the medication using which procedure? 1. Diluting the medication in cola and administering it to the client orally 2. Calling the respiratory department to administer the medication via inhaler 3. Obtaining a 1-mL syringe to administer the small dose via the subcutaneous route 4. Obtaining an appropriate-size syringe and needle for intramuscular injection in the ventrogluteal muscle

Diluting the medication in cola and administering it to the client orally Acetylcysteine can be given orally or by nasogastric tube to treat acetaminophen overdose. It is administered by inhalation for use as a mucolytic. Before administration of the medication as an antidote, the nurse would ensure that the client's stomach is empty through emesis or gastric lavage. The solution is diluted in cola, water, or juice to make the solution more palatable. It is not administered via nebulizer, subcutaneously, or intramuscularly for the client experiencing acetaminophen overdose.

A client with rapid-rate atrial fibrillation has a new prescription for diltiazem hydrochloride by intravenous (IV) bolus followed by a continuous IV infusion of the same medication. What should the nurse plan for with the administration of this medication? 1. Applying a nonrebreather mask 2. Discontinuing the infusion after 24 hours 3. Monitoring the cardiac rhythm every hour 4. Administering the IV bolus over 2 to 3 seconds

Discontinuing the infusion after 24 hours Diltiazem hydrochloride is a calcium channel blocker used in the treatment of atrial flutter and fibrillation. It decreases myocardial contractility and workload, thereby decreasing the need for oxygen. A bolus of 0.25 mg/kg is given slowly over 2 minutes, and a continuous infusion of 5 to 10 mg/hour may be administered for up to 24 hours. Therefore, the nurse should prepare to discontinue the infusion after 24 hours. Upon discontinuation of infusion, heart rate reduction may last from 0.5 hours to more than 10 hours (median duration 7 hours). A nonrebreather mask is not necessary. The client's cardiac rhythm is monitored continuously.

The nurse is preparing to hang the first bag of parenteral nutrition (PN) solution via the central line of an assigned client. The nurse should obtain which most essential piece of equipment before hanging the solution? 1. Urine test strips 2. Blood glucose meter 3. Electronic infusion pump 4. Noninvasive blood pressure monitor

Electronic infusion pump The nurse obtains an electronic infusion pump before hanging a PN solution. Because of the high glucose content, use of an infusion pump is necessary to ensure that the solution does not infuse too rapidly or fall behind. Because the client's blood glucose level is monitored every 4 to 6 hours during administration of PN, a blood glucose meter also will be needed, but this is not the most essential item needed before hanging the solution because it is not directly related to administering the PN. Urine test strips (to measure glucose) rarely are used because of the advent of blood glucose monitoring. Although the blood pressure will be monitored, a noninvasive blood pressure monitor is not the most essential piece of equipment needed for this procedure.

The nurse is developing a plan of care for a client who sustained an inhalation burn injury. Which nursing intervention should the nurse include in the plan of care for this client? 1. Elevate the head of the bed. 2. Monitor oxygen saturation levels every 4 hours. 3. Encourage coughing and deep breathing every 4 hours. 4. Assess respiratory rate and breath sounds every 4 hours.

Elevate the head of the bed. Nursing interventions for the client with an inhalation burn injury include assessing the respiratory rate every hour, monitoring oxygen saturation levels every hour, and assisting the client in coughing and deep breathing every hour. The head of the bed is elevated to facilitate lung expansion.

A client experiencing cocaine toxicity is brought to the emergency department. The nurse should prepare to take which initial action? 1. Administer naloxone. 2. Ensure a patent airway. 3. Establish an intravenous access. 4. Obtain a 12-lead electrocardiogram (ECG).

Ensure a patent airway. Initial management when caring for a client with cocaine toxicity is to ensure a patent airway. Although options 1, 3, and 4 are components of care, airway is the priority.

The nurse is monitoring the status of a client's fat emulsion (lipid) infusion and notes that the infusion is 1 hour behind. Which action should the nurse take? 1. Adjust the infusion rate to catch up over the next hour. 2. Increase the infusion rate to catch up over the next 2 hours. 3. Ensure that the fat emulsion infusion rate is infusing at the prescribed rate. 4. Adjust the infusion rate to run wide open until the solution is back on time.

Ensure that the fat emulsion infusion rate is infusing at the prescribed rate. The nurse should not increase the rate of a fat emulsion to make up the difference if the infusion timing falls behind. Doing so could place the client at risk for fat overload. In addition, increasing the rate suddenly can cause fluid overload. The same principle (not increasing the rate) applies to parenteral nutrition or any intravenous infusion. Therefore, the remaining options are incorrect.

The nurse notes redness, warmth, and a yellowish drainage at the insertion site of a central venous catheter in a client receiving total parenteral nutrition. What is the nurse's initial action? 1. Administer diphenhydramine. 2. Decrease the rate of infusion. 3. Notify the health care provider (HCP). 4. Evaluate for signs of septicemia.

Evaluate for signs of septicemia. Redness, warmth, and purulent drainage are signs of an infection, not of an allergic reaction. Infiltration causes the surrounding tissue to become cool and pale. An infection of a central venous catheter site can lead to septicemia. The nurse should assess for signs of septicemia and then notify the HCP so that all of the assessment data can be reported. If infection is suspected, the infusion should be stopped. Diphenhydramine is prescribed for allergic reactions.

The nurse is performing a vaginal assessment of a pregnant woman who is in labor. The nurse notes that the umbilical cord is protruding from the vagina. The nurse would immediately take which action? 1. Administer oxygen to the woman. 2. Transport the woman to the delivery room. 3. Place an external fetal monitor on the woman. 4. Exert upward pressure against the presenting part.

Exert upward pressure against the presenting part. If the umbilical cord is protruding from the vagina, no attempt should be made to replace it because doing so could traumatize it and further reduce blood flow. The nurse would place a gloved hand into the vagina to the cervix and exert upward pressure against the presenting part to relieve compression of the cord. The nurse also would wrap the cord loosely in a sterile towel saturated with warm, sterile normal saline solution. Oxygen, 8 to 10 L/min by face mask, would be administered to the mother to increase fetal oxygenation, and the woman would be prepared for immediate delivery. However, the immediate action is to relieve pressure on the cord. The woman should already have an external fetal monitor in place.

The hospital code team is responding to a respiratory emergency of a client admitted during the night with a fractured hip and pelvis after a motor vehicle collision (MVC). The client first became confused and then developed dyspnea, chest pain, and a petechial rash on his neck, upper arms, and chest. What condition is this client at risk for? 1. Pulmonary fibrosis 2. Hypovolemic shock 3. Blood clot embolism 4. Fat embolism syndrome (FES)

Fat embolism syndrome (FES) FES generally happens within 12 to 48 hours after a long bone, hip, or pelvis fracture and occurs most often in young men between ages 20 and 40 years and in older adults between ages 70 and 80. FES is a serious complication in which fat globules are released from the yellow bone marrow into the bloodstream and clog small blood vessels that supply vital organs, most commonly the lungs.

The nurse plans to administer a medication by intravenous (IV) bolus through the primary IV line. The nurse notes that the medication is incompatible with the primary IV solution. Which is the appropriate nursing action to safely administer the medication? 1. Start a new IV line for the medication. 2. Flush the tubing after the medication with sterile water. 3. Flush the tubing before and after the medication with normal saline. 4. Call the health care provider for a prescription to change the route of the medication.

Flush the tubing before and after the medication with normal saline. When giving a medication by IV bolus, if the medication is incompatible with the IV solution, the tubing is flushed before and after the medication with infusions of normal saline. Option 1 is premature and not necessary. Sterile water is not used for an IV flush. Option 4 is inappropriate.

A client is brought to the emergency department having experienced blood loss related to an arterial laceration. Which blood component should the nurse expect the health care provider to prescribe? 1. Platelets 2. Granulocytes 3. Fresh-frozen plasma 4. Packed red blood cells

Fresh-frozen plasma Fresh-frozen plasma is often used for volume expansion as a result of fluid and blood loss. It is rich in clotting factors and can be thawed quickly and transfused quickly. Platelets are used to treat thrombocytopenia and platelet dysfunction. Granulocytes may be used to treat a client with sepsis or a neutropenic client with an infection that is unresponsive to antibiotics. Packed red blood cells are a blood product used to replace erythrocytes.

The nurse is caring for a client who has overdosed on amphetamines. The nurse anticipates noting which assessment finding in this client? 1. Bradypnea 2. Bradycardia 3. Hypothermia 4. Hypertension

Hypertension An overdose from amphetamines can cause agitation, increased temperature, increased pulse, increased respiratory rate, increased blood pressure, cardiac dysrhythmias, myocardial infarction, hallucinations, seizures, and possible death. Therefore, the remaining options are incorrect.

A client receiving parenteral nutrition (PN) complains of a headache. The nurse notes that the client has an increased blood pressure, bounding pulse, jugular vein distention, and crackles bilaterally. The nurse determines that the client is experiencing which complication of PN therapy? 1. Sepsis 2. Air embolism 3. Hypervolemia 4. Hyperglycemia

Hypervolemia Hypervolemia is a critical situation and occurs from excessive fluid administration or administration of fluid too rapidly. Clients with cardiac, renal, or hepatic dysfunction are also at increased risk. The client's signs and symptoms presented in the question are consistent with hypervolemia. The increased intravascular volume increases the blood pressure, whereas the pulse rate increases as the heart tries to pump the extra fluid volume. The increased volume also causes neck vein distention and shifting of fluid into the alveoli, resulting in lung crackles. The signs and symptoms presented in the question do not indicate sepsis, air embolism, or hyperglycemia.

To perform cardiopulmonary resuscitation (CPR), the nurse should use the method pictured to open the airway in which situation? Refer to figure. 1. If neck trauma is suspected 2. In all situations requiring CPR 3. If the client has a history of seizures 4. If the client has a history of headaches

If neck trauma is suspected The jaw thrust without the head tilt maneuver is used when head or neck trauma is suspected. This maneuver opens the airway while maintaining proper head and neck alignment, reducing the risk of further damage to the neck. Options 2, 3, and 4 are incorrect. In addition, it is unlikely that the nurse would be able to obtain data about the client's history.

A client is scheduled for insertion of a peripherally inserted central catheter, and the nurse explains the advantages of this catheter. Which statement by the client indicates a need for follow-up? 1. "It is reasonable in cost." 2. "This type of catheter is very reliable." 3. "It is specifically designed for short-term use." 4. "I should not have pain or discomfort with this catheter."

It is specifically designed for short-term use." Peripherally inserted central catheters are intended to be used for clients who need long-term catheter placement. They can be left in place for several months. It is reasonable in cost because the catheter does not need routine replacement, as do traditional peripheral intravenous catheters. The catheter is more comfortable for the client because there is no repeated venipuncture with catheter change. The catheter is also very reliable. It is less likely to infiltrate and can be used for administration of a number of types of medications.

The nurse is teaching chest compressions for cardiopulmonary resuscitation (CPR) to a group of lay clients. Which behavior by one of the participants indicates a need for further teaching? 1. Keeping the shoulders directly over the hands 2. Straightening the arms and locking the elbows 3. Letting the right and left fingers rest on the chest 4. Performing compressions on the lower half of the sternum

Letting the right and left fingers rest on the chest To maximize the effectiveness of chest compressions, the rescuer avoids letting the fingers rest on the chest. This also helps prevent accidental injury to internal organs. The actions listed in the other options are all part of correct CPR procedures.

The nurse is performing an assessment on a client who was admitted with a diagnosis of carbon monoxide poisoning. Which assessment performed by the nurse would primarily elicit data related to a deterioration of the client's condition? 1. Skin color 2. Apical rate 3. Respiratory rate 4. Level of consciousness

Level of consciousness The neurological system is primarily affected by carbon monoxide poisoning. With high levels of carbon monoxide, the neurological status progressively deteriorates. Although skin color, apical rate, and respiratory rate would be components of the assessment of the client with carbon monoxide poisoning, assessment of the neurological status of the client would elicit data specific to a deterioration in the client's condition.

A client who has just suffered a large flail chest is experiencing severe pain and dyspnea. The client's central venous pressure (CVP) is rising, and the arterial blood pressure is falling. Which condition should the nurse interpret that the client is experiencing? 1. Fat embolism 2. Mediastinal shift 3. Mediastinal flutter 4. Hypovolemic shock

Mediastinal flutter The client with severe flail chest will have significant paradoxical chest movement. This causes the mediastinal structures to swing back and forth with respiration. This movement can affect hemodynamics. Specifically, the client's CVP rises, the filling of the right side of the heart is impaired, and the arterial blood pressure falls. This is referred to as mediastinal flutter. The client with fat embolism experiences chest pain and dyspnea, but this condition occurs as a complication of a bone fracture. Mediastinal shift is a condition in which the structures of the mediastinum shift or move to the opposite side of the chest cavity; this may be caused by a pleural effusion or tension pneumothorax. In hypovolemic shock, the blood pressure falls and the pulse rises; this occurs following hemorrhage.

A client develops atrial fibrillation with a ventricular rate of 140 beats/minute and signs of decreased cardiac output. Which medication should the nurse anticipate administering first? 1. Warfarin 2. Lidocaine 3. Metoprolol 4. Atropine sulfate

Metoprolol Beta blockers such as metoprolol slow conduction of impulses through the atrioventricular node and decrease the heart rate. In rapid atrial fibrillation, the goal first is to slow the ventricular rate and improve the cardiac output and then attempt to restore normal sinus rhythm. Atropine sulfate will further increase the heart rate and will further decrease the cardiac output. Although warfarin is administered to clients with atrial fibrillation to prevent clots from forming in the atria, it will have no effect in decreasing the ventricular rate or restoring normal sinus rhythm. Lidocaine is useful only in suppressing ventricular dysrhythmias.

The nurse is caring for a client with acute pulmonary edema. The health care provider (HCP) tells the nurse that medication will be prescribed to help reduce preload and afterload. Based on the HCP's statement, what medication should the nurse anticipate administering? 1. Digoxin 2. Prednisone 3. Furosemide 4. Nitroprusside sodium

Nitroprusside sodium Intravenous nitroprusside is a potent vasodilator that reduces preload and afterload. It is a medication used to treat the client with pulmonary edema. Prednisone is a steroidal antiinflammatory medication that is not usually prescribed for acute pulmonary edema and could aggravate the symptoms due to sodium and retention effects of this medication. Digoxin is a cardiac glycoside that increases cardiac contractility. Furosemide is a loop diuretic and can reduce preload by enhancing the renal excretion of sodium and water, which reduces circulating blood volume. Furosemide is often prescribed for acute pulmonary edema, but the action of the medication is not to decrease both preload and afterload.

The nurse is performing an assessment on a client who has been receiving total parenteral nutrition (TPN) at 125 mL/hour. On assessment, the nurse notes the presence of bilateral crackles in the lungs and 2+ pedal edema. The nurse also notes that the client has gained 3 pounds (1.5 kg) in 5 days. Which nursing action would be most appropriate for this client? 1. Slow the infusion rate to 100 mL/hour. 2. Encourage the client to cough and deep breathe. 3. Notify the health care provider (HCP) of the assessment findings. 4. Administer the prescribed daily diuretic and reassess the client in 2 hours.

Notify the health care provider (HCP) of the assessment findings. Crackles, edema, and weight gain signify fluid shifts from intravascular spaces to the interstitial spaces and indicate signs of fluid retention and possible excess fluid intake. The problem may or may not be related to the TPN. Other possible causes of fluid retention are impaired respiratory and cardiovascular function and impaired kidney function. Deep breathing and coughing will have little, if any, effect on peripheral edema and weight gain. Administering the prescribed daily diuretic and reassessing in 2 hours may delay necessary and immediate treatment and is incorrect. Decreasing the rate of infusion by 25 mL is not very helpful; in addition, the nurse should obtain an HCP prescription for doing so because increasing or decreasing the rate of the infusion presents the potential for hyperosmolar diuresis, hypoglycemia, or hyperglycemia.

The nurse is caring for a client following enucleation and notes the presence of bright red drainage on the dressing. Which most appropriate action should the nurse take at this time? 1. Document the finding. 2. Continue to monitor the drainage. 3. Notify the health care provider (HCP). 4. Mark the drainage on the dressing and monitor for any increase in bleeding.

Notify the health care provider (HCP). If the nurse notes the presence of bright red drainage on the dressing, it must be reported to the HCP, because this indicates hemorrhage. Options 1, 2, and 4 are inappropriate at this time.

A child undergoes surgical removal of a brain tumor. During the postoperative period, the nurse notes that the child is restless, the pulse rate is elevated, and the blood pressure has decreased significantly from the baseline value. The nurse suspects that the child is in shock. Which is the most appropriate nursing action? 1. Place the child in a supine position. 2. Notify the health care provider (HCP). 3. Place the child in Trendelenburg's position. 4. Increase the flow rate of the intravenous fluids.

Notify the health care provider (HCP). In the event of shock, the HCP is notified immediately before the nurse changes the child's position or increases intravenous fluids. After craniotomy, a child is never placed in the supine or Trendelenburg's position because it increases intracranial pressure (ICP) and the risk of bleeding. The head of the bed should be elevated. Increasing intravenous fluids can cause an increase in ICP.

A client's arterial blood gas results reveal a PaO2 of 55 mm Hg. The client's admitting diagnosis is acute respiratory failure secondary to community-acquired pneumonia. What is the nurse's best action? 1. Repeat arterial blood gas testing. 2. Maintain continuous pulse oximetry. 3. Notify the health care provider (HCP). 4. Decrease the amount of oxygen administered.

Notify the health care provider (HCP). Respiratory failure is defined as a PaO2 of 60 mm Hg or lower. The nurse should notify the HCP for further prescriptions. Common causes of hypoxemic respiratory failure are pneumonia, pulmonary embolism, and shock. This client should be receiving oxygen. Repeating the arterial blood gases and maintaining continuous pulse oximetry do nothing to correct the problem.

The nurse is preparing to hang fat emulsion (lipids) and notes that fat globules are visible at the top of the solution. The nurse should take which action? 1. Roll the bottle of solution gently. 2. Obtain a different bottle of solution. 3. Shake the bottle of solution vigorously. 4. Run the bottle of solution under warm water.

Obtain a different bottle of solution. Fat emulsion (lipids) is a white, opaque solution administered intravenously during parenteral nutrition therapy to prevent fatty acid deficiency. The nurse should examine the bottle of fat emulsion for separation of emulsion into layers of fat globules or for the accumulation of froth. The nurse should not hang a fat emulsion if any of these are observed and should return the solution to the pharmacy. Therefore, the remaining options are inappropriate actions.

The nurse is caring for a client with a chest tube who accidentally disconnects the tube from the drainage system when trying to get out of bed. The nurse immerses the end of the tube in sterile water. What immediate action should the nurse take? 1. Obtain a new drainage system. 2. Ask the client to hold his or her breath. 3. Place the client in a prone position. 4. Place a sterile dressing over the chest tube insertion site.

Obtain a new drainage system. If the drainage system is broken or interrupted or the tube disconnects, the end of the tube should be placed in a bottle of sterile water held below the level of the chest. A new drainage system is then immediately obtained and set up. Placing the client in the prone position and asking the client to hold his or her breath are not helpful. Placing a sterile dressing over the disconnection site will not prevent complications resulting from the tube disconnection. The nurse should also perform an assessment on the client and contact the health care provider.

The nurse is preparing a continuous intravenous (IV) infusion at the medication cart. As the nurse goes to insert the spike end of the IV tubing into the IV bag, the tubing drops and the spike end hits the top of the medication cart. The nurse should take which action? 1. Obtain a new IV bag. 2. Obtain new IV tubing. 3. Wipe the spike end of the tubing with povidone iodine. 4. Scrub the spike end of the tubing with an alcohol swab.

Obtain new IV tubing. The nurse should obtain new IV tubing because contamination has occurred and could cause systemic infection to the client. There is no need to obtain a new IV bag because the bag was not contaminated. Wiping with povidone iodine or alcohol is insufficient and is contraindicated because the spike will be inserted into the IV bag.

The nurse is planning care for a client being admitted to the nursing unit who attempted suicide. Which priority nursing intervention should the nurse include in the plan of care? 1. One-to-one suicide precautions 2. Suicide precautions with 30-minute checks 3. Checking the whereabouts of the client every 15 minutes 4. Asking the client to report suicidal thoughts immediately

One-to-one suicide precautions One-to-one suicide precautions are required for a client who has attempted suicide. Options 2 and 3 may be appropriate, but not at the present time, considering the situation. Option 4 also may be an appropriate nursing intervention, but the priority is identified in the correct option. The best intervention is constant supervision so that the nurse may intervene as needed if the client attempts to harm himself or herself.

External public access defibrillator (PAD) interprets that the rhythm of a pulseless victim is ventricular fibrillation and advises defibrillation. Which action should the rescuer take next? 1. Administer rescue breathing during the defibrillation. 2. Perform cardiopulmonary resuscitation (CPR) for 1 minute before defibrillating. 3. Charge the machine and immediately push the discharge buttons on the console. 4. Order people away from the client, charge the machine, and depress the discharge buttons.

Order people away from the client, charge the machine, and depress the discharge buttons. If the victim is in ventricular fibrillation, defibrillation is necessary. If the PAD advises to defibrillate, the rescuer orders all people away from the client, charges the machine, and pushes both of the discharge buttons on the console at the same time. The charge is delivered through the patch electrodes, so this method is known as "hands off" defibrillation, which is safer for the rescuer. The sequence of charges is similar to that of conventional defibrillation.

A client with no history of respiratory disease is admitted to the hospital with respiratory failure. Which results on the arterial blood gas report that are consistent with this disorder should the nurse expect to note? 1. PaO2 58 mm Hg, PaCO2 32 mm Hg 2. PaO2 60 mm Hg, PaCO2 45 mm Hg 3. PaO2 49 mm Hg, PaCO2 52 mm Hg 4. PaO2 73 mm Hg, PaCO2 62 mm Hg

PaO2 49 mm Hg, PaCO2 52 mm Hg Respiratory failure is described as a PaO2 of 60 mm Hg or lower and a PaCO2 of 50 mm Hg or higher in a client with no history of respiratory disease. In a client with a history of a respiratory disorder with hypercapnia, increases of 5 mm Hg or more (PaCO2) from the client's baseline are considered diagnostic

A client with a history of gastric ulcer complains of a sudden, sharp, severe pain in the midepigastric area, which then spreads over the entire abdomen. The client's abdomen is rigid and boardlike on palpation, and the client obtains most comfort from lying in the knee-chest position. The nurse suspects which condition and should perform which action? 1. Obstruction; call the operating room. 2. Perforation; notify the health care provider. 3. Intractability; administer cleansing enema. 4. Hemorrhage; increase intravenous fluid rate.

Perforation; notify the health care provider. The signs and symptoms described in the question are consistent with perforation of the ulcer, which may progress to peritonitis if the perforation is large enough. A client with intestinal obstruction would most likely complain of abdominal pain and distention and nausea and vomiting. Intractability is a term that refers to continued manifestations of a disease process despite ongoing medical treatment. A client with hemorrhage would vomit blood or coffee-ground material or would expel black, tarry, or bloody stools.

A client has just undergone insertion of a central venous catheter at the bedside. The nurse would be sure to check which results before initiating the flow rate of the client's intravenous (IV) solution at 100 mL/hour? 1. Serum osmolality 2. Serum electrolyte levels 3. Portable chest x-ray film 4. Intake and output record

Portable chest x-ray film Before beginning administration of IV solution, the nurse should assess whether the chest radiograph reveals that the central catheter is in the proper place. This is necessary to prevent infusion of IV fluid into pulmonary or subcutaneous tissues. The other options represent items that are useful for the nurse to be aware of in the general care of this client, but they do not relate to this procedure.

A client with a spinal cord injury suddenly experiences an episode of autonomic dysreflexia. After checking the client's vital signs, the nurse takes the following actions. Arrange the actions in the order they should be performed. All options must be used.

Raise the head of the bed. Loosen tight clothing on the client Check for bladder distention. Contact the health care provider (HCP). Administer an antihypertensive medication. Document the occurrence, treatment, and response. Autonomic dysreflexia is characterized by severe hypertension, bradycardia, severe headache, nasal stuffiness, and flushing. The cause is a noxious stimulus, most often a distended bladder or constipation. Autonomic dysreflexia is a neurological emergency and must be treated promptly to prevent a hypertensive stroke. Immediate nursing actions are to sit the client up in bed in a high Fowler's position and to remove the noxious stimulus. The nurse would loosen any tight clothing and then check for bladder distention. If the client has a Foley catheter, the nurse would check for kinks in the tubing. The nurse also would check for a fecal impaction and would disimpact the client, if necessary. The HCP is then contacted, especially if these actions do not relieve the signs and symptoms. Antihypertensive medication may be prescribed by the HCP to minimize cerebral hypertension. Finally, the nurse documents the occurrence, treatment, and client response.

A client with pancreatitis is being weaned from total parenteral nutrition (TPN). The client asks the nurse why the TPN cannot just be stopped. What is the nurse's best response? 1. Dehydration can result. 2. Hypokalemia may occur. 3. Hypernatremia will occur. 4. Rebound hypoglycemia is a risk.

Rebound hypoglycemia is a risk. Clients receiving TPN are receiving high concentrations of glucose. To give the pancreas time to adjust to decreasing glucose loads, the infusion rates are tapered down. Before discontinuing the TPN, the body must adjust to the lowered glucose level. If the TPN were suddenly withdrawn, the client could have rebound hypoglycemia. Although the other options are potential complications, they are not risks associated with discontinuing TPN abruptly.

The nurse has just obtained a unit of blood from the blood bank to transfuse into a client as prescribed. Before preparing the blood for transfusion, the nurse looks for which member of the health care team to assist in checking the unit of blood? 1. Phlebotomist 2. Medical student 3. Registered nurse (RN) 4. Blood bank technician

Registered nurse (RN) Depending on agency policy, two RNs or one RN and one licensed practical nurse (LPN) must check the label on the blood product together against the client's identification number, blood group, and complete name. This minimizes the risk of error in checking information on the blood bag and thereby minimizes the risk of harm or injury to the client. A blood bank technician verifies data with the nurse when the blood is obtained from the blood bank but does not verify information on the nursing unit or at the client's bedside. The other options are also incorrect.

The nurse notes that the client's intravenous (IV) site is cool, pale, and swollen and that the solution is not infusing. What is the nurse's priority action? 1. Elevate the extremity. 2. Remove the IV catheter. 3. Assess for signs of infection. 4. Decrease the rate of infusion.

Remove the IV catheter. The client's IV has infiltrated. An infiltrated IV is one that has dislodged from the vein and is lying in subcutaneous tissue. Pallor, coolness, and swelling are the results of IV fluid being deposited in the subcutaneous tissue. When the pressure in the tissues exceeds the pressure in the tubing, the flow of the IV solution will stop. The corrective action is to remove the catheter and start a new IV line at another site. Infection, phlebitis, and thrombosis are likely to be accompanied by warmth at the site, not coolness. Elevating the extremity should be implemented after removing the IV to reduce swelling.

A client experienced an open pneumothorax (sucking wound), which has been covered with an occlusive dressing. The client begins to experience severe dyspnea, and the blood pressure begins to fall. The nurse should first perform which action? 1. Remove the dressing. 2. Reinforce the dressing. 3. Call the health care provider (HCP). 4. Measure oxygen saturation by oximetry.

Remove the dressing. Placement of a dressing over a chest wound could convert an open pneumothorax to a closed (tension) pneumothorax. This may result in a sudden decline in respiratory status, mediastinal shift with twisting of the great vessels, and circulatory compromise. If clinical changes occur, the nurse should remove the dressing immediately, allowing air to escape. Therefore, reinforcing the dressing is an incorrect action. The nurse would measure oxygen saturation by oximetry and would call the HCP, but these would not be the first actions in this situation.

The nurse is caring for a client with a diagnosis of myocardial infarction (MI). The client is experiencing chest pain that is unrelieved by the administration of nitroglycerin. The nurse administers morphine sulfate to the client as prescribed by the health care provider. After administration of the morphine sulfate, what is the priority assessment 1. Respirations 2. Mental status 3. Urinary output 4. Blood pressure

Respirations Morphine sulfate is an opioid analgesic that may be administered to relieve pain in a client with MI. The nurse would monitor the client's respirations and blood pressure. Signs of morphine toxicity include respiratory depression and hypotension. Respiratory depression is the priority concern using the ABCs-Airway, Breathing, and Circulation-assessment. Although monitoring mental status is a component of the nurse's assessment, it is not the priority after administration of morphine sulfate. Urinary output is unrelated to the administration of this medication. Monitoring the temperature also is not associated with the use of this medication.

A client receiving a transfusion of packed red blood cells (PRBCs) begins to vomit. The client's blood pressure is 90/50 mm Hg from a baseline of 125/78 mm Hg. The client's temperature is 100.8°F (38.2°C) orally from a baseline of 99.2°F (37.3°C) orally. The nurse determines that the client may be experiencing which complication of a blood transfusion? 1. Septicemia 2. Hyperkalemia 3. Circulatory overload 4. Delayed transfusion reaction

Septicemia Septicemia occurs with the transfusion of blood contaminated with microorganisms. Signs include chills, fever, vomiting, diarrhea, hypotension, and the development of shock. Hyperkalemia causes weakness, paresthesias, abdominal cramps, diarrhea, and dysrhythmias. Circulatory overload causes cough, dyspnea, chest pain, wheezing, tachycardia, and hypertension. A delayed transfusion reaction can occur days to years after a transfusion. Signs include fever, mild jaundice, and a decreased hematocrit level.

The nurse is preparing to administer lipid emulsion to a client who has just been started on total parenteral nutrition. Before administering the lipid emulsion, the nurse asks the client about allergies. The nurse should withhold the lipid emulsion and contact the health care provider if the client identifies an allergy to which food item? 1. Milk 2. White bread 3. Soybean oil 4. Strawberries

Soybean oil Most fat emulsions are prepared from soybean oil, and the primary components are linoleic, oleic, palmitic, linolenic, and stearic acids. The nurse would ask the client about an allergy to soybean oil. The remaining options are not a concern.

A health care provider has written a prescription to discontinue an intravenous (IV) line. The nurse should obtain which item from the unit supply area for applying pressure to the site after removing the IV catheter? 1. Elastic wrap 2. Povidone iodine swab 3. Adhesive bandage 4. Sterile 2 × 2 gauze

Sterile 2 × 2 gauze A dry sterile dressing such as a sterile 2 × 2 gauze is used to apply pressure to the discontinued IV site. This material is absorbent, sterile, and nonirritating. A povidone iodine swab would irritate the opened puncture site and would not stop the blood flow. An adhesive bandage or elastic wrap may be used to cover the site once hemostasis has occurred.

The nurse is caring for a client who is receiving feedings by nasogastric tube. The client suddenly begins to vomit, and the nurse quickly repositions the client. The client is coughing and having difficulty breathing. What is the nurse's priority action? 1. Call a code. 2. Suction the client. 3. Check the client's vital signs 4. Call the health care provider (HCP

Suction the client. This client is at high risk for aspiration due to vomiting with a nasogastric tube in place. If the client aspirates a feeding, the nurse should immediately suction the client's airway. The client's respiratory status will then be monitored closely until a normal respiratory pattern resumes. The question presents no data indicating the need to call a code. The client's vital signs may need to be monitored, but this is not the priority action. Although the HCP may need to be notified, ensuring a patent airway is the priority.

The nurse is preparing to change the parenteral nutrition (PN) solution bag and tubing. The client's central venous line is located in the right subclavian vein. The nurse asks the client to take which essential action during the tubing change? 1. Breathe normally. 2. Turn the head to the right. 3. Exhale slowly and evenly. 4. Take a deep breath, hold it, and bear down.

Take a deep breath, hold it, and bear down. The client should be asked to perform the Valsalva maneuver during tubing changes. This helps avoid air embolism during tubing changes. The nurse asks the client to take a deep breath, hold it, and bear down. If the intravenous line is on the right, the client turns his or her head to the left. This position increases intrathoracic pressure. Breathing normally and exhaling slowly and evenly are inappropriate and could enhance the potential for an air embolism during the tubing change.

A client presents to the urgent care center with complaints of abdominal pain and vomits bright red blood. Which is the priority nursing action? 1. Take the client's vital signs. 2. Perform a complete abdominal assessment. 3. Obtain a thorough history of the recent health status. 4. Prepare to insert a nasogastric tube and test pH and occult blood.

Take the client's vital signs. The nurse should take the client's vital signs first to determine whether they are hypovolemic or in shock from blood loss. This also provides a baseline blood pressure and pulse by which to gauge the effectiveness of treatment. Signs and symptoms of shock include low blood pressure; rapid, weak pulse; increased thirst; cold, clammy skin; and restlessness. Therefore, the remaining options are incorrect.

A client has been discharged to home on parenteral nutrition (PN). With each visit, the home care nurse should assess which parameter most closely in monitoring this therapy? 1. Pulse and weight 2. Temperature and weight 3. Pulse and blood pressure 4. Temperature and blood pressure

Temperature and weight The client receiving PN at home should have her or his temperature monitored as a means of detecting infection, which is a potential complication of this therapy. An infection also could result in sepsis because the catheter is in a blood vessel. The client's weight is monitored as a measure of the effectiveness of this nutritional therapy and to detect hypervolemia. The pulse and blood pressure are important parameters to assess, but they do not relate specifically to the effects of PN.

An emergency department nurse is caring for a conscious child who was brought to the emergency department after the ingestion of half a bottle of acetylsalicylic acid (aspirin). The nurse anticipates that which will be the initial treatment? 1. Placement of a dialysis catheter 2. The administration of an emetic 3. The administration of vitamin K 4. The administration of sodium bicarbonate

The administration of an emetic Initial treatment of acetylsalicylic acid overdose includes the administration of an emetic or gastric lavage. Activated charcoal may be administered to decrease absorption. Fluids and sodium bicarbonate may be administered intravenously to enhance excretion but would not be the initial treatment. Dialysis is used in extreme cases if the child is unresponsive to therapy. Vitamin K is the antidote for warfarin sodium overdose.

The child with croup is being discharged from the hospital. The nurse provides instructions to the mother and advises the mother to bring the child to the emergency department if which occurs? 1. The child is irritable. 2. The child appears tired. 3. The child develops stridor. 4. The child takes fluids poorly.

The child develops stridor. The mother should be instructed to bring the child to the emergency department if the child develops stridor at rest, cyanosis, severe agitation or fatigue, or moderate to severe retractions or is unable to take oral fluids.

The client who has experienced a myocardial infarction (MI) is recovering from cardiogenic shock. The nurse knows that which observation of the client's clinical condition is most favorable? 1. Urine output of 40 mL/hr 2. Heart rate of 110 beats/minute 3. Frequent premature ventricular contractions 4. Central venous pressure (CVP) of 15 mm Hg

Urine output of 40 mL/hr Urine output of greater than 30 mL/hr indicates adequate perfusion to the kidneys, so the other organs are most likely equally perfused. Classic cardiovascular signs of cardiogenic shock include low blood pressure and tachycardia. Dysrhythmias commonly occur as a result of decreased oxygenation to the myocardium and are not a favorable sign. The CVP rises as the effects of the backward blood flow caused by the left ventricular failure became apparent.

To perform defibrillation, the defibrillator pads should be placed in which areas of the client's chest? 1. Behind the right and left shoulders in the scapular area 2. 1 inch (2.5 cm) below the sternum and 4 inches (10 cm) to the left of the sternum 3. 1 inch (2.5 cm) below the umbilicus and 2 inches (5 cm) to the right of the left nipple 4. To the right of the sternum just below the clavicle and to the left side, just below and to the left of the pectoral muscle

To the right of the sternum just below the clavicle and to the left side, just below and to the left of the pectoral muscle The anterior-apex placement works well for defibrillation and cardioversion, as well as for monitoring an electrocardiogram. In this placement, the anterior pad is placed on the right, below the clavicle. The other is applied to the left side of the client, just below and to the left of the pectoral muscle.

The nurse is caring for a client who sustained a burn injury to the anterior arms and anterior chest area from a fire. Which assessment finding would indicate that the client sustained a respiratory injury as a result of the burn? 1. Fear and anxiety 2. Complaints of pain 3. Clear breath sounds 4. Use of accessory muscles for breathing

Use of accessory muscles for breathing Clinical indicators of respiratory injury in a burn-injured client include the presence of facial burns, the presence of soot around the mouth or nose, and singed nasal hairs. Signs of respiratory difficulty include changes in respiratory rate and the use of accessory muscles for breathing. Although anxiety may be a sign of hypoxemia, anxiety along with bradycardia, dysrhythmias, and lethargy would be more likely to indicate a concern related to a respiratory injury. Abnormal breath sounds and abnormal arterial blood gas values also would be noted.

A client being admitted to the coronary care unit from the emergency department has a stat prescription to receive a dose of intravenous procainamide followed by a continuous infusion. Based on this prescription, the nurse should assess for which condition? 1. Dyspnea 2. Bradycardia 3. Hypertension 4. Ventricular ectopy

Ventricular ectopy Procainamide is an antidysrhythmic medication used to treat ventricular dysrhythmias unresponsive to lidocaine. The other options are not indications for giving this medication.

The nurse is making a note in the care plan for a client who has a multilumen central venous catheter. The nurse should write to change the injection caps on the lumens at which times? 1. Once a week 2. At the change of each shift 3. After administration of each medication 4. Whenever blood is drawn from the lumen

Whenever blood is drawn from the lumen Changing the injection caps is done to reduce systemic infection, which can be caused by contaminated caps. The injection cap should be discarded and a new one applied once it has been removed from the actual lumen. It is removed whenever blood work is drawn from the lumen. Once a week is too infrequent. At the change of shift is too frequent. It is not necessary to change the injection caps after administration of each medication because it is unnecessary to remove the cap to administer medication. In addition, agencies have policies that guide the frequency of routine injection cap changes (often every 48 hours). Agency policies should always be followed.

A pregnant client being admitted to the labor room tells the nurse that she felt a large gush of fluid before arriving at the hospital. The nurse performs an assessment on the client and notes that the fetal heart rate is 90 beats/minute and that the umbilical cord is protruding from the vagina. What is the appropriate nursing action 1. Place the woman in a high Fowler's position. 2. Palpate and evaluate contractions while administering a tocolytic. 3. Wrap the cord loosely in a sterile towel saturated with warm, sterile normal saline. 4. Start an intravenous (IV) line with fluids to be administered at a keep-vein-open (KVO) rate only.

Wrap the cord loosely in a sterile towel saturated with warm, sterile normal saline. When an umbilical cord is protruding, nursing actions are immediately directed at reducing cord compression and facilitating delivery of the fetus. The cord is wrapped loosely in a sterile towel saturated with warm normal saline to prevent it from drying out and becoming compressed. The client is placed in an extreme Trendelenburg's or modified Sims' position or knee-chest position to reduce compression. A tocolytic is used for inadequate uterine relaxation. IV solutions are administered at a rate greater than a KVO rate.

A client in ventricular fibrillation is about to be defibrillated. To convert this rhythm effectively, the monophasic defibrillator machine should be set at which energy level (in joules, J) for the first delivery? 1. 50 J 2. 120 J 3. 200 J 4. 360 J

360 J The energy level used for all defibrillation attempts with a monophasic defibrillator is 360 joules.

A client being hemodialyzed suddenly becomes short of breath and complains of chest pain. The client is tachycardic, pale, and anxious and the nurse suspects air embolism. What are the priority nursing actions? Select all that apply. 1. Administer oxygen to the client. 2. Continue dialysis at a slower rate after checking the lines for air. 3. Notify the health care provider (HCP) and Rapid Response Team. 4. Stop dialysis, and turn the client on the left side with head lower than feet. 5. Bolus the client with 500 mL of normal saline to break up the air embolus.

1. Administer oxygen to the client. 3. Notify the health care provider (HCP) and Rapid Response Team. 4. Stop dialysis, and turn the client on the left side with head lower than feet. If the client experiences air embolus during hemodialysis, the nurse should terminate dialysis immediately, position the client so the air embolus is in the right side of the heart, notify the HCP and Rapid Response Team, and administer oxygen as needed. Slowing the dialysis treatment or giving an intravenous bolus will not correct the air embolism or prevent complications.

A client in the postpartum unit complains of sudden, sharp chest pain. The client is tachycardic, and the respiratory rate is increased. The health care provider diagnoses a pulmonary embolism. Which actions should the nurse plan to take? Select all that apply. 1. Administer oxygen. 2. Assess the blood pressure. 3. Start an intravenous (IV) line. 4. Prepare to administer warfarin sodium. 5. Prepare to administer morphine sulfate. 6. Place the client on bed rest in a supine position.

1. Administer oxygen. 2. Assess the blood pressure. 3. Start an intravenous (IV) line. 5. Prepare to administer morphine sulfate. If pulmonary embolism is suspected, oxygen is administered to decrease hypoxia. The client also is kept on bed rest, with the head of the bed elevated to reduce dyspnea. Morphine sulfate may be prescribed for the client to reduce pain and apprehension. An IV line also will be required, and vital signs must be monitored. Heparin therapy (not warfarin sodium) is administered.

The nurse is assigned to a client receiving total parenteral nutrition (TPN) who had a blood glucose measurement done at 06:00. The nurse documents on the client's clinical worksheet for the day that the blood glucose level should be checked next at which time? 1. 08:00 2. 12:00 3. 16:00 4. 18:00

12:00 The client's blood glucose level should be monitored every 4 to 6 hours during TPN therapy. Depending on agency policy, this may be done every 8 hours instead. Monitoring the blood glucose level every 2 hours is unnecessary. Monitoring every 10 or 12 hours is insufficient.

A client involved in a motor vehicle crash presents to the emergency department with severe internal bleeding. The client is severely hypotensive and unresponsive. The nurse anticipates that which intravenous (IV) solution will most likely be prescribed for this client? 1. 5% dextrose in lactated Ringer's solution 2. 0.33% sodium chloride (⅓ normal saline) 3. 0.45% sodium chloride (½ normal saline) 4. 0.225% sodium chloride (¼ normal saline)

5% dextrose in lactated Ringer's solution For this client, the goal of therapy is to expand intravascular volume as quickly as possible. In this situation, the client will likely experience a decrease in intravascular volume from blood loss, resulting in decreased blood pressure. Therefore, a solution that increases intravascular volume, replaces immediate blood loss volume, and increases blood pressure is needed. The 5% dextrose in lactated Ringer's (hypertonic) solution would increase intravascular volume and immediately replace lost fluid volume until a transfusion could be administered, resulting in an increase in the client's blood pressure. The solutions in the remaining options would not be given to this client because they are hypotonic solutions and, instead of increasing intravascular space, the solutions would move into the cells via osmosis.

The nurse is told by a health care provider that a client in hypovolemic shock will require plasma expansion. The nurse should prepare which supplies for transfusion? 1. Bag of platelets with filtered tubing 2. Bottle of albumin with vented tubing 3. Cryoprecipitate bag with vented tubing 4. Infusion pump and bag of packed red blood cells

Bottle of albumin with vented tubing Albumin may be used as a plasma expander. Albumin is supplied in a bottle, and vented tubing is required for transfusion. Platelets are used when the client's platelet count is low. Cryoprecipitate is useful in treating bleeding from hemophilia or disseminated intravascular coagulopathy because it is rich in clotting factors. Cryoprecipitate is usually supplied in bags, so vented tubing is not required. Packed red blood cells replace erythrocytes and are not a plasma expander.

The nurse overhears a health care provider (HCP) stating that a client diagnosed with disseminated intravascular coagulation (DIC) requires a transfusion. Which blood product should the nurse anticipate that the HCP will write a prescription for? 1. Albumin 2. Platelets 3. Cryoprecipitate 4. Packed red blood cells

Cryoprecipitate Cryoprecipitate is useful in treating bleeding from hemophilia or DIC because it is rich in clotting factors. Albumin may be used as a plasma expander in hypovolemia with or without shock. Platelets are used when the client's platelet count is low. Packed red blood cells replace erythrocytes, not fibrinogen.

A client with heart failure and hypotension has been started on intravenous medication therapy with inamrinone. The nurse determines which finding, if noted in the client, is an adverse effect of the medication? 1. Decreased weight 2. Decreased blood pressure 3. Absence of lung crackles 4. Reduced peripheral edema

Decreased blood pressure Inamrinone is an inotropic agent used to relieve the manifestations of heart failure. Therapeutic effects include a decrease in weight (fluid), lung crackles, dyspnea, and edema. Blood pressure should remain stable or increase (if the client is hypotensive). Hypotension is an adverse effect of the medication.

Which nursing action is essential prior to initiating a new prescription for 500 mL of fat emulsion (lipids) to infuse at 50 mL/hour? 1. Ensure that the client does not have diabetes. 2. Determine whether the client has an allergy to eggs. 3. Add regular insulin to the fat emulsion, using aseptic technique. 4. Contact the health care provider (HCP) to have a central line inserted for fat emulsion infusion.

Determine whether the client has an allergy to eggs. The client beginning infusions of fat emulsions must be first assessed for known allergies to eggs to prevent anaphylaxis. Egg yolk is a component of the solution and provides emulsification. The remaining options are unnecessary and are not related to the administration of fat emulsion.

The emergency department nurse is monitoring a client who received treatment for a severe asthma attack. The nurse determines that the client's respiratory status has worsened if which is noted on assessment? 1. Diminished breath sounds 2. Wheezing during inhalation 3. Wheezing during exhalation 4. Wheezing throughout the lung fields

Diminished breath sounds Diminished breath sounds may be an indication of severe obstruction and possibly respiratory failure. Wheezing is not a reliable manifestation to determine the severity of an asthma attack. For wheezing to occur, the client must be able to move sufficient air to produce breath sounds. Wheezing usually occurs first on exhalation. As the asthma attack progresses, the client may wheeze during both inspiration and expiration.

The nurse has a prescription to administer acetylcysteine to a client admitted to the emergency department with acetaminophen overdose. Before giving this medication, what is the nurse's best action? 1. Administer the full-strength solution. 2. Empty the stomach by emesis or lavage. 3. Check that the antidote is readily available. 4. Ensure that the client knows how to use a nebulizer.

Empty the stomach by emesis or lavage. Acetylcysteine can be given orally or by nasogastric tube to treat acetaminophen overdose, or it may be given by inhalation for use as a mucolytic. Before giving the medication as an antidote to acetaminophen, the nurse ensures that the client's stomach is empty through emesis or gastric lavage. The solution is diluted in cola, water, or juice to make it more palatable. It is then administered orally or by nasogastric tube. Acetylcysteine is the antidote to acetaminophen.

A client who suffered carbon monoxide poisoning from working on an automobile in a closed garage has a carbon monoxide level of 15%. The nurse should anticipate observing which sign or symptom? 1. Coma 2. Flushing 3. Dizziness 4. Tachycardia

Flushing The signs and symptoms worsen as the carbon monoxide level rises in the bloodstream. Impaired visual acuity occurs at 5% to 10%, whereas flushing and headache are seen at 11% to 20%. Nausea and impaired dexterity appear at levels of 21% to 30%, and a 31% to 40% level is accompanied by vomiting, dizziness, and syncope. Levels of 41% to 50% cause tachypnea and tachycardia, and those higher than 50% result in coma and death.

The nurse is reviewing the laboratory test results for a client admitted to the burn unit 3 hours after an explosion that occurred at a worksite. The client has a severe burn injury that covers 35% of the total body surface area (TBSA). The nurse is most likely to note which finding on the laboratory report? 1. Hematocrit 60% (0.60) 2. Serum albumin 4.8 g/dL (48 g/L) 3. Serum sodium 144 mEq/L (144 mmol/L) 4. White blood cell (WBC) count 9000 mm3 (9 × 109/L)

Hematocrit 60% (0.60) Extensive burns greater than 25% of the TBSA result in generalized body edema in both burned and unburned tissues and a decrease in circulating intravascular blood volume. Hematocrit levels are elevated in the first 24 hours after injury as a result of hemoconcentration from the loss of intravascular fluid. The normal hematocrit level ranges from 37% to 47% (0.37 to 0.47) (female) and 42% to 52% (0.42 to 0.52) (male). The remaining options identify normal laboratory values.

The client has developed atrial fibrillation, with a ventricular rate of 150 beats/minute. The nurse should assess the client for which associated signs and/or symptoms? 1. Flat neck veins 2. Nausea and vomiting 3. Hypotension and dizziness 4. Hypertension and headache

Hypotension and dizziness The client with uncontrolled atrial fibrillation with a ventricular rate more than 100 beats/minute is at risk for low cardiac output because of loss of atrial kick. The nurse assesses the client for palpitations, chest pain or discomfort, hypotension, pulse deficit, fatigue, weakness, dizziness, syncope, shortness of breath, and distended neck veins.

A client who is receiving intravenous (IV) fluid therapy complains of burning and a feeling of tightness at the IV insertion site. On assessment, the nurse detects coolness and swelling at the site and notes that the IV rate has slowed. The nurse determines that which complication has occurred? 1. Infection 2. Phlebitis 3. Infiltration 4. Thrombosis

Infiltration An infiltrated IV line is one that has dislodged from the vein and is lying in subcutaneous tissue. Pallor, coolness, and swelling at the IV site result when IV fluid is deposited in the subcutaneous tissue. When the pressure in the tissues exceeds the pressure in the tubing, the flow of IV solution will slow down or stop. The corrective action is to remove the catheter and start a new IV line at another site. The conditions identified in the remaining options are likely to be accompanied by warmth at the site, not coolness.

A 5-year-old boy is brought by his mother to the emergency department after ingesting a bottle of acetylsalicylic acid. Which procedure should be initially instituted with this child? 1. Administer ipecac by mouth and monitor emesis. 2. Institute a gastric lavage and administer activated charcoal. 3. Administer a chelating agent such as edetate calcium disodium. 4. Institute a gastric lavage and administer the antidote acetylcysteine.

Institute a gastric lavage and administer activated charcoal. A gastric lavage must be performed after ingestion of acetylsalicylic acid, and activated charcoal is administered to prevent further absorption of the substance. N-acetylcysteine is the antidote for acetaminophen. Administering ipecac or edetate calcium disodium is not a treatment measure for acetylsalicylic acid poisoning. Edetate calcium disodium may be prescribed for the treatment of lead poisoning. Ipecac causes vomiting, and this substance is used only in specific poisoning conditions; in this situation, vomiting can cause irritation of the esophagus.

The nurse is assessing a client in the fourth stage of labor and notes that the fundus is firm, but that bleeding is excessive. Which should be the initial nursing action? 1. Record the findings. 2. Massage the fundus. 3. Notify the health care provider (HCP). 4. Place the client in Trendelenburg's position.

Notify the health care provider (HCP). If bleeding is excessive, the cause may be laceration of the cervix or birth canal. Massaging the fundus if it is firm would not assist in controlling the bleeding. Trendelenburg's position should be avoided because it may interfere with cardiac and respiratory function. Although the nurse would record the findings, the initial nursing action would be to notify the HCP.

A client begins to experience drainage of small amounts of bright red blood from the tracheostomy tube 24 hours after a supraglottic laryngectomy. Which is the best nursing action? 1. Notify the health care provider (HCP). 2. Increase the frequency of suctioning. 3. Add moisture to the oxygen delivery system. 4. Document the character and amount of drainage.

Notify the health care provider (HCP). Immediately after laryngectomy, a small amount of bleeding occurs from the tracheostomy that resolves within the first few hours. Otherwise, bleeding that is bright red may be a sign of impending rupture of a vessel. The bleeding in this instance represents a potential threat to life, and the HCP is notified to further evaluate the client and suture or repair the source of the bleeding. The other options do not address the urgency of the problem. Failure to notify the HCP places the client at risk.

The nurse assigned to the pediatric unit finds an infant unresponsive and without respirations or a pulse. What is the nurse's next action after calling for help? 1. Check for carotid pulse. 2. Call anesthesia for intubation. 3. Begin rescue breathing with head tilt-chin lift. 4. Perform compressions at 100 to 120 times per minute.

Perform compressions at 100 to 120 times per minute. After pressing the emergency response button in the room, the nurse should begin cardiopulmonary resuscitation (CPR) on the infant, starting with chest compressions. The rate of chest compressions is 100 to 120 times per minute. The brachial pulse is assessed on infants; the carotid pulse is difficult to palpate due to their short, thick necks. When a cardiopulmonary arrest alert is called, an experienced staff member with intubation skills is usually included on the response team. Compressions are started before rescue breathing.

The nurse is watching the cardiac monitor and notices that the rhythm suddenly changes. There are no P waves, the QRS complexes are wide, and the ventricular rate is regular but more than 140 beats/minute. The nurse determines that the client is experiencing which dysrhythmia? 1. Sinus tachycardia 2. Ventricular fibrillation 3. Ventricular tachycardia 4. Premature ventricular contractions

Ventricular tachycardia Ventricular tachycardia is characterized by the absence of P waves, wide QRS complexes (longer than 0.12 seconds), and typically a rate between 140 and 180 impulses/minute. The rhythm is regular.

The nurse has obtained a unit of blood from the blood bank and has checked the blood bag properly with another nurse. Just before beginning the transfusion, the nurse should assess which priority item? 1. Vital signs 2. Skin color 3. Urine output 4. Latest hematocrit level

Vital signs A change in vital signs during the transfusion from baseline may indicate that a transfusion reaction is occurring. This is why the nurse assesses vital signs before the procedure and again after the first 15 minutes and thereafter per agency policy. The other options do not identify assessments that are a priority just before beginning a transfusion.

A client with cancer is placed on permanent total parenteral nutrition (TPN). The nurse considers psychosocial support when planning care for this client when the client makes which correct statement? 1. "Death is imminent for me because of this." 2. "This treatment requires disfiguring surgery." 3. "Nausea and vomiting occur regularly and will prevent social activity." 4. "I'll need to adjust to the idea of living without eating by the usual route."

"I'll need to adjust to the idea of living without eating by the usual route." Permanent TPN is indicated for clients who can no longer absorb nutrients via the enteral route. These clients may no longer be able to take nutrition orally and will need to adjust to the idea of living without eating by the usual route. The remaining options are incorrect statements. There are no data in the question that indicate that death is imminent. Port implantation does not require disfiguring surgery. Nausea and vomiting are not associated with administering TPN.

The mother of a 3-year-old boy calls the emergency department and states that she found an empty bottle of acetaminophen on the floor. She states that she thinks her child ingested all of the medication. What is the priority question for the nurse to ask the mother? 1. "Is your child breathing okay?" 2. "Is your child alert and oriented?" 3. "Where is your child at this moment?" 4. "Do you know how many tablets were in the bottle?"

"Is your child breathing okay?" Airway is always the highest assessment data to obtain during a poison control call. Once this information is obtained, the child's neurological status can be determined in terms of his orientation and other information, such as that referred to in the remaining options.

A client is scheduled for placement of a peripherally inserted central catheter (PICC). The nurse has explained the advantages of this catheter to the client. Which statement made by the client indicates a need for further explanation? 1. "It is reasonable in cost." 2. "There is less pain and discomfort." 3. "This type of catheter is very reliable." 4. "It is specifically designed for short-term use."

"It is specifically designed for short-term use." A PICC is a central line that is inserted in the upper arm. It is a flexible catheter that terminates in the superior vena cava of the heart. Placement is verified by x-ray or other methods prior to use. PICCs are intended to be used for clients needing long-term intravenous catheter placement. They are reasonable in cost because they do not need routine replacement, as do traditional peripheral intravenous catheters. The catheter also is reliable. The catheter is less likely to result in infiltration and can be used for administration of a number of different types of medications without extravasation.

The nurse is providing care for a client who sustained burns over 30% of the body from a fire. On assessment, the nurse notes that the client is edematous in both burned and unburned body areas. The client's wife asks why her husband "looks so swollen." What is the nurse's best response? 1. "Constricted blood vessels have caused a loss of protein in the blood." 2 "Leaking blood vessels have led to increased protein amounts in the blood." 3 "Leaking blood vessels have led to decreased protein amounts in the blood." 4. "Constricted blood vessels have led to increased protein amounts in the blood."

"Leaking blood vessels have led to decreased protein amounts in the blood." In extensive burn injuries (greater than 25% of total body surface area), the edema occurs in both burned and unburned areas as a result of the increase in capillary permeability and hypoproteinemia. Edema also may be caused by the volume and oncotic pressure effects of the large fluid resuscitation volumes required.

A client has a prescription to receive a unit of packed red blood cells. The nurse should obtain which intravenous (IV) solution from the IV storage area to hang with the blood product at the client's bedside? 1. Lactated Ringer's 2. 0.9% sodium chloride 3. 5% dextrose in 0.9% sodium chloride 4. 5% dextrose in 0.45% sodium chloride

0.9% sodium chloride Sodium chloride 0.9% (normal saline) is a standard isotonic solution used to precede and follow infusion of blood products. Dextrose is not used because it could result in clumping and subsequent hemolysis of red blood cells (RBCs). Lactated Ringer's is not the solution of choice with this procedure.

A client develops an anaphylactic reaction after receiving morphine. The nurse should plan to institute which actions? Select all that apply. 1. Administer oxygen. 2. Quickly assess the client's respiratory status. 3. Document the event, interventions, and client's response. 4. Leave the client briefly to contact a health care provider (HCP). 5. Keep the client supine regardless of the blood pressure readings. 6. Start an intravenous (IV) infusion of D5W and administer a 500-mL bolus.

1. Administer oxygen. 2. Quickly assess the client's respiratory status. 3. Document the event, interventions, and client's response. An anaphylactic reaction requires immediate action, starting with quickly assessing the client's respiratory status. Although the HCP and the Rapid Response Team must be notified immediately, the nurse must stay with the client. Oxygen is administered and an IV of normal saline is started and infused per HCP prescription. Documentation of the event, actions taken, and client outcomes needs to be done. The head of the bed should be elevated if the client's blood pressure is normal.

A new nursing graduate is caring for a client who is attached to a cardiac monitor. While assisting the client with bathing, the nurse observes the sudden development of ventricular tachycardia (VT), but the client remains alert and oriented and has a pulse. Which interventions would the nurse take? Select all that apply. 1. Administer oxygen. 2. Defibrillate the client. 3. Obtain an electrocardiogram (ECG). 4. Contact the health care provider (HCP). 5. Assess circulation, airway, and breathing. 6. Initiate cardiopulmonary resuscitation (CPR).

1. Administer oxygen. 3. Obtain an electrocardiogram (ECG). 4. Contact the health care provider (HCP). 5. Assess circulation, airway, and breathing. With VT in a stable client, the nurse assesses circulation, airway, and breathing; administers oxygen; and confirms the rhythm via a 12-lead ECG. The HCP is contacted, and antidysrhythmics may be prescribed. With pulseless VT, the HCP or a specially trained nurse must immediately defibrillate the client or initiate CPR followed by defibrillation as soon as possible.

A client requiring surgery is anxious about the possible need for a blood transfusion during or after the procedure. The nurse suggests to the client to take which actions to reduce the risk of possible transfusion complications? Select all that apply. 1. Ask a family member to donate blood ahead of time. 2. Give an autologous blood donation before the surgery. 3. Take iron supplements before surgery to boost hemoglobin levels. 4. Request that any donated blood be screened twice by the blood bank. 5. Take adequate amounts of vitamin C several days prior to the surgery date.

1. Ask a family member to donate blood ahead of time. 2. Give an autologous blood donation before the surgery. A donation of the client's own blood before a scheduled procedure is autologous. Donating autologous blood to be reinfused as needed during or after surgery reduces the risk of disease transmission and potential transfusion complications. The next most effective way is to ask a family member to donate blood before surgery. Blood banks do not provide extra screening on request. Preoperative iron supplements are helpful for iron deficiency anemia but are not helpful in replacing blood lost during the surgery. Vitamin C enhances iron absorption, but also is not helpful in replacing blood lost during surgery

The nurse who is about to begin a blood transfusion knows that blood cells start to deteriorate after a certain period of time. The nurse takes which actions in order to prevent a complication of the blood transfusion as it relates to deterioration of blood cells? Select all that apply. 1. Checks the expiration date 2. Inspects for the presence of clots 3. Checks the blood group and type 4. Checks the blood identification number 5. Hangs the blood within the specified time frame per agency policy

1. Checks the expiration date 5. Hangs the blood within the specified time frame per agency policy The nurse notes the expiration date on the unit of blood to ensure that the blood is fresh. Blood cells begin to deteriorate over time, so safe storage usually is limited to 35 days. Careful notation of the expiration date by the nurse is an essential part of the verification process before hanging a unit of blood. The nurse also needs to hang the blood within the specified time frame after receiving it from the blood bank per agency policy to ensure that the blood being transfused is fresh. The blood bank keeps the blood regulated at a specific temperature, and therefore it must be infused within a specified time frame once received on the unit. The nurse also notes the blood identification (unit) number, blood group and type, and client's name, but this is not specifically related to the degradation of blood cells. The nurse also inspects the unit of blood for leaks, abnormal color, clots, and bubbles and returns the unit to the blood bank if clots are noted. Again, this is not related to the degradation of blood cells over time.

A client is brought to the emergency department complaining of substernal chest pain. To distinguish between angina and myocardial infarction, the nurse assesses for which characteristics of angina? Select all that apply 1. Chest pain that resolves with rest 2. Chest pain requiring an opioid for relief 3. Chest pain that is relieved by nitroglycerin 4. Chest pain that lasts longer than 30 minutes 5. Chest pain that is usually precipitated by exertion

1. Chest pain that resolves with rest 3. Chest pain that is relieved by nitroglycerin 5. Chest pain that is usually precipitated by exertion Angina is chest pain caused by a temporary imbalance between the coronary arteries' ability to supply oxygen and the cardiac muscle's demand for oxygen. Myocardial infarction refers to injury and necrosis of myocardial tissue that occurs when the tissue is abruptly and severely deprived of oxygen. When a client complains of chest pain, it is critical that treatment is immediately initiated and that the nurse assesses for characteristics of angina versus those associated with myocardial infarction. Angina is characterized by substernal chest pain radiating to the left arm. The pain is usually precipitated by exertion or stress, is relieved by nitroglycerin or rest, and lasts less than 15 minutes. Characteristics of myocardial infarction include substernal chest pain that radiates to the left arm; pain in the jaw, abdomen, back, or shoulder can also occur. The substernal chest pain occurs without cause, usually in the morning; is relieved only by opioids, and lasts 30 minutes or longer.

The nurse is assisting in monitoring a client who is receiving a transfusion of packed red blood cells (PRBCs). Before leaving the room, the nurse tells the client to immediately report which symptoms of a transfusion reaction? Select all that apply. 1. Chills 2. Fatigue 3. Sleepiness 4. Chest pain 5. Lower back pain 6. Difficulty breathing

1. Chills 4. Chest pain 5. Lower back pain 6. Difficulty breathing The nurse should instruct the client to immediately report signs of a transfusion reaction, which can include chest pain, lower back pain, chills, itching, rash, or difficulty breathing. These signs of transfusion reaction would require the nurse to stop the transfusion. Fatigue and sleepiness are unrelated to transfusion reaction.

The occupational health nurse is called to care for an employee who experienced a traumatic amputation of a finger. Which actions should the nurse take to provide emergency care and prepare the client for transport to the hospital? Select all that apply. 1. Elevate the extremity above heart level. 2. Assess the employee for airway or breathing problems. 3. Remove the layered gauze every 10 minutes to check the bleeding. 4. Wrap the severed finger in moistened gauze, and place it in a bag of ice water. 5. Examine the amputation site and apply direct pressure to the site using layers of gauze.

1. Elevate the extremity above heart level. 2. Assess the employee for airway or breathing problems. 5. Examine the amputation site and apply direct pressure to the site using layers of gauze. The nurse would assess the victim for airway or breathing problems immediately. Then the nurse would examine the amputation site and apply direct pressure using layers of dry gauze or another type of cloth. Sterile gloves and sterile gauze should always be used if available. If sterile materials are not available, clean materials should be used if possible. The gauze that is applied is a pressure dressing and is not removed because of the risk of dislodgment of a clot that may be forming; the pressure dressing will be removed at the hospital. The extremity is elevated above the victim's heart level to decrease the bleeding. The severed finger should be wrapped in dry, sterile gauze (if available) or a clean cloth. It is placed in a watertight, sealed plastic bag. Then the watertight, sealed plastic bag is placed in a bag of ice water. The severed finger is never placed directly on ice; contact between the finger and water is avoided because of the risk of tissue damage. The severed part is transported to the hospital with the victim for possible replanting. Additionally, emergency medical services is called to transport the victim to the hospital.

An emergency department nurse is caring for a child with suspected acute epiglottitis. Which nursing interventions apply in the care of this child? Select all that apply. 1. Ensure a patent airway. 2. Obtain a throat culture. 3. Maintain the child in a supine position. 4. Obtain a pediatric-size tracheostomy tray. 5. Prepare the child for a chest radiographic study. 6. Place the child on an oxygen saturation monitor.

1. Ensure a patent airway. 4. Obtain a pediatric-size tracheostomy tray. 5. Prepare the child for a chest radiographic study. 6. Place the child on an oxygen saturation monitor. Acute epiglottitis is a serious obstructive inflammatory process that requires immediate intervention. The nurse immediately ensures a patent airway. To reduce respiratory distress, the child should sit upright. Examining the throat with a tongue depressor or attempting to obtain a throat culture is contraindicated because it could precipitate further obstruction. A complete blood count is obtained, and the child is placed on an oxygen saturation monitor. Lateral neck and chest radiographic films are obtained to determine the degree of obstruction, if present. A pediatric-size tracheostomy tray should be readily available, and intubation may be necessary if respiratory distress is severe.

A client with type 1 diabetes mellitus in the emergency department is diagnosed with diabetic ketoacidosis (DKA). Which interventions should the nurse anticipate being prescribed initially? Select all that apply. 1. Monitoring urine for ketones 2. Intravenous potassium replacement 3. Administration of intravenous insulin 4. A bolus of 5% dextrose intravenously 5. Administration of a liter of 0.9% NaCl intravenously.

1. Monitoring urine for ketones 2. Intravenous potassium replacement 3. Administration of intravenous insulin 5. Administration of a liter of 0.9% NaCl intravenously. DKA is caused by a profound deficiency of insulin and is characterized by hyperglycemia (blood glucose level greater than or equal to 250 mg/dL [13.9 mmol/L]), ketosis (ketones in urine or serum), metabolic acidosis, dehydration, and hypokalemia. Therefore, the correct options are 1, 2, 3, and 5. Option 4 is incorrect, as clients with DKA have a profound deficiency of insulin and an elevated blood glucose; therefore, dextrose would not be administered as an initial intervention.

The nurse is providing care for a client with new onset of a dysrhythmia. The nurse anticipates which prescriptions from the health care provider? Select all that apply. Refer to Figure. 1. Oxygen therapy 2. An echocardiogram 3. An intravenous dose of metoprolol 4. One dose of atropine to promote slowing of the rate 5. A bolus of intravenous heparin followed by a continuous infusion

1. Oxygen therapy 2. An echocardiogram 3. An intravenous dose of metoprolol 5. A bolus of intravenous heparin followed by a continuous infusion In atrial fibrillation the ventricles often beat with a rapid rate in response to the numerous atrial impulses. Heart dilation and blood pooling in the atria can lead to thrombus formation, which increases the risk for stroke or other embolic events; therefore, heparin is indicated. The rapid and irregular ventricular rate decreases ventricular filling and reduces cardiac output, further impairing the heart's perfusion ability. Therefore, oxygen and metoprolol (to slow the ventricular response) are appropriate. An echocardiogram will help to assess heart valve function because mitral valve disease can lead to atrial fibrillation. Atropine will increase the heart rate further.

A client complains of pain at the site of an intravenous (IV) infusion. The nurse assesses the site and determines that phlebitis has developed. The nurse should take which actions in the care of this client? Select all that apply. 1. Remove the IV catheter at that site. 2. Apply warm, moist packs to the site. 3. Notify the health care provider (HCP). 4. Start a new IV line in a proximal portion of the same vein. 5. Document the occurrence, actions taken, and the client's response.

1. Remove the IV catheter at that site. 2. Apply warm, moist packs to the site. 3. Notify the health care provider (HCP). 5. Document the occurrence, actions taken, and the client's response. Phlebitis is an inflammation of the vein that can occur from mechanical or chemical (medication) trauma or from a local infection and can cause the development of a clot (thrombophlebitis). The nurse should remove the IV at the phlebitic site and apply warm, moist compresses to the area to speed resolution of the inflammation. Because phlebitis has occurred, the nurse also notifies the HCP about the IV complication. The nurse should restart the IV in a vein other than the one that has developed phlebitis. Finally, the nurse documents the occurrence, actions taken, and the client's response.

The nurse is performing discharge teaching for a client with a peripherally inserted central catheter (PICC). Which instructions should the nurse include? Select all that apply. 1. Wear a MedicAlert tag or bracelet. 2. Report redness or swelling at the catheter insertion site. 3. Have a repair kit available in the home for use if needed. 4. Keep activity level to a minimum while this catheter is in place. 5. Cover the PICC dressing with plastic when in the shower or bath

1. Wear a MedicAlert tag or bracelet. 2. Report redness or swelling at the catheter insertion site. 3. Have a repair kit available in the home for use if needed. 5. Cover the PICC dressing with plastic when in the shower or bath. The client should be taught that there are only minor activity restrictions with this catheter. The client should protect the site during bathing and should carry MedicAlert identification. The client should have a repair kit in the home for PRN use, because it is a long-term catheter. Redness or swelling at the catheter insertion site needs to be reported because this could indicate a sign of infection

A client arrives at the emergency department following a burn injury that occurred in the basement at home, and an inhalation injury is suspected. What would the nurse anticipate to be prescribed for the client? 1. 100% oxygen via an aerosol mask 2. Oxygen via nasal cannula at 6 L/minute 3. Oxygen via nasal cannula at 15 L/minute 4. 100% oxygen via a tight-fitting, nonrebreather face mask

100% oxygen via a tight-fitting, nonrebreather face mask If an inhalation injury is suspected, administration of 100% oxygen via a tight-fitting nonrebreather face mask is prescribed until carboxyhemoglobin levels fall (usually below 15%). In inhalation injuries, the oropharynx is inspected for evidence of erythema, blisters, or ulcerations. The need for endotracheal intubation also is assessed. Administration of oxygen by aerosol mask and cannula are incorrect and would not provide the necessary oxygen supply needed for adequate tissue perfusion for the client with a likely inhalation injury.

A client is diagnosed with atrial fibrillation, and the health care provider prescribes medication therapy. Prior to initiating medication therapy, which cardiac rhythm would the nurse expect to note on the cardiac monitor? Click on the image to indicate your answer.

2 In atrial fibrillation, multiple rapid impulses from many foci depolarize in the atria in a totally disorganized manner. The atria quiver, which can lead to thrombus formation. Usually no definitive P waves can be observed, only fibrillatory waves before each QRS. Medication therapy is often effective for treating atrial fibrillation. In normal sinus rhythm (option 1), atrial and ventricular rhythms are regular and the rates are 60 to 100 beats per minute. Ventricular tachycardia (option 3) occurs because of a repetitive firing of an irritable ventricular ectopic focus at a rate of 140 to 250 beats/minute or more; it can lead to cardiac arrest. In ventricular fibrillation (option 4), impulses from many irritable foci in the ventricles fire in a totally disorganized manner. It is characterized by a chaotic rapid rhythm in which the ventricles quiver and there is no cardiac output. The client lacks a pulse, blood pressure, respirations, and heart sounds. Ventricular fibrillation is fatal if not successfully terminated within 3 to 5 minutes

A client is brought to the emergency room with a snake bite to the arm. Which treatment interventions should the nurse anticipate? Select all that apply. 1. Apply ice to the site. 2. Deliver supplemental oxygen. 3. Apply a tourniquet just above the site. 4. Maintain the extremity at the level of the heart. 5. Infuse crystalloid fluids through 2 large-bore intravenous (IV) lines. 6. Immobilize the affected extremity in a position of function with a splint.

2. Deliver supplemental oxygen. 4. Maintain the extremity at the level of the heart. 5. Infuse crystalloid fluids through 2 large-bore intravenous (IV) lines. 6. Immobilize the affected extremity in a position of function with a splint. Interventions include giving supplemental oxygen, keeping the arm at the level of the heart, infusing crystalloid fluids through 2 large-bore IV lines, and immobilizing the arm in a position of function with a splint. Applying a tourniquet and placing ice on the area are contraindicated because they enhance the effect of the venom. Keep the person warm and provide calm reassurance. Also, apply continuous cardiac and blood pressure monitoring equipment to quickly detect clinical deterioration. Because venom can cause severe pain at the bite site, opioids are indicated. Provide tetanus prophylaxis and wound care as part of the collaborative plan of care.

The nurse in the labor room is performing an initial assessment on a newborn. The infant is exhibiting mild to moderate respiratory distress, audible bowel sounds in the chest, and a scaphoid abdomen. The infant is responding poorly to bag and mask ventilation. The nurse plans for which actions in the care of this infant? Select all that apply. 1. Start chest compressions. 2. Notify the health care provider (HCP). 3. Orally administer a sucrose solution. 4. Position the infant flat on his or her right side. 5. Prepare for endotracheal tube (ET) placement. 6. Insert an orogastric tube and connect it to low suction.

2. Notify the health care provider (HCP). 5. Prepare for endotracheal tube (ET) placement. 6. Insert an orogastric tube and connect it to low suction. Worsening respiratory distress, audible bowel sounds in the chest, and a flat or scaphoid abdomen are all signs and symptoms of a congenital diaphragmatic hernia. For this condition to be verified, appropriate x-rays must be prescribed by the HCP. If verified, operative intervention is necessary. As mechanical ventilation is administered, the bowel fills with air, thus worsening the respiratory status of the infant; therefore, gastrointestinal decompression is necessary as well as ventilation via an ET tube rather than a bag and mask. There is no evidence in the question that the infant requires chest compressions. The infant must be kept NPO (nothing by mouth) for operative intervention and to decrease gastric distention. Proper positioning for this condition is supine with the head elevated.

The nurse is making initial rounds on the nursing unit to assess the condition of assigned clients. Which assessment findings are consistent with infiltration? Select all that apply. 1. Pain and erythema 2. Pallor and coolness 3. Numbness and pain 4. Edema and blanched skin 5. Formation of a red streak and purulent drainage

2. Pallor and coolness 3. Numbness and pain 4. Edema and blanched skin An infiltrated intravenous (IV) line is one that has dislodged from the vein and is lying in subcutaneous tissue. Pallor, coolness, edema, pain, numbness, and blanched skin are the results of IV fluid being deposited in the subcutaneous tissue. When the pressure in the tissues exceeds the pressure in the tubing, the flow of the IV solution will stop, and if an electronic pump is being used, it will alarm. Erythema can be associated with infection, phlebitis, or thrombosis. Formation of a red streak and purulent drainage is associated with phlebitis and infection

The nurse is caring for a client recovering from a subtotal thyroidectomy. Which supplies should be readily accessible for the care of this client? Select all that apply. 1. Tourniquet 2. Suction supplies 3. Calcium gluconate 4. Prefilled syringe of 50% glucose 5. Tracheostomy tube insertion set

2. Suction supplies 3. Calcium gluconate 5. Tracheostomy tube insertion set Although not common, airway obstruction after thyroid surgery is an emergency situation. Therefore, oxygen, suction equipment, calcium gluconate (to treat tetany if it occurs), and a tracheostomy tube insertion set should be readily available in the client's room. These items will be needed to treat this emergency situation. Therefore, options 2, 3, and 5 are correct. There is no reason that a tourniquet needs to be readily available; 50% glucose is used to treat severe hypoglycemia.

Which client situation is most appropriate for the nurse to consult with the Rapid Response Team (RRT)? 1. A 56-year-old client, fourth hospital day after coronary artery bypass procedure, sore chest, pain with walking, temperature 97°F (36.1°C), heart rate 84 beats/min, respirations 22 breaths/min, blood pressure 122/78 mm Hg, bored with hospitalization 2. A 45-year-old client, 2 years after kidney transplant, second hospital day for treatment of pneumonia, no urine output for 6 hours, temperature 101.4°F (38.6°C), heart rate 98 beats/min, respirations 20 breaths/min, blood pressure 168/94 mm Hg 3. A 72-year-old client, 24 hours after removal of a chest tube that was used to drain pleural fluid (effusion), temperature 97.8°F (36.6°C), heart rate 92 beats/min, respirations 28 breaths/min, blood pressure 136/86 mm Hg, anxious about going home 4. An 86-year-old client, 48 hours after operative repair of fractured hip, alert, oriented, using patient-controlled analgesia pump, temperature 96.8°F (36°C), heart rate 60 beats/min, respirations 16 breaths/min, blood pressure 120/82 mm Hg, talking with daughter

A 45-year-old client, 2 years after kidney transplant, second hospital day for treatment of pneumonia, no urine output for 6 hours, temperature 101.4°F (38.6°C), heart rate 98 beats/min, respirations 20 breaths/min, blood pressure 168/94 mm Hg The role of an RRT is to provide internal consultative services to staff nurses to detect client problems early. Absence of urine output and temperature and blood pressure elevation describe a client who may be rejecting a transplanted kidney. The constellation of symptoms described indicates possible rejection. Internal consultation could validate that assessment. The remaining options indicate expected characteristics of the clients described and provide no indication of need for RRT consultation.

Vasopressin therapy is prescribed for a client with a diagnosis of bleeding esophageal varices. The nurse is preparing to administer the medication to the client. Which essential item is needed during the administration of this medication? 1. An airway 2. A suction setup 3. A cardiac monitor 4. A tracheotomy set

A cardiac monitor The major action of vasopressin is constriction of the splanchnic blood flow. Continuous electrocardiogram and blood pressure monitoring are essential because of the constrictive effects of the medication on the coronary arteries. The remaining options are not essential items required during the administration of this medication. However, these items may be needed if a complication arises.

A client hospitalized with a diagnosis of myocardial infarction calls for the unit nurse because the client is experiencing chest pain. The nurse administers a sublingual nitroglycerin tablet as prescribed. The client, who is receiving oxygen by nasal cannula, reports that her chest pain is unrelieved by the nitroglycerin. Which is the next nursing action for this client? 1. Call the client's family. 2. Increase the flow rate of oxygen. 3. Contact the health care provider (HCP). 4. Administer another nitroglycerin tablet.

Administer another nitroglycerin tablet. For the hospitalized client, nitroglycerin tablets are administered 1 tablet every 5 minutes, for a total of 3 tablets per episode of chest pain, as long as the client maintains a systolic blood pressure of 100 mm Hg or higher. Increasing the flow rate of oxygen may be prescribed by the HCP but would not be the next nursing action. If 3 nitroglycerin tablets do not relieve the client's chest pain, the HCP needs to be notified. It is premature to call the client's family.

A client in a postpartum unit complains of sudden sharp chest pain and dyspnea. The nurse notes that the client is tachycardic and the respiratory rate is elevated. The nurse suspects a pulmonary embolism. Which should be the initial nursing action? 1. Initiate an intravenous line. 2. Assess the client's blood pressure. 3. Prepare to administer morphine sulfate. 4. Administer oxygen, 8 to 10 L/minute, by face mask.

Administer oxygen, 8 to 10 L/minute, by face mask. If pulmonary embolism is suspected, oxygen should be administered, 8 to 10 L/minute, by face mask. Oxygen is used to decrease hypoxia. The client also is kept on bed rest with the head of the bed slightly elevated to reduce dyspnea. Morphine sulfate may be prescribed for the client, but this would not be the initial nursing action. An intravenous line also will be required, and vital signs need to be monitored, but these actions would follow the administration of oxygen.

Fetal distress is occurring with a laboring client. As the nurse prepares the client for a cesarean birth, what is the most important nursing action? 1. Slow the intravenous flow rate. 2. Continue the oxytocin drip if infusing. 3. Place the client in a high Fowler's position. 4. Administer oxygen, 8 to 10 L/minute, via face mask.

Administer oxygen, 8 to 10 L/minute, via face mask. Oxygen is administered, 8 to 10 L/minute, via face mask to optimize oxygenation of the circulating blood. Option 1 is incorrect because the intravenous infusion should be increased (per health care provider prescription) to increase the maternal blood volume. Option 2 is incorrect because oxytocin stimulation of the uterus is discontinued if fetal heart rate patterns change for any reason. Option 3 is incorrect because the client is placed in the lateral position with her legs raised to increase maternal blood volume and improve fetal perfusion.

The nurse enters the room of a client who began receiving a blood transfusion 45 minutes earlier to check on the client. The client is complaining of "itching all over" and has a generalized rash. The client's temperature has not changed from baseline and the lungs are clear to auscultation. Which complication of blood transfusion therapy should the nurse determine that this client is most likely experiencing? 1. Bacteremia 2. Fluid overload 3. Hypovolemic shock 4. Allergic transfusion reaction

Allergic transfusion reaction The client is most likely experiencing an allergic transfusion reaction based on the clinical manifestation of pruritus. Bacteremia usually manifests with fever. With fluid overload, the client has the presence of crackles in the lungs in addition to dyspnea. Other clinical manifestations of fluid overload include hypertension, a bounding pulse, distended jugular veins, restlessness, and confusion. Hypovolemic shock is not likely a transfusion reaction because intravascular fluid is being administered.

The nurse is performing cardiopulmonary resuscitation (CPR) on a client who has had a cardiac arrest. An automatic external defibrillator (AED) is available to treat the client. Which activity will allow the nurse to assess the client's cardiac rhythm? 1. Hold the defibrillator paddles firmly against the chest. 2. Apply adhesive patch electrodes to the chest and move away from the client. 3. Connect standard electrocardiographic electrodes to a transtelephonic monitoring device. 4. Apply standard electrocardiographic monitoring leads to the client, and observe the rhythm.

Apply adhesive patch electrodes to the chest and move away from the client. The nurse or rescuer puts two adhesive patch electrodes on the client's chest in the usual defibrillator positions. The nurse stops CPR and requests that anyone near the client move away and not touch the client. The defibrillator then analyzes the rhythm, which may take up to 30 seconds. The machine then indicates whether defibrillation is necessary.

Which step should occur first when using an automated external defibrillator (AED)? 1. Press the shock button if indicated. 2. Place the AED in the analyze mode. 3. Apply defibrillator pads on the client. 4. Check to see that no one is touching the client.

Apply defibrillator pads on the client. Knowing that the device needs to be in contact with the client in order to read the heart rhythm will lead you to select the option of placing the defibrillator pads on the client as the first step. The nurse next checks to see that no one is touching the client. The nurse then places the AED in the analyze mode. Then the shock button is pressed, if indicated.

The nurse witnesses an accident whereby a pedestrian is hit by an automobile. The nurse stops at the scene and assesses the victim. The nurse notes that the victim is responsive and has suffered trauma to the thorax resulting in a flail chest involving at least 3 ribs. What is the nurse's priority action for this victim? 1. Assist the victim to sit up. 2. Remove the victim's shirt. 3. Turn the victim onto the side opposite the flail chest. 4. Apply firm but gentle pressure with the hands to the flail segment.

Apply firm but gentle pressure with the hands to the flail segment. If flail chest is present, the nurse applies firm but gentle pressure to the flail segments of the ribs to stabilize the chest wall, which will ultimately help the victim's respiratory status. The nurse does not move an injured client for fear of worsening an undetected spinal injury. Removing the victim's shirt is of no value in this situation and could in fact result in chilling the victim, which is counterproductive. Injured clients should be kept warm until help arrives at the scene.

The client sustains a contusion of the eyeball following a traumatic injury with a blunt object. Which intervention should be initiated immediately? 1. Apply ice to the affected eye. 2. Irrigate the eye with cool water. 3. Notify the health care provider (HCP). 4. Accompany the client to the emergency department.

Apply ice to the affected eye. Treatment for a contusion begins at the time of injury. Ice is applied immediately. The client then should be seen by an HCP and receive a thorough eye examination to rule out the presence of other eye injuries.

The nurse is caring for a client diagnosed with cirrhosis of the liver with portal hypertension. The client vomited 500 mL bright red emesis and states that he is feeling lightheaded. In which priority order should the nurse perform these interventions? Arrange the actions in the order they should be performed. All options must be used.

Apply oxygen. Ensure that 2 large-bore intravenous lines are present with an isotonic solution infusing. Check the client's blood pressure. Ask the client if he is taking any nonsteroidal antiinflammatory medications. The client has an upper gastrointestinal (GI) bleed. Upper GI bleeding is an emergency because it can lead to hypovolemic shock. The first intervention of those listed should be to apply oxygen in an attempt to maximize the amount of oxygen being delivered by the decreased number of red blood cells due to the bleeding. The next action should be to ensure that 2 large-bore intravenous (IV) lines are present, and begin replacement of the intravascular fluid volume with an isotonic IV fluid. The nurse should then check the blood pressure. These are all actions to stabilize and assess the client's current condition. The last intervention is to ask the client about nonsteroidal antiinflammatory medications. Although it is important to identify the cause of the bleeding and obtain a complete history of events leading up to the bleeding episode, this needs to be deferred until emergency care is initiated.

A client who attempted suicide by hanging is brought to the emergency department by emergency medical services. Which is the immediate nursing action? 1. Take the client's vital signs, including pulse oximetry reading. 2. Assess the client's respiratory status and for the presence of neck injuries. 3. Perform a focused assessment, paying particular attention to the client's neurological status. 4. Call the mental health crisis team and notify them that a client who attempted suicide is being admitted.

Assess the client's respiratory status and for the presence of neck injuries. The immediate nursing action for a client who attempted suicide is to assess physiological status. Airway is always the priority. Therefore, assessing the client's respiratory status and for the presence of neck injuries is the immediate action that the nurse takes. Although the remaining options identify appropriate nursing actions, they are not the priority.

A client with angina pectoris is experiencing chest pain that radiates down the left arm. The nurse administers a sublingual nitroglycerin tablet to the client. The client's pain is unrelieved, and the nurse determines that the client needs another nitroglycerin tablet. Which vital sign is the most important for the nurse to check before administering the medication? 1. Temperature 2. Respirations 3. Blood pressure 4. Radial pulse rate

Blood pressure Nitroglycerin acts directly on the smooth muscle of the blood vessels, causing relaxation and dilation. As a result, hypotension can occur. The nurse would check the client's blood pressure before administering the second nitroglycerin tablet. Although the respirations and apical pulse may be checked, these vital signs are not affected by this medication. The temperature also is not associated with administration of this medication.

A client is having frequent premature ventricular contractions. The nurse should place priority on assessment of which item? 1. Sensation of palpitations 2. Causative factors, such as caffeine 3. Blood pressure and oxygen saturation 4. Precipitating factors, such as infection

Blood pressure and oxygen saturation Premature ventricular contractions can cause hemodynamic compromise. Therefore, the priority is to monitor the blood pressure and oxygen saturation. The shortened ventricular filling time can lead to decreased cardiac output. The client may be asymptomatic or may feel palpitations. Premature ventricular contractions can be caused by cardiac disorders; states of hypoxemia; any number of physiological stressors, such as infection, illness, surgery, or trauma; and intake of caffeine, nicotine, or alcohol.

A child is receiving succimer for the treatment of lead poisoning. The nurse should monitor which most important laboratory result? 1. Iron level 2. Calcium level 3. Red blood cell count 4. Blood urea nitrogen level

Blood urea nitrogen level Succimer is a medication that is used to treat lead poisoning. Renal function (blood urea nitrogen and creatinine) is monitored closely during the administration of chelation therapy because the medication is excreted via the kidneys. Although it is important to monitor the iron level, calcium level, and red blood cell count, these results are not specific to chelation therapy, so they are not the most important lab values to monitor.

A client with severe blood loss resulting from multiple trauma requires rapid transfusion of several units of blood. The nurse asks another health team member to obtain which device for use during the transfusion procedure to help reduce the risk of cardiac dysrhythmias? 1. Infusion pump 2. Pulse oximeter 3. Cardiac monitor 4. Blood-warming device

Blood-warming device If several units of blood are to be administered rapidly, a blood warmer should be used. Rapid transfusion of cool blood places the client at risk for cardiac dysrhythmias. To prevent this, the nurse warms the blood with a blood-warming device. Pulse oximetry and cardiac monitoring equipment are useful for the early assessment of complications but do not reduce the occurrence of cardiac dysrhythmias. Electronic infusion devices are not helpful in this case because the infusion must be rapid, and infusion devices generally are used to control the flow rate. In addition, not all infusion devices are made to handle blood or blood products.

A client is admitted to the hospital with a diagnosis of neurogenic shock after a traumatic motor vehicle collision. Which manifestation best characterizes this diagnosis? 1. Bradycardia 2. Hyperthermia 3. Hypoglycemia 4. Increased cardiac output

Bradycardia Neurogenic shock can occur after a spinal cord injury. Usually the body attempts to compensate massive vasodilation by becoming tachycardic to increase the amount of blood flow and oxygen delivered to the tissues; however, in neurogenic shock, the sympathetic nervous system is disrupted, so the parasympathetic system takes over, resulting in bradycardia. This insufficient pumping of the heart leads to a decrease in cardiac output. Hypoglycemia is not an indicator of neurogenic shock. Hypothermia develops because of the vasodilation and the inability to control body temperature through vasoconstriction.

Vasopressin is prescribed for a client with a diagnosis of bleeding esophageal varices. The nurse should prepare to administer this medication by which route? 1. Orally 2. By inhalation 3. By intravenous infusion 4. Through a Sengstaken-Blakemore tube

By intravenous infusion If bleeding occurs, the health team intervenes quickly to control it by combining vasoactive medications with endoscopic therapies. Vasoactive medications reduce portal pressure. Vasopressin is a synthetic antidiuretic hormone. Administration of this hormone reduces bleeding. It acts directly on gastrointestinal smooth muscle as a vasoconstrictor. To take advantage of these effects, it should be administered via continuous intravenous infusion. It can also be administered via the subcutaneous route. Therefore, the remaining options are incorrect.

The nurse receives a telephone call from a neighbor, who states that her 3-year-old child was found sitting on the kitchen floor with an empty bottle of liquid furniture polish. The mother of the child tells the nurse that the bottle was half full, that the child's breath smells like the polish, and that spilled polish is present on the front of the child's shirt. What should the nurse tell the mother to do first? 1. Call the pediatrician. 2. Induce vomiting immediately. 3. Call the Poison Control Center. 4. Bring the child to the emergency department.

Call the Poison Control Center. If a poisoning occurs, the Poison Control Center should be contacted immediately. Calling the pediatrician would not be the immediate action because this would delay treatment. In addition, the pediatrician would immediately make a referral to the Poison Control Center. Vomiting should not be induced if the victim is unconscious or the substance ingested was a strong corrosive or a petroleum-based product. The Poison Control Center may advise the mother to bring the child to the emergency department, and if this is the case, the mother should call an ambulance.

A depressed client is found unconscious on the floor in the dayroom of a health care facility. The nurse finds several empty bottles of a prescribed tricyclic antidepressant lying near the client. Which is the priority action of the nurse? 1. Call the Poison Control Center. 2. Call the emergency response team. 3. Determine the exact number of pills taken. 4. Induce vomiting and notify the health care provider.

Call the emergency response team. Tricyclic antidepressants can be fatal when taken as an overdose, regardless of the amount ingested. Life-threatening symptoms can develop after an overdose. Immediate emergency medical attention and cardiac monitoring are necessary with an overdose of tricyclic antidepressants. Options that delay immediate intervention would not be the priority actions. Vomiting is not induced in an unconscious client.

The nurse is caring for a client who had a resection of an abdominal aortic aneurysm yesterday. The client has an intravenous (IV) infusion at a rate of 150 mL/hour, unchanged for the last 10 hours. The client's urine output for the last 3 hours has been 90, 50, and 28 mL (28 mL is most recent). The client's blood urea nitrogen level is 35 mg/dL (12.6 mmol/L) and the serum creatinine level is 1.8 mg/dL (159 mcmol/L), measured this morning. Which nursing action is the priority? 1. Check the urine specific gravity. 2. Call the health care provider (HCP). 3. Put the IV line on a pump so that the infusion rate is sure to stay stable. 4. Check to see if the client had a blood sample for a serum albumin level drawn.

Call the health care provider (HCP). Following abdominal aortic aneurysm resection or repair, the nurse monitors the client for signs of acute kidney injury. Acute kidney injury can occur because often much blood is lost during the surgery and, depending on the aneurysm location, the renal arteries may be hypoperfused for a short period during surgery. Normal reference levels are BUN, 10-20 mg/dL (3.6-7.1 mmol/L), and creatinine, male, 0.6-1.2 mg/dL (53-106 mcmol/L) and female, 0.5-1.1 mg/dL (44-97 mcmol/L). Options 1 and 4 are not associated with the data in the question. The IV should have already been on a pump. Urine output lower than 30 mL/hour is reported to the HCP.

A client returning to the nursing unit after a cardiac catheterization procedure has a stat prescription to receive a dose of intravenous procainamide. Which piece of equipment would be most appropriate for the nurse to use in determining the client's response to this medication? 1. Glucometer 2. Pulse oximeter 3. Cardiac monitor 4. Noninvasive blood pressure cuff

Cardiac monitor Procainamide is an antiarrhythmic medication often used to treat ventricular arrhythmias that do not respond adequately to lidocaine. The effectiveness of this medication is best determined by evaluating the client's cardiac rhythm. Therefore, a cardiac monitor would be the most appropriate device for determining the client's response, although the blood pressure cuff and the pulse oximeter would provide general information about the client's cardiovascular status. A glucometer is not needed for this client with the information presented.

The nurse has a prescription to give amiodarone intravenously to a client. What is the priority assessment during administration of this medication? 1. Blood pressure 2. Cardiac rhythm 3. Skin color and dryness 4. Oxygen saturation level

Cardiac rhythm Amiodarone is an antidysrhythmic used to treat life-threatening ventricular dysrhythmias. The client requires continuous cardiac monitoring, with infusion of the medication by an intravenous pump. Although the other assessments are not incorrect, monitoring of cardiac rhythm is the priority nursing action.

A client is admitted to the emergency department with chest pain that is consistent with myocardial infarction based on elevated troponin levels. Heart sounds are normal and vital signs are noted on the client's chart. The nurse should alert the health care provider because these changes are most consistent with which complication? Refer to chart below. Time Pulse Respiratory Blood pressure 11:00 a.m. 92 beats/min 24 breaths/min 140/88 mm Hg 11:15 a.m. 96 beats/min 26 breaths/min 128/82 mm Hg 11:30 a.m. 104 beats/min 28 breaths/min 104/68 mm Hg 11:45 a.m. 118 beats/min 32 breaths/min 88/58 mm Hg 1. Cardiogenic shock 2. Cardiac tamponade 3. Pulmonary embolism 4. Dissecting thoracic aortic aneurysm

Cardiogenic shock Cardiogenic shock occurs with severe damage (more than 40%) to the left ventricle. Classic signs include hypotension; a rapid pulse that becomes weaker; decreased urine output; and cool, clammy skin. Respiratory rate increases as the body develops metabolic acidosis from shock. Cardiac tamponade is accompanied by distant, muffled heart sounds and prominent neck vessels. Pulmonary embolism presents suddenly with severe dyspnea accompanying the chest pain. Dissecting aortic aneurysms usually are accompanied by back pain.

The nurse has a prescription to hang a 1000-mL intravenous (IV) bag of 5% dextrose in water with 20 mEq of potassium chloride. The nurse also needs to hang an IV infusion of piperacillin/tazobactam. The client has one IV site. The nurse should plan to take which action first? 1. Start a second IV site. 2. Check compatibility of the medication and IV fluids. 3. Mix the prepackaged piperacillin/tazobactam per agency policy. 4. Prime the tubing with the IV solution, and back-prime the medication.

Check compatibility of the medication and IV fluids. When hanging an IV antibiotic, the nurse should first check compatibility of the medication and the IV fluids currently prescribed. If the fluids and medication are incompatible, it would then be appropriate to start a second IV site. If they are compatible, the nurse should hang them together so as to avoid having to start another IV site. After this, the nurse should prepare the prepackaged piperacillin/tazobactam per agency policy, then prime the tubing with the IV solution, and then back-prime the medication. Back-priming prevents any medication from being lost during the priming process.

The nurse is documenting information in a client's chart when the electrocardiogram telemetry alarm sounds, and the nurse notes that the client is in ventricular tachycardia (VT). The nurse rushes to the client's bedside and should perform which assessment first? 1. Heart rate 2. Blood pressure 3. Respiratory rate 4. Check responsiveness

Check responsiveness VT is associated with a significant decrease in cardiac output. Assessing for unresponsiveness determines whether the client is affected by the decreased cardiac output. Therefore, the first action is to determine responsiveness of the client. Then the nurse should check the client's pulse to determine the next treatment strategy.

The nurse has a prescription to administer phenytoin by intravenous (IV) push through an IV line infusing 1000 mL of 0.9% sodium chloride. Arrange the actions in the order that they should be performed. All options must be used.

Check the compatibility of phenytoin with the IV solution. Draw up the medication in a 3-mL syringe. Check the client's identification (ID) bracelet. Pinch off the IV tubing above the injection port. Inject the medication. Document that the medication was administered.

The nurse has just received a prescription to transfuse a unit of packed red blood cells for an assigned client. What action should the nurse take next? 1. Check a set of vital signs. 2. Order the blood from the blood bank. 3. Obtain Y-site blood administration tubing. 4. Check to be sure that consent for the transfusion has been signed.

Check to be sure that consent for the transfusion has been signed. After receiving a prescription for a blood transfusion, the first action the nurse should take should be to check to be sure that consent for the transfusion has been signed by the client. If the client has consented, the nurse should then check a set of vital signs to be sure there is no contraindication for a transfusion at that time, such as an elevation in temperature. If the vital signs are acceptable, the nurse can then gather supplies to administer the transfusion and order the blood from the blood bank.

The nurse is assisting to defibrillate a client in ventricular fibrillation. After placing the pad on the client's chest and before discharge, which intervention is a priority? 1. Ensure that the client has been intubated. 2. Set the defibrillator to the "synchronize" mode. 3. Administer an amiodarone bolus intravenously. 4. Confirm that the rhythm is actually ventricular fibrillation.

Confirm that the rhythm is actually ventricular fibrillation. Until the defibrillator is attached and charged, the client is resuscitated by using cardiopulmonary resuscitation. Once the defibrillator has been attached, the electrocardiogram is checked to verify that the rhythm is ventricular fibrillation or pulseless ventricular tachycardia. Leads also are checked for any loose connections. A nitroglycerin patch, if present, is removed. The client does not have to be intubated to be defibrillated. The machine is not set to the synchronous mode because there is no underlying rhythm with which to synchronize. Amiodarone may be given subsequently but is not required before defibrillation.

A client with the recent diagnosis of myocardial infarction and impaired renal function is recuperating on the cardiac step-down unit. The client's blood pressure has been borderline low, and intravenous (IV) fluids have been infusing at 100 mL/hour via a central line catheter in the right internal jugular vein for approximately 24 hours to increase renal output and maintain the blood pressure. Upon entering the client's room, the nurse notes that the client is breathing rapidly and is coughing. The nurse determines that the client is most likely experiencing which complication of IV therapy? 1. Hematoma 2. Air embolism 3. Systemic infection 4. Circulatory overload

Circulatory overload Circulatory (fluid) overload is a complication of IV therapy. Signs include rapid breathing, dyspnea, a moist cough, and crackles. When circulatory overload is present, the client's blood pressure also increases. Hematoma is characterized by ecchymosis, swelling, and leakage at the IV insertion site, as well as hard and painful lumps at the site. Air embolism is characterized by tachycardia, dyspnea, hypotension, cyanosis, and decreased level of consciousness. Systemic infection is characterized by chills, fever, malaise, headache, nausea, vomiting, backache, and tachycardia.

The nurse enters a client's room to assess the client, who began receiving a blood transfusion 45 minutes earlier, and notes that the client is flushed and dyspneic. On assessment, the nurse auscultates the presence of crackles in the lung bases. The nurse determines that this client most likely is experiencing which complication of blood transfusion therapy? 1. Bacteremia 2. Hypovolemia 3. Circulatory overload 4. Transfusion reaction

Circulatory overload Circulatory overload is caused by the infusion of blood at a rate too rapid for the client to tolerate. With circulatory overload, crackles are present in addition to dyspnea. An allergic reaction, which is one type of blood transfusion reaction, would produce symptoms such as flushing, dyspnea, itching, and a generalized rash. Hypovolemia is not likely a complication of blood transfusion. With bacteremia, the client would have a fever, which is not part of the clinical picture presented.

The nurse is monitoring a client receiving total parenteral nutrition (TPN). The client suddenly develops respiratory distress, dyspnea, and chest pain, and the nurse suspects air embolism. In order of priority, how should the nurse plan the actions to take? Arrange the actions in the order that they should be performed. All options must be used.

Clamp the intravenous (IV) catheter. Position the client in a left Trendelenburg's position. Contact the health care provider (HCP). Administer oxygen. Take the client's vital signs. Document the occurrence.

The nurse is administering lidocaine hydrochloride by the intravenous route. Which finding(s) should the nurse report to the health care provider immediately? 1. Urine output of 275 mL over the past 8 hours 2 Client complaints of blurred vision and nausea 3. Heart rate of 70 beats/min, blood pressure of 130/72 mm Hg 4. Client complaints of a headache and a temperature of 100°F (37.8°C) orally

Client complaints of blurred vision and nausea Blurred vision and nausea are common indicators of lidocaine toxicity. Urine output is greater than the minimum amount of 30 mL/hr, and therefore is adequate. The heart rate of 70 beats/min and the blood pressure of 130/72 mm Hg are normal. A headache and elevated temperature are important to note but are not related to the lidocaine.

The nurse is changing the central line dressing of a client receiving parenteral nutrition (PN) and notes that the catheter insertion site appears reddened. The nurse should next assess which item? 1. Client's temperature 2. Expiration date on the bag 3. Time of last dressing change 4. Tightness of tubing connections

Client's temperature Redness at the catheter insertion site is a possible indication of infection. The nurse would next assess for other signs of infection. Of the options given, the temperature is the next item to assess. The tightness of tubing connections should be assessed each time the PN is checked; loose connections would result in leakage, not skin redness. The expiration date on the bag is a viable option, but this also should be checked at the time the solution is hung and with each shift change. The time of the last dressing change should be checked with each shift change.

The nurse hears that a client receiving total parenteral nutrition (TPN) at 100 mL/hr has bilateral crackles and 1+ pedal edema during shift report. When the nurse obtains a daily weight, the nurse notes that the client has gained 4 lbs (1.8 kg) in 2 days. Which action should the nurse take first? 1. Administer the prescribed daily diuretic. 2. Encourage the client to cough and deep breathe. 3. Compare the intake and output records of the past 2 days. 4. Slow the TPN infusion rate to 50 mL/hr per infusion pump.

Compare the intake and output records of the past 2 days. The client is showing signs of fluid retention and possible excess fluid intake, as noted by the presence of crackles, edema, and weight gain. Noting the client's intake and output records adds to the database of information, which should then be reported. The problem may or may not be related to the TPN. (Other possible causes are impaired respiratory, cardiovascular, or renal function.) The nurse should not decrease the infusion rate without a health care provider's (HCP's) prescription. In addition, it is not recommended to increase or decrease the rate of TPN infusions because of the problems of hyperosmolar diuresis, hypoglycemia, or hyperglycemia. Coughing and deep breathing will have little, if any, effect on peripheral edema and weight gain. Administering a daily diuretic may delay necessary and immediate treatment and is incorrect.

The nurse discovers that an infusion of total parenteral nutrition (TPN) through a central line is empty, and a replacement bag is not yet ready. What should the nurse do next while waiting for the replacement bag? 1. Hang an intravenous infusion of normal saline. 2. Convert the intravenous infusion to a saline lock. 3. Hang an intravenous infusion of 5% dextrose in water. 4. Hang an intravenous infusion of 10% dextrose in water.

Hang an intravenous infusion of 10% dextrose in water. If TPN is discontinued abruptly, rebound hypoglycemia may occur because the pancreas has not yet had time to adjust its secretion of insulin in response to the lower amount of glucose. Therefore, a dextrose in water solution of 10% or 20% is infused temporarily until the replacement bag is available.

The nurse is monitoring the amount of lochia drainage in a client who is 2 hours postpartum and notes that the client has saturated a perineal pad in 15 minutes. How should the nurse respond to this finding initially? 1. Document the finding. 2. Encourage the client to ambulate. 3. Encourage the client to increase fluid intake. 4. Contact the health care provider (HCP) and inform the HCP of this finding.

Contact the health care provider (HCP) and inform the HCP of this finding. Lochia is the discharge from the uterus in the postpartum period; it consists of blood from the vessels of the placental site and debris from the decidua. The following can be used as a guide to determine the amount of flow: scant = less than 2.5 cm (<1 inch) on menstrual pad in 1 hour; light = less than 10 cm (<4 inches) on menstrual pad in 1 hour; moderate = less than 15 cm (<6 inches) on menstrual pad in 1 hour; heavy = saturated menstrual pad in 1 hour; and excessive = menstrual pad saturated in 15 minutes. If the client is experiencing excessive bleeding, the nurse should contact the HCP in the event that postpartum hemorrhage is occurring. It may be appropriate to encourage increased fluid intake, but this is not the initial action. It is not appropriate to encourage ambulation at this time. Documentation should occur once the client has been stabilized.

A client with a left arm fracture supported in a cast complains of loss of sensation in the left fingers. The nursing assessment identifies pallor in the distal portion of the arm, poor capillary refill, and a diminished left radial pulse. On the basis of these findings, the nurse would take which as a priority action? 1. Apply ice to the site. 2. Document the findings. 3. Administer pain medication. 4. Contact the health care provider (HCP).

Contact the health care provider (HCP). The client with limb pallor, slow capillary refill, weakened or lost pulse, and absence of sensation or motion in the distal limb may have arterial damage from a lacerated, contused, thrombosed, or severed artery. These signs also can occur with constriction from a tight cast. Regardless of the cause, the nurse notifies the HCP immediately. Emergency intervention is needed, which could entail removal of the constricting bandage, fracture reduction, or surgery to repair the area. Application of ice could exacerbate the problem. Although the nurse would document the findings, this is not the priority action. No data are presented in the question to indicate that the client needs pain medication.

The community health nurse is providing a teaching session to firefighters in a small community regarding care of a burn victim at the scene of injury. The nurse instructs the firefighters that in the event of a tar burn, which is the immediate action? 1. Cooling the injury with water 2. Removing all clothing immediately 3. Removing the tar from the burn injury 4. Leaving any clothing that is saturated with tar in place

Cooling the injury with water Scald burns and tar or asphalt burns are treated by immediate cooling with saline solution or water, if available, or immediate removal of the saturated clothing. Clothing that is burned into the skin is not removed because increased tissue damage and bleeding may result. No attempt is made to remove tar from the skin at the scene.

Intravenous (IV) fluids have been infusing at 100 mL/hour via a central line catheter in the right internal jugular for approximately 24 hours to increase urine output and maintain the client's blood pressure. Upon entering the client's room, the nurse notes that the client is breathing rapidly and coughing. For which additional signs of a complication should the nurse assess based on the previously known data? 1. Excessive bleeding 2. Crackles in the lungs 3. Incompatibility of the infusion 4. Chest pain radiating to the left arm

Crackles in the lungs Circulatory (fluid) overload is a complication of IV therapy. Signs include rapid breathing, dyspnea, a moist cough, and crackles. Blood pressure and heart rate also increase if circulatory overload is present. Therefore, since the nurse previously noted rapid breathing and coughing, the nurse should then assess for a moist cough and crackles. Hematoma is another potential complication and is characterized by ecchymosis, swelling, and leakage at the IV insertion site, as well as hard and painful lumps at the site. Allergic reaction is a complication of administration of IV fluids or medication and is characterized by chills, fever, malaise, headache, nausea, vomiting, backache, and tachycardia; this type of reaction could also occur if the IV solutions infused are incompatible; however, there was no indication of multiple solutions being infused simultaneously in this question. Chest pain radiating to the left arm is a classic sign of cardiac compromise and is not specifically related to a complication of IV therapy.

The nurse is monitoring a client who required a Sengstaken-Blakemore tube because other measures for treating bleeding esophageal varices were unsuccessful. The client complains of severe pain of abrupt onset. Which nursing action is most appropriate? 1. Cut the tube. 2. Reposition the client. 3. Assess the lumens of the tubes. 4. Administer the prescribed analgesics.

Cut the tube. Spontaneous rupture of the gastric balloon, upward migration of the tube, and occlusion of the airway are possible complications associated with a Sengstaken-Blakemore tube. Esophageal rupture also may occur and is characterized by the abrupt onset of severe pain. In the event of any of these life-threatening emergencies, the tube is cut and removed.

The nurse is preparing to infuse (piggyback) a 50-mL dose of a compatible medication through the primary intravenous (IV) line. How should the nurse correctly attach the medication bag? 1. Hanging the medication bag level with the primary IV bag 2. Hanging the medication bag lower than the primary IV bag 3. Hanging the medication bag higher than the primary IV bag 4. Disconnecting the primary IV solution and plugging in the medication

Hanging the medication bag higher than the primary IV bag For an intermittent IV infusion that is piggybacked to the primary IV line, the bag for the intermittent infusion is placed higher than the primary solution bag. This allows gravity to assist in infusing the medication. Once the intermittent infusion is complete, the primary IV infusion will resume at the drip rate set for the intermittent infusion. For this reason, it also is important to remember to check the infusion frequently and reset the primary IV drip rate correctly once the intermittent infusion is complete.

A client is being weaned from parenteral nutrition (PN) and is expected to begin taking solid food today. The ongoing solution rate has been 100 mL/hour. The nurse anticipates that which prescription regarding the PN solution will accompany the diet prescription? 1. Discontinue the PN. 2. Decrease PN rate to 50 mL/hour. 3. Start 0.9% normal saline at 25 mL/hour. 4. Continue current infusion rate prescriptions for PN.

Decrease PN rate to 50 mL/hour. When a client begins eating a regular diet after a period of receiving PN, the PN is decreased gradually. PN that is discontinued abruptly can cause hypoglycemia. Clients often have anorexia after being without food for some time, and the digestive tract also is not used to producing the digestive enzymes that will be needed. Gradually decreasing the infusion rate allows the client to remain adequately nourished during the transition to a normal diet and prevents the occurrence of hypoglycemia. Even before clients are started on a solid diet, they are given clear liquids followed by full liquids to further ease the transition. A solution of normal saline does not provide the glucose needed during the transition of discontinuing the PN and could cause the client to experience hypoglycemia.

A client has received a transfusion of platelets. The nurse evaluates that the client is benefiting most from this therapy if the client exhibits which finding? 1. Increased hematocrit level 2. Increased hemoglobin level 3. Decline of elevated temperature to normal 4. Decreased oozing of blood from puncture sites and gums

Decreased oozing of blood from puncture sites and gums Platelets are necessary for proper blood clotting. The client with insufficient platelets may exhibit frank bleeding or oozing of blood from puncture sites, wounds, and mucous membranes. Increased hemoglobin and hematocrit levels would occur when the client has received a transfusion of red blood cells. An elevated temperature would decline to normal after infusion of granulocytes because these cells were instrumental in fighting infection in the body.

Packed red blood cells have been prescribed for a female client with a hemoglobin level of 7.6 g/dL (76 mmol/L) and a hematocrit level of 30% (0.30). The nurse takes the client's temperature before hanging the blood transfusion and records 100.6°F (38.1°C) orally. Which action should the nurse take? 1. Begin the transfusion as prescribed. 2. Administer an antihistamine and begin the transfusion. 3. Delay hanging the blood and notify the health care provider (HCP). 4. Administer 2 tablets of acetaminophen and begin the transfusion.

Delay hanging the blood and notify the health care provider (HCP). If the client has a temperature higher than 100°F (37.8°C), the unit of blood should not be hung until the HCP is notified and has the opportunity to give further prescriptions. The HCP likely will prescribe that the blood be administered regardless of the temperature, or may instruct the nurse to administer prescribed acetaminophen and wait until the temperature has decreased before administration, but the decision is not within the nurse's scope of practice to make. The nurse needs an HCP's prescription to administer medications to the client.

An ultrasound is performed on a client at term gestation who is experiencing moderate vaginal bleeding. The results of the ultrasound indicate that abruptio placentae is present. On the basis of these findings, the nurse should prepare the client for which anticipated prescription? 1. Delivery of the fetus 2. Strict monitoring of intake and output 3. Complete bed rest for the remainder of the pregnancy 4. The need for weekly monitoring of coagulation studies until the time of delivery

Delivery of the fetus Abruptio placentae is the premature separation of the placenta from the uterine wall after the twentieth week of gestation and before the fetus is delivered. The goal of management in abruptio placentae is to control the hemorrhage and deliver the fetus as soon as possible. Delivery is the treatment of choice if the fetus is at term gestation or if the bleeding is moderate to severe and the client or fetus is in jeopardy. Because delivery of the fetus is necessary, options 2, 3, and 4 are incorrect regarding management of a client with abruptio placentae.

One unit of packed red blood cells has been prescribed for a client with severe anemia. The client has received multiple transfusions in the past, and it is documented that the client has experienced urticaria-type reactions from the transfusions. The nurse anticipates that which medication will be prescribed before administration of the red blood cells to prevent this type of reaction? 1. Ibuprofen 2. Acetaminophen 3. Diphenhydramine 4. Acetylsalicylic acid

Diphenhydramine An urticaria-type reaction is characterized by a rash accompanied by pruritus. This type of transfusion reaction is prevented by pretreating the client with an antihistamine such as diphenhydramine. The remaining medications would not prevent an urticaria-type reaction. Acetaminophen may be prescribed before the administration to assist in preventing an elevated temperature.

The nurse is monitoring a client who is receiving a blood transfusion. After 30 minutes of the infusion, the client begins to have chills and back pain. His temperature is 100.1°F (37.8°C). What action should the nurse take first? 1. Assess the client for other symptoms. 2. Slow the blood transfusion and monitor the client's vital signs. 3. Remind the client that these are expected reactions to a blood transfusion. 4. Discontinue the infusion and start an infusion of normal saline using new tubing.

Discontinue the infusion and start an infusion of normal saline using new tubing. Signs of a transfusion reaction include fever, chills, tachycardia, tachypnea, dyspnea, hives or skin rash, flushing, backache, and decreased blood pressure. If the client shows any symptoms of a blood transfusion reaction, the nurse needs to discontinue the infusion immediately and start an infusion of normal saline using new tubing connected to the hub of the intravenous insertion site. The nurse should stay with the client and monitor his or her condition while asking a colleague to notify the health care provider immediately.

The nurse is caring for a client in the emergency department who has sustained a head injury. The client momentarily lost consciousness at the time of the injury and then regained it. The client now has lost consciousness again. The nurse takes quick action, knowing that this sequence is compatible with which most likely condition? 1. Concussion 2. Skull fracture 3. Subdural hematoma 4. Epidural hematoma

Epidural hematoma he changes in neurological signs from an epidural hematoma begin with loss of consciousness as arterial blood collects in the epidural space and exerts pressure. The client regains consciousness as the cerebrospinal fluid is reabsorbed rapidly to compensate for the rising intracranial pressure. As the compensatory mechanisms fail, even small amounts of additional blood cause the intracranial pressure to rise rapidly, and the client's neurological status deteriorates quickly.

A client has experienced high blood pressure and crackles in the lungs during previous blood transfusions. The client asks the nurse whether it is safe to receive another transfusion. The nurse explains that which medication most likely will be prescribed before the transfusion is begun? 1. Furosemide 2. Acetaminophen 3. Diphenhydramine 4. Acetylsalicylic acid

Furosemide Fluid overload is one of the potential complications of a blood transfusion and is characterized by a variety of signs, including high blood pressure, fluid in the lungs manifesting as crackles, and distended jugular veins. This type of transfusion reaction is prevented by pretreating the client with a diuretic such as furosemide. Acetaminophen and aspirin are analgesics, which can also be used for analgesia. These medications may reduce fever as well but do not treat fluid overload.

A client has a closed head injury with increased intracranial pressure (ICP). The increased ICP is being managed by mannitol 25 g by the intravenous (IV) route every 2 hours. The nurse is planning to administer this medication via IV pump in what manner? 1. Mixed in solution with the IV antibiotics 2. Giving it slowly over 30 to 90 minutes 3. Piggybacked into the packed red blood cells 4. Giving it rapidly over 5 minutes by IV bolus

Giving it slowly over 30 to 90 minutes Mannitol is an osmotic diuretic. When used to treat increased ICP, it is given slowly over 30 to 90 minutes, not rapidly and not via IV bolus. Mannitol should not be mixed in solution with antibiotics, and nothing should be piggybacked with packed red blood cells.

While changing the tapes on a newly inserted tracheostomy tube, the client coughs and the tube is dislodged. Which is the initial nursing action? 1. Call the health care provider to reinsert the tube. 2. Grasp the retention sutures to spread the opening. 3. Call the respiratory therapy department to reinsert the tracheotomy. 4. Cover the tracheostomy site with a sterile dressing to prevent infection.

Grasp the retention sutures to spread the opening. If the tube is dislodged accidentally, the initial nursing action is to grasp the retention sutures and spread the opening. If agency policy permits, the nurse then attempts to replace the tube immediately. Calling ancillary services or the health care provider will delay treatment in this emergency situation. Covering the tracheostomy site will block the airway.

A client is undergoing fluid replacement after being burned on 20% of her body 12 hours ago. The nursing assessment reveals a blood pressure of 90/50 mm Hg, a pulse rate of 110 beats/minute, and a urine output of 20 mL over the past hour. The nurse reports the findings to the health care provider (HCP) and anticipates which prescription? 1. Transfusing 1 unit of packed red blood cells 2. Administering a diuretic to increase urine output 3. Increasing the amount of intravenous (IV) lactated Ringer's solution administered per hour 4. Changing the IV lactated Ringer's solution to one that contains 5% dextrose in water

Increasing the amount of intravenous (IV) lactated Ringer's solution administered per hour Fluid management during the first 24 hours following a burn injury generally includes the infusion of (usually) lactated Ringer's solution. Lactated Ringer's solution is an isotonic solution that contains electrolytes that will maintain fluid volume in the circulation. Fluid resuscitation is determined by urine output and hourly urine output should be at least 30 mL/hour. The client's urine output is indicative of insufficient fluid resuscitation, which places the client at risk for inadequate perfusion of the brain, heart, kidneys, and other body organs. Therefore, the HCP would prescribe an increase in the amount of IV lactated Ringer's solution administered per hour. There is nothing in the situation that calls for blood resplacement, which is not used for fluid therapy for burn injuries. Administering a diuretic would not correct the problem because fluid replacement is needed. Diuretics promote the removal of the circulating volume, thereby further compromising the inadequate tissue perfusion. Intravenous 5% dextrose solution is isotonic before administered but is hypotonic once the dextrose is metabolized. Hypotonic solutions are not appropriate for fluid resuscitation of a client with significant burn injuries.

The nurse is assigned the care of a client who experienced a myocardial infarction and is being monitored by cardiac telemetry. The nurse notes the sudden onset of this cardiac rhythm on the monitor. The nurse should immediately take which action? Refer to Figure. 1. Take the client's blood pressure. 2. Initiate cardiopulmonary resuscitation (CPR). 3. Place a nitroglycerin tablet under the client's tongue. 4. Continue to monitor the client for 1 minute and then contact the health care provider (HCP).

Initiate cardiopulmonary resuscitation (CPR). This cardiac rhythm identifies a coarse ventricular fibrillation (VF). The goals of treatment are to terminate VF promptly and to convert it to an organized rhythm. The HCP or an advanced cardiac life support-qualified nurse must immediately defibrillate the client. If a defibrillator is not readily available, CPR must be initiated until the defibrillator arrives. The remaining options are incorrect; these are not immediate actions and would delay life-saving treatment.

A 2-year-old child is being transported to the trauma center from a local community hospital for treatment of a burn injury that is estimated as covering more than 40% of the body. The burns are both partial- and full-thickness burns. The nurse is asked to prepare for the arrival of the child and gathers supplies, anticipating that which treatment will be prescribed initially? 1. Insertion of a Foley catheter 2. Insertion of a nasogastric tube 3. Administration of an anesthetic agent for sedation 4. Application of an antimicrobial agent to the burns

Insertion of a Foley catheter A Foley catheter is inserted into the child's bladder so that urine output can be accurately measured on an hourly basis. Although pain medication may be required, the child would not receive an anesthetic agent and should not be sedated. The burn wounds would be cleansed after assessment, but this would not be the initial action. Intravenous fluids are administered at a rate sufficient to keep the child's urine output at 1 to 2 mL/kg of body weight per hour for children weighing less than 30 kg, thus reflecting adequate tissue perfusion. A nasogastric tube may or may not be required but would not be the priority intervention.

The nurse in the hospital emergency department is preparing to administer fomepizole to a client with ethylene glycol (antifreeze) intoxication. The nurse should plan to administer this medication by which route? 1. Oral route 2. Intramuscular route 3. Intravenous (IV) route 4. Through a nasogastric tube

Intravenous (IV) route Fomepizole is a medication that is used to treat known or suspected ethylene glycol (antifreeze) intoxication. It is administered via the IV route. The routes in the remaining options are incorrect.

A woman was working in her garden. She accidentally sprayed insecticide into her right eye. She calls the emergency department, frantic and screaming for help. The nurse should instruct the woman to take which immediate action? 1. Irrigate the eyes with water. 2. Come to the emergency department. 3. Call the health care provider (HCP). 4. Irrigate the eyes with diluted hydrogen peroxide.

Irrigate the eyes with water. In this type of accident, the client is instructed to irrigate the eyes immediately with running water for at least 20 minutes, or until the emergency medical services personnel arrive. In the emergency department, the cleansing agent of choice is usually normal saline. Calling the HCP and going to the emergency department delays necessary intervention. Hydrogen peroxide is never placed in the eyes.

A client has frequent bursts of ventricular tachycardia on the cardiac monitor. What should the nurse be most concerned about with this dysrhythmia? 1. It can develop into ventricular fibrillation at any time. 2. It is almost impossible to convert to a normal rhythm. 3. It is uncomfortable for the client, giving a sense of impending doom. 4. It produces a high cardiac output that quickly leads to cerebral and myocardial ischemia.

It can develop into ventricular fibrillation at any time. Ventricular tachycardia is a life-threatening dysrhythmia that results from an irritable ectopic focus that takes over as the pacemaker for the heart. The low cardiac output that results can lead quickly to cerebral and myocardial ischemia. Clients frequently experience a feeling of impending doom. Ventricular tachycardia is treated with antidysrhythmic medications, cardioversion (if the client is awake), or defibrillation (loss of consciousness). Ventricular tachycardia can deteriorate into ventricular fibrillation at any time.

The nurse is caring for a child who was burned in a house fire. The nurse develops a plan of care for monitoring the child during the treatment for burn shock. The nurse identifies which assessment as providing the most accurate guide to determine the adequacy of fluid resuscitation? 1. Heart rate 2. Lung sounds 3. Level of consciousness 4. Amount of edema at the site of the burn injury

Level of consciousness The sensorium, or level of consciousness, is an important guide to the adequacy of fluid resuscitation. The burn injury itself does not affect the sensorium, so the child should be alert and oriented. Any alteration in sensorium should be evaluated further. A neurological assessment would determine the level of consciousness in the child. Although the remaining options are important in the assessment of the child with a burn injury, they would not provide an accurate assessment of the adequacy of fluid resuscitation.

The nurse is admitting a young child who arrived from the emergency department after treatment for acetaminophen overdose. After administering the antidote, the nurse should reassess the child, including which priority laboratory value? 1. Thyroid panel 2. Urine drug screen 3. Liver function panel 4. Kidney function tests

Liver function panel Acetaminophen overdose is harmful to the liver. Thyroid function is not affected by acetaminophen. A urine medication screen and kidney function tests may be evaluated; however, these laboratory values are not the priority concern.

The nurse is caring for a client who sustained a thermal burn caused by the inhalation of steam 24 hours ago. The nurse determines that the priority nursing action is to assess which item? 1. Pain level 2. Lung sounds 3. Ability to swallow 4. Laboratory results

Lung sounds The priority nursing action would be to assess lung sounds. Thermal burns to the lower airways can occur with the inhalation of steam or explosive gases or with the aspiration of scalding liquids. Thermal burns to the upper airways are more common and cause erythema and edema of the airways and mucosal blisters or ulcerations. The mucosal edema can lead to upper airway obstruction, particularly during the first 24 to 48 hours after burn injury.

A client begins experiencing wheezing, anxiety, swelling, and hives after eating shellfish and is brought to the emergency department. Which immediate action should the nurse implement? 1. Administer epinephrine. 2. Maintain a patent airway. 3. Administer a corticosteroid. 4. Apply a MedicAlert bracelet.

Maintain a patent airway Swelling, hives, lowered blood pressure, anxiety, and wheezing are indicative of anaphylaxis. If the client experiences an anaphylactic reaction, the immediate action would be to maintain a patent airway. The client then would receive epinephrine. Corticosteroids may also be prescribed. The client will need to be instructed about obtaining and wearing a MedicAlert bracelet, but this is not the immediate action.

A mother brings her child to the emergency department. Based on the child's sitting position, drooling, and apparent respiratory distress, a diagnosis of epiglottitis is suspected. In anticipation of the health care provider's prescriptions, in which order of priority would the nurse implement the actions? Arrange the actions in the order that they should be performed. All options must be used.

Maintain a patent airway. Assess breath sounds by auscultation. Obtain an oxygen saturation level using pulse oximetry. Insert an intravenous line for fluid administration. Obtain an axillary temperature. Administer an antipyretic. Maintenance of a patent airway is essential. Emergency intubation equipment should be readily available. Once a patent airway is determined or established and maintained, the breath sounds should be auscultated. This action is followed by checking the pulse oximetry. All of these interventions relate to the respiratory status and are the priority. Following this, fluid needs should be considered because their administration relates to maintaining circulatory status. Then the temperature is taken and an antipyretic is administered (usually by the rectal route) if needed.

he nurse prepares to administer acetylcysteine to the client with an overdose of acetaminophen. What is the appropriate action when administering this antidote? 1. Administer the medication subcutaneously in the deltoid muscle. 2. Administer the medication by intramuscular (IM) injection in the gluteal muscle. 3. Mix the medication in a flavored ice drink, and allow the client to drink the medication. 4. Administer the medication mixed in 50 mL of normal saline and piggybacked through the main intravenous (IV) line.

Mix the medication in a flavored ice drink, and allow the client to drink the medication. Acetylcysteine is the antidote for acetaminophen. Because acetylcysteine has a pervasive flavor of rotten eggs, it must be disguised in a flavored ice drink and is preferably drunk through a straw to minimize contact with the mouth. It is a solution that also is used as a mucolytic agent, administered via nebulization. It is not administered by the IV, IM, or subcutaneous route.

The nurse has developed a nursing care plan for a client with a burn injury to implement during the emergent phase. Which priority intervention should the nurse include in the plan of care? 1. Monitor vital signs every 4 hours. 2. Monitor mental status every hour. 3. Monitor intake and output every shift. 4. Obtain and record weight every other day.

Monitor mental status every hour. During the emergent phase after a burn injury, because of fluid volume deficits secondary to a burn injury, vital signs should be monitored every hour (every 4 hours is too infrequent) until the client is hemodynamically stable. The nurse should monitor the mental status of the client every hour for the first 48 hours. The weight should be obtained and recorded daily or twice daily, and intake and output measurements should be recorded on an hourly basis.

The nurse is developing a nursing care plan for a client with a circumferential burn injury of the right arm. What is the nurse's priority action? 1. Monitor the radial pulse every hour. 2. Keep the extremity in a dependent position. 3. Document any changes that occur in the pulse. 4. Place pressure dressings and wraps around the burn sites.

Monitor the radial pulse every hour. In a client with ineffective tissue perfusion related to a circumferential burn injury, peripheral pulses should be assessed every hour for 72 hours. The affected extremities should be elevated, and the health care provider should be notified of any changes in pulses, capillary refill, or pain sensation. Pressure dressings and wraps should not be applied around the circumferential burn because they could cause a further alteration in peripheral circulation.

The nurse in the recovery room is caring for a client who underwent neurosurgery. Sequential compression devices (SCDs) have been applied to prevent venous stasis. While awaiting client transfer to the intensive care unit, the recovery room nurse should perform which critical assessment? 1. Assess radial pulses. 2. Log roll client to check skin integrity. 3. Monitor hemoglobin and hematocrit levels. 4. Monitor vascular status of the lower extremities.

Monitor vascular status of the lower extremities. SCDs may be useful to provide circulatory assistance after surgery. The critical nursing assessment includes monitoring the vascular status of the lower extremities. The remaining options may be components of the nursing assessment, but these actions are not part of the critical assessment required with use of SCDs.

A client has frequent runs of ventricular tachycardia. The health care provider has prescribed flecainide. What is the best nursing action related to the effects of this medication while the client is hospitalized? 1. Monitor the client's urinary output. 2. Assess the client for neurological changes. 3. Keep the call bell within the client's reach. 4. Monitor vital signs and cardiac rhythm frequently.

Monitor vital signs and cardiac rhythm frequently. Flecainide is an antidysrhythmic medication that slows conduction and decreases excitability, conduction velocity, and automaticity. The nurse needs to monitor the client's vital signs for changes and cardiac rhythm for the development of a new or a worsening dysrhythmia. The remaining options are components of standard care.

The nurse is caring for a client who is receiving total parenteral nutrition (TPN) via a central line. Which nursing intervention would specifically provide assessment data related to the most common complication associated with TPN 1. Weighing the client daily 2. Monitoring the temperature 3. Monitoring intake and output (I&O) 4. Monitoring the blood urea nitrogen (BUN) level

Monitoring the temperature The most common complication associated with TPN is infection. Monitoring the temperature provides assessment data that would indicate infection in the client. Weighing the client daily and monitoring I&O provides information related to fluid volume overload. Monitoring the BUN level does not provide information about infection and is most closely related to assessing renal function

A postpartum client who received an epidural analgesic after giving birth by cesarean section is lethargic and has a respiratory rate of 8 breaths per minute. The nurse should obtain which medication from the emergency cart after notifying the health care provider? 1. Naloxone 2. Betamethasone 3. Morphine sulfate 4. Meperidine hydrochloride

Naloxone Opioids are used for epidural analgesia, which can lead to delayed respiratory depression. For this reason, respirations are monitored for 24 hours after administration of epidural analgesia. Naloxone is an opioid antagonist, which reverses the effects of opioids and is given if the respiratory rate falls below 8 breaths per minute. Betamethasone is a corticosteroid administered to enhance fetal lung maturity. Morphine sulfate and meperidine hydrochloride are opioids and would further compromise the respiratory rate.

A client is admitted to the hospital with a diagnosis of myocardial infarction (MI) and is going to have an intravenous (IV) nitroglycerin infusion started. Noting that the client does not have an intra-arterial monitoring line in place, what piece of equipment should the nurse obtain for use at the bedside? 1. Defibrillator 2. Pulse oximeter 3. Noninvasive blood pressure monitor 4. Central venous pressure (CVP) insertion tray

Noninvasive blood pressure monitor Nitroglycerin dilates both arteries and veins, causing peripheral blood pooling, thereby reducing preload, afterload, and myocardial work. This also accounts for the primary side effect of nitroglycerin, which is hypotension. In the absence of continuous direct arterial pressure (intra-arterial) monitoring, the nurse should use an automatic noninvasive blood pressure monitor. The remaining options are not specifically associated with the administration of IV nitroglycerin.

The nurse reviewing the operative record for a client who has just undergone cardiac surgery notes that the client's cardiac output immediately after surgery was 3.6 L/min. Which intervention is appropriate based on the client's cardiac output reading? 1. Notify the health care provider (HCP). 2. Continue to monitor the cardiac output. 3. Place the client in the shock position. 4. Increase the intravenous (IV) fluid rate.

Notify the health care provider (HCP). The normal cardiac output for the adult can range from 4 to 8 L/min and varies greatly with body size. The heart normally pumps 5 L of blood every minute. The HCP should be notified because the result falls below the normal limit. Continuing to monitor the client dismisses the fact that the cardiac output is decreased. Increasing fluids may exacerbate the cardiac workload, and the nurse should consult with the HCP before this intervention. There are no data to suggest the client is currently in shock, so placing the client in shock position would be of no benefit

The nurse is preparing a continuous intravenous (IV) infusion at the medication cart. As the nurse goes to insert the spike end of the IV tubing into the IV bag, the tubing drops and the spike end hits the top of the medication cart. The nurse should take which action? 1. Obtain a new IV bag. 2. Obtain new IV tubing. 3 Wipe the spike end of the tubing with povidine-iodine. 4. Scrub the spike end of the tubing with an alcohol swab.

Obtain new IV tubing. The nurse should obtain new IV tubing because contamination has occurred and could cause systemic infection in the client. There is no need to obtain a new IV bag because the bag was not contaminated. Wiping with povidine-iodine or alcohol is insufficient and is contraindicated because the spike will be inserted into the IV bag.

A client with parenteral nutrition (PN) infusing has disconnected the tubing from the central line catheter. The nurse assesses the client and suspects an air embolism. The nurse should immediately place the client in which position? 1. On the left side, with the head lower than the feet 2. On the left side, with the head higher than the feet 3. On the right side, with the head lower than the feet 4. On the right side, with the head higher than the feet

On the left side, with the head lower than the feet Air embolism occurs when air enters the catheter system, such as when the system is opened for intravenous (IV) tubing changes or when the IV tubing disconnects. Air embolism is a critical situation; if it is suspected, the client should be placed in a left side-lying position. The head should be lower than the feet. This position is used to minimize the effect of the air traveling as a bolus to the lungs by trapping it in the right side of the heart. The positions in the remaining options are inappropriate if an air embolism is suspected.

A client with total parenteral nutrition (TPN) infusing has disconnected the tubing from the central line catheter. The nurse assesses the client and suspects an air embolism. The nurse should immediately place the client in which position? 1. On the left side, with the head lower than the feet 2. On the left side, with the head higher than the feet 3. On the right side, with the head lower than the feet 4. On the right side, with the head higher than the feet

On the left side, with the head lower than the feet Air embolism occurs when air enters the catheter system, such as when the system is opened for intravenous (IV) tubing changes or when the IV tubing disconnects. Air embolism is a critical situation; if it is suspected, the client should be placed in a left side-lying position. The head should be lower than the feet. This position is used to minimize the effect of the air traveling as a bolus to the lungs by trapping it in the right side of the heart. The positions in the remaining options are inappropriate if an air embolism is suspected.

The nurse is initiating 1-rescuer cardiopulmonary resuscitation on an adult client. The nurse should place the hands in which position to begin chest compressions? 1. On the lower half of the sternum 2. On the upper half of the sternum 3. On the lower third of the sternum 4. On the upper third of the sternum

On the lower half of the sternum Chest locations are found by placing the hands on the lower half of the sternum. To locate this area, find the notch where the rib margin meets the sternum, and place the middle finger on this notch and the index finger next to it. Next, place the heel of the opposite hand on the lower half of the sternum, close to the index finger. Remove the first hand, place it on top of the hand on the sternum, and begin chest compressions. Chest compressions will not be as effective with the hand placements described in the remaining options.

A delivery room nurse is caring for a client in labor. The client tells the nurse that she feels that something is coming through the vagina. The nurse performs an assessment and notes the presence of the umbilical cord protruding from the vagina. The nurse should immediately place the client in which position? 1. Prone 2. Supine 3. On the side 4. Reverse Trendelenburg's

On the side If cord prolapse or compression is suspected, the client is immediately repositioned. Cord compression needs to be relieved to allow for adequate fetal oxygenation. The client may be turned to the side or the hips may be elevated to shift the fetal presenting part toward the diaphragm, thereby relieving cord compression. A hands-and-knees position may reduce compression on a cord that is entrapped behind the fetus. Prone, supine, and reverse Trendelenburg's positions will not shift the presenting part toward the diaphragm and could worsen the condition.

A client with depression receiving phenelzine sulfate suddenly complains of a severe headache and neck stiffness and soreness and then begins to vomit. The nurse takes the client's blood pressure and notes that it is 210/102 mm Hg. On the basis of the findings, the nurse should obtain which medication from the emergency drawer of the medication cart? 1. Phentolamine 2. Protamine sulfate 3. Calcium gluconate 4. Phenobarbital sodium

Phentolamine The antidote for hypertensive crisis is phentolamine. Hypertensive crisis may be manifested by hypertension, occipital headache radiating frontally, neck stiffness and soreness, nausea, vomiting, sweating, fever and chills, clammy skin, dilated pupils, and palpitations. Tachycardia or bradycardia and constricting chest pain also may be present.

The nurse notes that the site of a client's peripheral intravenous (IV) catheter is reddened, warm, painful, and slightly edematous proximal to the insertion point of the IV catheter. After taking appropriate steps to care for the client, the nurse should document in the medical record that the client experienced which condition? 1. Phlebitis of the vein 2. Infiltration of the IV line 3. Hypersensitivity to the IV solution 4. Allergic reaction to the IV catheter material

Phlebitis of the vein Phlebitis at an IV site can be distinguished by client discomfort at the site and by redness, warmth, and swelling proximal to the catheter. If phlebitis occurs, the nurse should discontinue the IV line and insert a new IV line at a different site. Coolness at the site would be noted if the IV catheter was infiltrated. An allergic reaction produces a rash, redness, and itching. A major reaction, such as hypersensitivity, can cause dyspnea, a swollen tongue, and cyanosis.

The nurse notes that the site of a client's peripheral intravenous (IV) catheter is reddened, warm, painful, and slightly edematous proximal to the insertion point of the IV catheter. After taking appropriate steps to care for the client, the nurse should document in the medical record that which occurred? 1. Phlebitis of the vein 2. Infiltration of the IV line 3. Hypersensitivity to the IV solution 4. Allergic reaction to the IV catheter

Phlebitis of the vein Phlebitis at an IV site can be distinguished by client discomfort at the site, as well as redness, warmth, and swelling proximal to the catheter. The IV line should be discontinued, and a new line should be inserted at a different site. The remaining options are incorrect occurrences.

The nurse in a labor room is performing a vaginal assessment on a pregnant client in labor. The nurse notes the presence of the umbilical cord protruding from the vagina. What is the first nursing action with this finding? 1. Gently push the cord into the vagina. 2. Place the client in Trendelenburg's position. 3. Find the closest telephone and page the health care provider stat. 4. Call the delivery room to notify the staff that the client will be transported immediately.

Place the client in Trendelenburg's position. When cord prolapse occurs, prompt actions are taken to relieve cord compression and increase fetal oxygenation. The client should be positioned with the hips higher than the head to shift the fetal presenting part toward the diaphragm. The nurse should push the call light to summon help, and other staff members should call the health care provider and notify the delivery room. If the cord is protruding from the vagina, no attempt should be made to replace it because to do so could traumatize it and reduce blood flow further. Also as a first action, the examiner should place a gloved hand into the vagina, and hold the presenting part off the umbilical cord. Oxygen, 8 to 10 L/minute, by face mask is administered to the client to increase fetal oxygenation.

Following infusion of a unit of packed red blood cells, the client has developed new onset of tachycardia, bounding pulses, crackles, and wheezes. Which action should the nurse implement first? 1. Maintain bed rest with legs elevated. 2. Place the client in high Fowler's position. 3. Increase the rate of infusion of intravenous fluids. 4. Consult with the health care provider (HCP) regarding initiation of oxygen therapy.

Place the client in high Fowler's position. New onset of tachycardia, bounding pulses, and crackles and wheezes posttransfusion is evidence of fluid overload, a complication associated with blood transfusions. Placing the client in a high Fowler's (upright) position will facilitate breathing. Measures that increase blood return to the heart, such as leg elevation and administration of IV fluids, should be avoided at this time. In addition, administration of fluids cannot be initiated without a prescription. Consulting with the HCP regarding administration of oxygen may be necessary, but positional changes take a short amount of time to do and should be initiated first.

A client receiving total parenteral nutrition (TPN) is demonstrating signs and symptoms of an air embolism. What is the first action by the nurse 1. Stop the TPN solution. 2. Notify the health care provider (HCP). 3. Place the client in the high Fowler's position. 4. Place the client on the left side in the Trendelenburg's position.

Place the client on the left side in the Trendelenburg's position. Although stopping the TPN solution will not treat the problem, it will prevent it from worsening, and is a quick action that can be completed first. Lying on the left side may prevent air from flowing into the pulmonary veins. Trendelenburg's position increases intrathoracic pressure, which decreases the amount of blood pulled into the vena cava during inspiration. The high Fowler's position is not helpful at this time. The HCP should be notified, but this is not the first action.

The nurse is caring for a client with a chest tube drainage system. While the client is being assisted to sit up in bed in preparation for ambulation, the chest tube accidentally disconnects from the chest drainage system. Which is the initial nursing action? 1. Contact the health care provider (HCP). 2. Call a respiratory therapist to come to the bedside. 3. Encourage the client to perform the Valsalva maneuver. 4. Place the end of the chest tube in a container of sterile water.

Place the end of the chest tube in a container of sterile water. If a chest tube becomes disconnected, the nurse should as quickly as possible place the end of the tube in a container of sterile water until the drainage system can be replaced. It is not necessary to contact a respiratory therapist at this time. It may be necessary to contact the HCP, but this would not be the initial nursing action. Asking the client to perform a Valsalva maneuver is not appropriate and could be harmful.

The nurse caring for a client with a chest tube turns the client to the side and the chest tube accidentally disconnects from the water seal chamber. Which initial action should the nurse take? 1. Call the health care provider (HCP). 2. Place the tube in a bottle of sterile water. 3. Replace the chest tube system immediately. 4. Place a sterile dressing over the disconnection site.

Place the tube in a bottle of sterile water. If the chest drainage system is disconnected, the end of the tube is placed in a bottle of sterile water held below the level of the chest. The HCP may need to be notified, but this is not the initial action. The system is replaced if it breaks or cracks or if the collection chamber is full. Placing a sterile dressing over the disconnection site will not prevent complications resulting from the disconnection.

The nurse is caring for a client who sustained multiple fractures in a motor vehicle crash 12 hours earlier. The client now exhibits severe dyspnea, tachycardia, and mental confusion, and the nurse suspects fat embolism. Which is the initial nursing action? 1. Reassess the vital signs. 2. Palpate bilateral peripheral pulses. 3. Perform a neurological assessment. 4. Position the client in a Fowler's position.

Position the client in a Fowler's position. Clients with fractures are at risk for fat embolism. With suspected fat embolism, the nurse would position the client in a sitting (Fowler's) position to relieve dyspnea. Supplemental oxygen is indicated to reduce the signs of hypoxia. The health care provider needs to be notified. Vital signs will need to be taken, but this action may delay initial and required interventions. Peripheral pulse assessment is not a priority action. A neurological assessment needs to be performed, but this would not be the initial nursing action

A client in sinus bradycardia, with a heart rate of 45 beats/minute, complains of dizziness and has a blood pressure of 82/60 mm Hg. Which prescription should the nurse anticipate will be prescribed? 1. Administer digoxin. 2. Defibrillate the client. 3. Continue to monitor the client. 4. Prepare for transcutaneous pacing.

Prepare for transcutaneous pacing. Sinus bradycardia is noted with a heart rate less than 60 beats per minute. This rhythm becomes a concern when the client becomes symptomatic. Hypotension and dizziness are signs of decreased cardiac output. Transcutaneous pacing provides a temporary measure to increase the heart rate and thus perfusion in the symptomatic client. Defibrillation is used for treatment of pulseless ventricular tachycardia and ventricular fibrillation. Digoxin will further decrease the client's heart rate. Continuing to monitor the client delays necessary intervention.

A client receiving parenteral nutrition (PN) suddenly develops a fever. The nurse notifies the health care provider (HCP), and the HCP initially prescribes that the solution and tubing be changed. What should the nurse do with the discontinued materials? 1. Discard them in the unit trash. 2. Return them to the hospital pharmacy. 3. Save them for return to the manufacturer. 4. Prepare to send them to the laboratory for culture.

Prepare to send them to the laboratory for culture. When the client who is receiving PN develops a fever, a catheter-related infection should be suspected. The solution and tubing should be changed, and the discontinued materials should be cultured for infectious organisms per HCP prescription. The other options are incorrect. Because culture for infectious organisms is necessary, the discontinued materials are not discarded or returned to the pharmacy or manufacturer.

A client admitted with hypertensive crisis has an intravenous (IV) infusion of 1000 mL of normal saline with 20 mEq of potassium chloride added. A prescription is written to administer sodium nitroprusside by continuous IV infusion. The nurse should plan to do which to administer this medication? 1. Monitor the blood pressure every 15 minutes during administration. 2. Protect the sodium nitroprusside from light with an opaque material. 3. Check the solution for a faint brown coloration and discard it if this is noticed. 4. Piggyback the sodium nitroprusside into the IV line containing the potassium chloride.

Protect the sodium nitroprusside from light with an opaque material. Sodium nitroprusside can be degraded by light and should be protected with an opaque material. It is dispensed in powdered form and must be dissolved and diluted for the IV solution. A fresh solution may have a faint brown coloration, but solutions that are deeply colored, such as blue-green or dark red, should be discarded. No other medication should be mixed with the infusion solution. During the infusion, the blood pressure should be monitored continuously either through an arterial line or with an electronic monitoring device.

A mother brings her child to the emergency department. Based on the child's sitting position, drooling, and apparent respiratory distress, a diagnosis of epiglottitis is suspected. The nurse should plan for which priority intervention? 1. Obtaining a chest x-ray 2. Asking the mother about the precipitating events 3. Obtaining weight for correct antibiotic dose infusion 4. Providing assisted ventilation and obtaining the necessary equipment

Providing assisted ventilation and obtaining the necessary equipment The highest priority with epiglottitis is to have assisted ventilation available because the highest risk with this child is complete airway obstruction. Therefore, interventions related to airway are the priority. Physiological interventions continue to have the highest priority, with assessment of breath and heart sounds and then obtaining pulse oximetry being priorities. Once the airway is stabilized, the temperature, weight, and a chest x-ray can be obtained. The last priority is asking about precipitating events, which is done once physiological needs are met.

Which readings obtained from a client's pulmonary artery catheter suggest that the client is in left-sided heart failure? 1. Cardiac output of 5 L/min 2. Right atrial pressure of 9 mm Hg 3. Pulmonary capillary wedge pressure (PCWP) of 20 mm Hg 4. Pulmonary artery systolic/diastolic pressures of 24/10 mm Hg

Pulmonary capillary wedge pressure (PCWP) of 20 mm Hg Normal PCWP ranges from 8 to 15 mm Hg. A PCWP of 20 mm Hg is elevated and corresponds to volume overload of the left ventricle, such as occurs in heart failure. Options 1, 2, and 4 are normal values.

A client is admitted to the hospital for an acute episode of angina pectoris. Which parameter is the priority for the nurse to monitor? 1. Pulse and blood pressure 2. Temperature and respirations 3. Food tolerance and urinary output 4. Right upper quadrant pain and fatigue

Pulse and blood pressure Angina pectoris is transient chest pain or discomfort that is caused by an imbalance between myocardial oxygen supply and demand. The discomfort typically occurs in the retrosternal area; may or may not radiate; and is described as a tight, heavy, squeezing, burning, or choking sensation. The two major types of angina pectoris are stable (classic exertional) angina and unstable angina. Stable angina, the most common type, is usually precipitated by physical exertion or emotional stress, lasts 3 to 5 minutes, and is relieved by rest and nitroglycerin. Acute intervention for the client who has an anginal attack includes vital signs, oxygen, pain relief, and continuous electrocardiographic monitoring.

The health care provider (HCP) prescribes fat emulsion, given intravenously, for a client. The nurse should consult with the HCP before administering the fat emulsion solution if which is noted in the client's record? 1. The client has an allergy to iodine. 2. The client has an allergy to egg yolks. 3. The client is receiving total parenteral nutrition. 4. The client has a blood glucose level of 120 mg/dL (6.9 mmol/L).

The client has an allergy to egg yolks. Before administering any medication, the nurse must assess for allergy or hypersensitivity to substances used in producing the medication. Fat emulsions may contain an emulsifying agent obtained from egg yolks. Clients sensitive to eggs are at risk for developing hypersensitivity reactions. Options 1, 3, and 4 are not contraindications to the administration of fat emulsions.

The nurse is assessing a client's peripheral intravenous (IV) site after completion of a vancomycin infusion and notes that the area is reddened, warm, painful, and slightly edematous proximal to the insertion point of the IV catheter. At this time, which action by the nurse is best? 1. Check for the presence of blood return. 2. Remove the IV site and restart at another site. 3. Document the findings and continue to monitor the IV site. 4. Call the health care provider (HCP) and request that the vancomycin be given orally.

Remove the IV site and restart at another site. Phlebitis at an IV site can be distinguished by client discomfort at the site and by redness, warmth, and swelling proximal to the catheter. If phlebitis occurs, the nurse should remove the IV line and insert a new IV line at a different site, in a vein other than the one that has developed phlebitis. Checking for the presence of blood return should be done before the administration of vancomycin because this medication is a vesicant. Documenting the findings and continuing to monitor the IV site and calling the HCP and requesting that the vancomycin be given orally do not address the immediate problem. Additionally, there could be indications for the prescription of IV as opposed to oral vancomycin for the client. The HCP should be notified of the complications with the IV site, but not asked for a prescription for oral vancomycin.

The industrial nurse is providing instructions to a group of employees regarding care to a client in the event of a chemical burn injury. The nurse instructs the employees that which is the first consideration in immediate care? 1. Removing all clothing, including gloves, shoes, and any undergarments 2. Determining the antidote for the chemical and placing the antidote on the burn site 3. Leaving all clothing in place until the client is brought to the emergency department 4. Lavaging the skin with water and avoiding brushing powdered chemicals off the clothing

Removing all clothing, including gloves, shoes, and any undergarments In a chemical burn injury the burning process continues as long as the chemical is in contact with the skin. All clothing, including gloves, shoes, and undergarments, is removed immediately, and water lavage is instituted before and during transport to the emergency department. Powdered chemicals are first brushed off the client before lavage is performed.

Fat emulsion is prescribed for the client receiving total parenteral nutrition. The nurse is preparing to administer the fat emulsion and notes the presence of fat globules in the solution. What should the nurse do? 1. Call the health care provider (HCP). 2. Return the solution to the pharmacy. 3. Shake the solution to dissolve the globules. 4. Place the solution in a bath of warm water until the globules dissolve.

Return the solution to the pharmacy. The nurse should not hang a fat emulsion that has visible fat globules. Another solution should be obtained and used instead. Shaking the solution or placing the solution in a warm-water bath is not an appropriate action. Contacting the HCP also is inappropriate; the HCP's permission is not needed to obtain another bottle of solution.

A client has been admitted with chest trauma after a motor vehicle crash and has undergone subsequent intubation. The nurse checks the client when the high-pressure alarm on the ventilator sounds, and notes that the client has absence of breath sounds in the right upper lobe of the lung. The nurse immediately assesses for other signs of which condition? 1. Right pneumothorax 2. Pulmonary embolism 3. Displaced endotracheal tube 4. Acute respiratory distress syndrome

Right pneumothorax Pneumothorax is characterized by restlessness, tachycardia, dyspnea, pain with respiration, asymmetrical chest expansion, and diminished or absent breath sounds on the affected side. Pneumothorax can cause increased airway pressure because of resistance to lung inflation. Acute respiratory distress syndrome and pulmonary embolism are not characterized by absent breath sounds. An endotracheal tube that is inserted too far can cause absent breath sounds, but the lack of breath sounds most likely would be on the left side because of the degree of curvature of the right and left mainstem bronchi.

The nurse has a prescription to hang a 1000-mL intravenous (IV) bag of 5% dextrose in water with 20 mEq of potassium chloride and needs to add the medication to the IV bag. The nurse should plan to take which action immediately after injecting the potassium chloride into the port of the IV bag? 1. Rotate the bag gently. 2. Attach the tubing to the client. 3. Prime the tubing with the IV solution. 4. Check the solution for yellowish discoloration.

Rotate the bag gently. After adding a medication to a bag of IV solution, the nurse should agitate or rotate the bag gently to mix the medication evenly in the solution. The nurse should then attach a completed medication label. The nurse can then prime the tubing. The IV solution should have been checked for discoloration before the medication was added to the solution. The tubing is attached to the client last.

The nurse determines that a client is having a transfusion reaction. After the nurse stops the transfusion, which action should be taken next? 1. Remove the intravenous (IV) line. 2. Run a solution of 5% dextrose in water. 3. Run normal saline at a keep-vein-open rate. 4. Obtain a culture of the tip of the catheter device removed from the client.

Run normal saline at a keep-vein-open rate. If the nurse suspects a transfusion reaction, the nurse stops the transfusion and infuses normal saline at a keep-vein-open rate pending further health care provider prescriptions. This maintains a patent IV access line and aids in maintaining the client's intravascular volume. The nurse would not remove the IV line because then there would be no IV access route. Obtaining a culture of the tip of the catheter device removed from the client is incorrect. First, the catheter should not be removed. Second, cultures are performed when infection, not transfusion reaction, is suspected. Normal saline is the solution of choice over solutions containing dextrose because saline does not cause red blood cells to clump.

A client receiving thrombolytic therapy with a continuous infusion of alteplase suddenly becomes extremely anxious and complains of itching. The nurse hears stridor and notes generalized urticaria and hypotension. Which nursing action is the priority? 1. Administer oxygen and protamine sulfate. 2. Cut the infusion rate in half and sit the client up in bed. 3. Stop the infusion and call for the Rapid Response Team (RRT). 4. Administer diphenhydramine and epinephrine and continue the infusion.

Stop the infusion and call for the Rapid Response Team (RRT). The client is experiencing an anaphylactic reaction. Therefore, the priority action is to stop the infusion and notify the RRT. The health care provider should be contacted once the client has been stabilized. The client may be treated with epinephrine, antihistamines, and corticosteroids as prescribed, but the infusion should not be continued.

The nurse is caring for a restless client who is beginning nutritional therapy with parenteral nutrition (PN). The nurse should plan to ensure that which action is taken to prevent the client from sustaining injury? 1. Calculate daily intake and output. 2. Monitor the temperature once daily. 3. Secure all connections in the PN system. 4. Monitor blood glucose levels every 12 hours.

Secure all connections in the PN system. The nurse should plan to secure all connections in the tubing (connections are used per agency protocol). This helps to prevent the restless client from pulling the connections apart accidentally. The nurse should also monitor intake and output, but this does not relate specifically to a risk for injury as presented in the question. Also, monitoring the temperature and blood glucose levels does not relate to a risk for injury as presented in the question. In addition, the client's temperature and blood glucose levels are monitored more frequently than the time frames identified in the options to detect signs of infection and hyperglycemia, respectively

The nurse is caring for a client who has overdosed on phenobarbital. The nurse anticipates which assessment finding with this client? 1. Hyperthermia 2. Hyperreflexia 3. Deep respirations 4. Shallow respirations

Shallow respirations A client experiencing an overdose from barbiturates (such as phenobarbital) will experience shallow respirations; cold, clammy skin; weak, rapid pulse; hyporeflexia; coma; and possible death. Therefore, the remaining options are incorrect.

A client had a 1000-mL bag of 5% dextrose in 0.9% sodium chloride hung at 1500. The nurse, making rounds at 1545, finds that the client is apprehensive, complaining of a pounding headache, is dyspneic with chills, and has an increased pulse rate. The intravenous (IV) bag has 400 mL remaining. The nurse should take which action first? 1. Shut off the infusion. 2. Sit the client up in bed. 3. Remove the angiocatheter and IV quickly. 4. Place the client in the Trendelenburg's position.

Shut off the infusion. The client's symptoms are compatible with speed shock (fluid volume overload). This may be verified by noting that 600 mL has infused in the course of 45 minutes. The first action of the nurse from the options presented is to shut off the infusion. The nurse may elevate the head of the bed to aid the client's breathing. Placing the client in the Trendelenburg's position is not an appropriate action and will increase the dyspnea. The nurse also notifies the health care provider. The angiocatheter is not removed. An IV access may be needed to treat the complication and may continue to be needed when the complication has been managed.

A client had a 1000-mL bag of 5% dextrose in 0.9% sodium chloride hung at 1500. The nurse making rounds at 1545 finds that the client is complaining of a pounding headache and is dyspneic, experiencing chills, and apprehensive, with an increased pulse rate. The intravenous (IV) bag has 400 mL remaining. The nurse should take which action first? 1. Slow the IV infusion. 2. Sit the client up in bed. 3. Remove the IV catheter. 4. Call the health care provider (HCP).

Slow the IV infusion. The client's symptoms are compatible with circulatory overload. This may be verified by noting that 600 mL has infused in the course of 45 minutes. The first action of the nurse is to slow the infusion. Other actions may follow in rapid sequence. The nurse may elevate the head of the bed to aid the client's breathing, if necessary. The nurse also notifies the HCP. The IV catheter is not removed; it may be needed for the administration of medications to resolve the complication.

The nurse notes that a 14-year-old child is choking but is awake and alert at this time. The nurse rushes to perform the abdominal thrust maneuver. The child becomes unconscious. What procedure should the nurse perform next? 1. Perform a finger sweep. 2. Start chest compressions. 3. Attempt rescue breathing. 4. Ask the parent what happened.

Start chest compressions. To perform the abdominal thrust maneuver for a conscious child, the rescuer stands or kneels behind the child and places the arms directly under the child's axillae and then around the child. The thumb side of 1 fist is placed against the child's abdomen in the midline slightly above the umbilicus and well below the tip of the xiphoid process. The xiphoid process and ribs are avoided to prevent damage to internal organs. The fist is grasped with the other hand, and upward thrusts are delivered. If the child becomes unconscious, the nurse should start cardiopulmonary resuscitation, first beginning compressions. Performing a blind finger sweep is not recommended. If the object can be visualized and is retrievable, it is acceptable to attempt to remove the object. Rescue breathing is not appropriate at this time but may be necessary later. It will be necessary at some point to determine what happened, but this would not be the nurse's next action.

A health care provider has written a prescription to discontinue an intravenous (IV) line. The nurse should obtain which item from the unit supply area for applying pressure to the site after removing the IV catheter? 1. Elastic wrap 2. Sterile 2 × 2 gauze 3. Adhesive bandage 4. Povidine-iodine swab

Sterile 2 × 2 gauze A dry sterile dressing such as sterile 2 × 2 gauze is used to apply pressure to the discontinued IV site. This material is absorbent, sterile, and nonirritating. A povidine-iodine swab would irritate the opened puncture site and would not stop the blood flow. An adhesive bandage or elastic wrap may be used to cover the site once hemostasis has occurred.

A client has an epidural catheter in place after colon surgery and is receiving pain medication through the catheter. During the night the client calls the nurse and says, "I have a terrible headache that just started now." The nurse checks the epidural catheter insertion site and notes a small amount of clear drainage leaking from the bandage. What is the first action the nurse should take? 1. Stop the infusion. 2. Change the dressing bandage. 3. Remove the epidural catheter. 4. Notify the health care provider (HCP).

Stop the infusion. If a client complains of a sudden headache and clear drainage is present near the epidural insertion site, it is possible that the catheter has migrated. The immediate actions by the nurse are to stop the infusion and then to notify the HCP. The HCP needs to be notified, but the nurse can delegate that task to a colleague while caring for the client. It is not appropriate for the nurse to remove the epidural catheter without a prescription. Simply changing the bandage does not address the critical problem.

The nurse is caring for a client immediately after removal of the endotracheal tube. The nurse should report which sign immediately if experienced by the client? 1. Stridor 2. Occasional pink-tinged sputum 3. Respiratory rate of 24 breaths/minute 4. A few basilar lung crackles on the right

Stridor

A postoperative client receives a dose of naloxone hydrochloride for respiratory depression shortly after transfer to the nursing unit from the postanesthesia care unit. After administration of the medication, the nurse should assess the client for which change? 1. Pupillary changes 2. Scattered lung wheezes 3. Sudden increase in pain 4. Sudden episodes of vomiting

Sudden increase in pain Naloxone hydrochloride is an antidote to opioids, and it also may be given to postoperative clients to treat respiratory depression. When given postoperatively for respiratory depression, it may reverse the effects of analgesics. Therefore, the nurse must assess the client for a sudden increase in level of postoperative pain. The remaining options are unrelated to the administration of naloxone.

A child is admitted to the hospital after being seen in the emergency department with complaints of right lower quadrant abdominal pain, nausea and vomiting, fever, and chills. The health care provider (HCP) suspects appendicitis. Which assessment finding should the nurse immediately report to the HCP? 1. Decreasing oral temperature 2. Increasing complaints of pain 3. Refusal to take fluids by mouth 4. Sudden relief of abdominal pain

Sudden relief of abdominal pain A sudden relief of pain from a suspected appendicitis is commonly indicative of a ruptured appendix. This places the individual at risk for peritonitis and shock. The HCP should be notified immediately because of the need to begin intravenous antibiotics to prevent further complications. Although increasing complaints of pain is a concern, the higher priority is sudden relief of pain because of the risk of peritonitis and shock. Temperature should be monitored but is not of highest priority. The child will be placed on NPO (nothing by mouth) status in anticipation of surgery; therefore, option 4 is incorrect.

A client has been discharged to home on total parenteral nutrition (TPN). With each visit, the home care nurse should assess which parameters most closely in monitoring this therapy? 1. Pulse and weight 2. Temperature and weight 3. Pulse and blood pressure 4. Temperature and blood pressure

Temperature and weight The client receiving TPN at home should have the temperature monitored as a means of detecting infection, which is a potential complication of this therapy. An infection also could result in sepsis because the catheter is in a blood vessel. The client's weight is monitored as a measure of the effectiveness of this nutritional therapy and to detect hypervolemia. The pulse and blood pressure are important parameters to assess, but they do not relate specifically to the effects of TPN.

An adult client has been unsuccessfully defibrillated for ventricular fibrillation, and cardiopulmonary resuscitation (CPR) is resumed. The nurse confirms that CPR is being administered effectively by noting which action? 1. The ratio of compressions to ventilations is 30:2. 2. The carotid pulse is palpable with each compression. 3. Respirations are given at a rate of 10 breaths per minute. 4. The chest compressions are given at a depth of 1.5 to 2 inches (2.5 to 5 cm).

The carotid pulse is palpable with each compression. With effective compressions, carotid pulsations should be present. At its best, CPR produces only 30% of the normal cardiac output, so correct technique is vital. Assessment of the carotid pulse during CPR is the most accurate way to assess the effectiveness of CPR. Correct procedure for CPR in an adult includes a compression-to-ventilation ratio of 30:2. With adults, compressions are performed at a depth of at least 2 inches (5 cm). The 30:2 compression-to-ventilation ratio yields an effective rate of 10 breaths per minute.

The health care provider (HCP) arrives on the nursing unit and deflates the esophageal balloon of a client's Sengstaken-Blakemore tube. The nurse should contact the HCP immediately if which occurs? 1. The client has some diarrhea that is bloody. 2. The client's blood pressure is 128/78 mm Hg. 3. The client complains of abdominal discomfort. 4. The client complains of nausea and vomits blood.

The client complains of nausea and vomits blood. A Sengstaken-Blakemore tube may be used in a client with a diagnosis of cirrhosis and ruptured esophageal varices if other interventions are unsuccessful. The tube has an esophageal and a gastric balloon. The esophageal balloon exerts pressure on the ruptured esophageal varices and stops the bleeding. The pressure of the esophageal balloon is released at intervals to decrease the risk of trauma to esophageal tissues, including esophageal rupture or necrosis. When the balloon is deflated, the client may begin to bleed again from the esophageal varices; this would be manifested by vomiting of blood (hematemesis). The remaining options are not specifically associated with esophageal deflation.

Which should the nurse do when setting up an arterial line? 1. Tighten all tubing connections. 2. Use macrodrop intravenous tubing. 3. Level the transducer to the ventricle. 4. Raise the height of the normal saline infusion to prevent backup.

Tighten all tubing connections. Because the arterial vasculature is a high-pressure system, all tubing connections must be tight to avoid blood loss from loose connections. High-pressure tubing with a transducer is used (not macrodrip tubing). The transducer should be level to the atrium, not the ventricle. Raising the height of the infusion is not sufficient to prevent backflow.

The nurse walking in a downtown business area witnesses a worker fall from a ladder. The nurse rushes to the victim, who is unresponsive. A layperson is attempting to perform resuscitative measures. The nurse should intervene if which action by the layperson is noted? 1. Use of the head tilt-chin lift 2. Checking the scene for safety 3. Use of the jaw thrust maneuver 4. Moving the client away from a busy traffic road

Use of the head tilt-chin lift Whenever a neck injury is suspected, the jaw thrust maneuver should be used during basic life support (BLS) to open the airway. The head tilt-chin lift produces hyperextension of the neck and could cause complications if a neck injury is present. The scene should be checked for safety, and the client should be moved away from a busy traffic road in order to ensure safety.

The low-pressure alarm sounds on a ventilator. The nurse assesses the client and then attempts to determine the cause of the alarm. If unsuccessful in determining the cause of the alarm, the nurse should take what initial action? 1. Administer oxygen. 2. Check the client's vital signs. 3. Ventilate the client manually. 4. Start cardiopulmonary resuscitation.

Ventilate the client manually. If at any time an alarm is sounding and the nurse cannot quickly ascertain the problem, the client is disconnected from the ventilator and manual resuscitation is used to support respirations until the problem can be corrected. No reason is given to begin cardiopulmonary resuscitation. Checking vital signs is not the initial action. Although oxygen is helpful, it will not provide ventilation to the client.

A client with myocardial infarction is developing cardiogenic shock. Because of the risk of myocardial ischemia, what condition should the nurse carefully assess the client for? 1. Bradycardia 2. Ventricular dysrhythmias 3. Rising diastolic blood pressure 4. Falling central venous pressure

Ventricular dysrhythmias Classic signs of cardiogenic shock as they relate to myocardial ischemia include low blood pressure and tachycardia. The central venous pressure would rise as the backward effects of the severe left ventricular failure became apparent. Dysrhythmias commonly occur as a result of decreased oxygenation and severe damage to greater than 40% of the myocardium.

The nurse notes that a client with sinus rhythm has a premature ventricular contraction that falls on the T wave of the preceding beat. The client's rhythm suddenly changes to one with no P waves, no definable QRS complexes, and coarse wavy lines of varying amplitude. How should the nurse correctly interpret this rhythm? 1. Asystole 2. Atrial fibrillation 3. Ventricular fibrillation 4. Ventricular tachycardia

Ventricular fibrillation Ventricular fibrillation is characterized by irregular chaotic undulations of varying amplitudes. Ventricular fibrillation has no measurable rate and no visible P waves or QRS complexes and results from electrical chaos in the ventricles.

The nurse monitors the client receiving parenteral nutrition (PN) for complications of the therapy and should assess the client for which manifestations of hyperglycemia? 1. Fever, weak pulse, and thirst 2. Nausea, vomiting, and oliguria 3. Sweating, chills, and abdominal pain 4. Weakness, thirst, and increased urine output

Weakness, thirst, and increased urine output The high glucose concentration in PN places the client at risk for hyperglycemia. Signs of hyperglycemia include excessive thirst, fatigue, restlessness, confusion, weakness, Kussmaul respirations, diuresis, and coma when hyperglycemia is severe. If the client has these symptoms, the blood glucose level should be checked immediately. The remaining options do not identify signs specific to hyperglycemia.

A client is receiving total parenteral nutrition (TPN). The nurse monitors the client for complications of the therapy and should assess the client for which manifestations of hyperglycemia? 1. Fever, weak pulse, and thirst 2. Nausea, vomiting, and oliguria 3. Sweating, chills, and abdominal pain 4. Weakness, thirst, and increased urine output

Weakness, thirst, and increased urine output The high glucose concentration in TPN places the client at risk for hyperglycemia. Signs of hyperglycemia include excessive thirst, fatigue, restlessness, confusion, weakness, Kussmaul respirations, diuresis, and coma when hyperglycemia is severe. If the client has these symptoms, the blood glucose level should be checked immediately. The remaining options do not identify signs specific to hyperglycemia.

The nurse, listening to the morning report, learns that an assigned client received a unit of granulocytes the previous evening. The nurse makes a note to assess the results of which daily serum laboratory studies to assess the effectiveness of the transfusion? 1. Hematocrit level 2. Erythrocyte count 3. Hemoglobin level 4. White blood cell count

White blood cell count The client who has neutropenia may receive a transfusion of granulocytes, or WBCs. These clients often have severe infections and are unresponsive to antibiotic therapy. The nurse notes the results of follow-up WBC counts and differential to evaluate the effectiveness of the therapy. The nurse also continues to monitor the client for signs and symptoms of infection. Erythrocyte count and hemoglobin and hematocrit levels are determined after transfusion of packed red blood cells.


Related study sets